CFP CLASS #4 TEST QUESTIONS

Réussis tes devoirs et examens dès maintenant avec Quizwiz!

If Maria and Herman use married filing jointly, with gross income of $330,000 and taxable income of $303,000, what is their effective tax rate? Refer to the 2022 tax table provided in your course references. A) 19.93% B) 22.45% C) 32.00% D) 24.00%

24% x ($303,000 ‒ $178,150) + $30,427 = $29,964 + $30,427 = $60,391 ÷ $303,000 = 19.93%.

Which of these statements is NOT correct regarding cash value life insurance products? A) A MEC is not a life insurance contract. B) Income earned on funds invested in cash value insurance accumulates on a tax-deferred basis. C) Insurance products are a type of tax shelter. D) Proceeds payable before death of the insured are taxable to the extent they exceed the insured's cost basis.

A MEC is a life insurance contract—it must meet one of the two Internal Revenue Code tests for life insurance, and the state law definition. The MEC is a life insurance contract that fails to meet the seven-pay test.

A casualty loss deduction is treated as A) a nondeductible item. B) an itemized deduction. C) an adjustment to income. D) a deduction for AGI.

A casualty loss deduction is treated as an itemized deduction, not a deduction for AGI. It is not deductible in arriving at the adjusted gross income.

Which of the following is the type of audit usually performed for a minor issue? A) Office audit B) None of these types of audits C) Field audit D) Correspondence audit

A correspondence audit is usually performed through the mail because the disputed tax issue is minor.

Chris Burdick anticipates adjusted gross income of $200,000 for the current tax year. He contributed appreciated stock to a public charity. Chris's adjusted basis in this stock is $50,000. The stock has a current fair market value of $140,000. Chris has owned the stock for 12 years. If Chris does gift the stock to the United Way, what is the maximum allowable charitable deduction he can receive in the current tax year? A) $140,000 B) $60,000 C) $100,000 D) $50,000

A gift of long-term capital gain property to a 50% organization is based on the FMV of the property, with the deduction for the current year limited to 30% of AGI.

Which of the following taxpayers may use the married filing jointly filing status? 1. A married couple, even though one spouse did not have any income during the tax year 2. A married couple that is legally separated on the last day of the tax year if they share custody of a dependent child 3. A married couple that is legally separated on the last day of the tax year 4. A married couple that is not legally separated on the last day of the tax year A) I, II, and III B) I, II, and IV C) II and IV D) I and IV

A married couple may file a joint return even though one spouse has no income or deductions if they are not legally separated or divorced on the last day of the tax year.

Which of the following is a public pronouncement that contains official guidance about regulations or interpretations of the Internal Revenue Code (IRC)? A) Private Letter Ruling B) Revenue Procedure C) Revenue Ruling D) Notice

A notice is a public pronouncement that contains official guidance about regulations or interpretations of the Code. The guidance is often substantial but again, ultimately it only points to higher regulations. Notices in and of themselves do not carry the weight of law.

With respect to the at-risk rules, qualified nonrecourse financing is a debt A) from a loan provided by a related person at "below market" terms. B) secured by specific real property. C) for which at least one individual is personally liable. D) that is convertible into an equity interest.

A partner's share of nonrecourse financing established basis in the partnership, but is not treated as an amount at risk. Remember that nonrecourse financing is debt that is secured by the property, but for which no individual has personal liability.

A revenue ruling is A) a judicial interpretation of an Internal Revenue Code provision or a Treasury regulation. B) a judicial interpretation of specific circumstances related to a taxpayer. C) an administrative interpretation of statutory tax law that is generally related to specific circumstances of fact. D) a court's general administrative interpretation of the Internal Revenue Code and regulations.

A revenue ruling is an administrative interpretation of statutory tax law that is generally related to specific circumstances of fact.

Which of the following dispositions of Section 1245 recapture property would result in the immediate recapture of some or all of previous depreciation deductions? A) A sale for cash and an interest-bearing note B) A transfer at death C) A distribution by a partnership to its partners D) A gift of the property

A sale of Section 1245 property at a gain will result in Section 1245 recapture. None of the other choices are considered taxable dispositions that would trigger recapture of depreciation (cost recovery) deductions.

Which of the following statements best describes a weakness of the sole proprietorship form of business? A) The owner has too many partners. B) The owner has no control. C) The business depends solely on the owner. D) The business is in constant danger of bankruptcy.

A sole proprietorship is owned by one individual who is personally liable for the business and all its operations. Therefore, the business depends solely on the owner.

Which one of the following best describes the role of a special allocation in a limited partnership? A) It allows an allocation of items of income and expense that is not pro rata. B) It allocates management responsibility to the general partners. C) It establishes the standards for allocating the proceeds of non-routine or "special" items of income. D) It requires all items to be distributed pro rata based on a partner's capital account balance.

A special allocation allows an allocation of items in a manner that differs from the "normal" pro rata allocation of deduction, income, credit, etc.

Which of the following who do not maintain a household for a dependent must use the single filing status? A) Legally separated taxpayer B) All of these C) Divorced taxpayer D) Unmarried taxpayer

A taxpayer who is an unmarried, legally separated, or divorced individual and does not maintain a household for a dependent must use the single filing status.

Which of these statements are CORRECT with respect to active participation rental real estate? 1.) The interest may be held through a limited partnership. 2.) A deduction-equivalent tax credit of up to $25,000 is available. 3.) The taxpayer must hold a 10% or greater ownership interest. 4.)The taxpayer must participate in the management of the property in a bona fide sense. A) I and II B) III and IV C) I, II, and III D) II and IV

Active participation real estate requires involvement in a bona fide sense. The taxpayer must, at least, make the major management decisions. Also, by definition, the taxpayer must have a 10% or greater ownership interest in the property. Up to a $25,000 loss, not deduction-equivalent tax credit, may be claimed on an annual basis. The interest specifically may not be a limited partnership interest.

Which one of the following is a statute of limitations that restricts the IRS in auditing a return? A) Twelve years if 50% of gross income is unreported B) Ten years for failure to file or if a fraudulent return is filed C) Six years if 25% of gross income is unreported D) Seven years from the filing date of the return or due date if later

After the statute of limitations has passed, except in cases of fraud, the IRS cannot audit a return. The statutes to be aware of are as follows: three years from the filing date of the return or due date if later, six years if 25% of gross income is unreported, and no statute of limitations for failure to file or if a fraudulent return is filed.

Ron and Sandy gave a painting to the local art museum in the current year. The painting had a fair market value of $34,000. He paid $16,500 for it 5 months ago. The museum will display the painting among its collection. What is Ron's charitable contribution deduction? A) $13,800 B) $16,500 C) $34,000 D) $23,000

Because the artwork is appreciated property held for less than one year, its sale would result in a short-term capital gain. Consequently, the deduction is limited to the lesser of the fair market value or the basis of the gifted property.

Trudy is a single taxpayer. Her Aunt Diane has come to live with her after it was determined she could no longer live independently. She is physically challenged and needs full-time care. Diane's deceased parents set up a trust for her that supplies all of her support, including paying for in-home care and all medical bills. Trudy wants to know how this will affect her own income tax situation. What does the planner tell her? A) Diane is Trudy's qualifying relative. B) Trudy's filing status changes to head of household. C) Trudy must file as single. D) Trudy may list Diane as a dependent on her tax return.

Because the trust pays for all of Diane's support, Trudy's income tax return is unaffected and she will file as single.

Beth's husband died in Year 1. Assume that Beth does not remarry and continues to maintain a home for herself and her dependent child during Year 2, Year 3, and Year 4, providing full support for her child throughout those years. For Year 4, Beth's filing status will be A) head of household. B) single. C) qualifying widow. D) married filing jointly.

Beth's Year 4 filing status is head of household. Qualifying widow filing status is only available for 2 years following the death of a spouse (Year 2 and Year 3).

Which of the following statements regarding limited partnerships is CORRECT? 1) A limited partner is subject to the passive activity rules when accounting for income and losses from the limited partnership. 2) The limited partner is liable to the creditors of the partnership only to the extent of that partner's contributed or promised cash or property. A) Both I and II B) II only C) Neither I nor II D) I only

Both of these statements are correct.

Which of the following statements regarding replacement property under Section 1033 is CORRECT? 1) For an owner-user, the taxpayer's use of the replacement property and of the involuntarily converted property must be the same (functional-use test). 2) The owner-investor's properties must be used in similar endeavors as the previously held properties (taxpayer-use test). A) II only B) Neither I nor II C) Both I and II D) I only

Both statements are correct.

Which of the following is a permissible method for determining the business part of a residence for purposes of the home office expense deduction? A) Amount of utility use compared to that of the whole house B) Accrual method of allocation C) Number of rooms in the house used for business compared to total rooms in the house D) Relative use method of allocation

Both the relative square footage of the home office area and the relative number of rooms may be used in determining the business part of a home. The amount of utility use can be a deduction, but is not used in determining the business part of a home. There is no recognized method known as the relative use method of allocation.

Which of the following are preference items or adjustments for purposes of the individual alternative minimum tax? 1. Interest on qualified private-activity municipal bonds issued in 2008 2. Excess of percentage depletion over the property's adjusted basis 3. Investment interest expense in excess of net investment income 4. Qualified housing interest A) I and II B) I, II, III, and IV C) II and III D) I only

By definition, investment interest expense in excess of net investment income and qualified housing interest are not preference items or adjustments for purposes of the alternative minimum tax. Remember that interest on qualified private-activity municipal bonds issued in 2009 and 2010 is not a preference item.

Caroline, age 48, has a filing status of single and she earned a salary of $55,000 in 2022. Her employer also paid $6,000 for health insurance premiums for Caroline. Caroline had a $3,000 capital loss during the year. What is Caroline's adjusted gross income (AGI) in 2022? A) $61,000 B) $49,000 C) $55,000 D) $52,000

Caroline's adjusted gross income (AGI) is $52,000 ($55,000 - $3,000). Caroline's $3,000 capital loss reduces her gross income by $3,000 to $52,000. The health insurance premiums were never included in her income, thus cannot be deducted.

Which of the following sources of authority on a tax issue may be relied upon in tax research? 1) Treasury Regulation 2) Tax Court opinion 3) United States Supreme Court opinion A) I and III B) II and III C) I, II, and III D) I and II

Congress has authorized the Secretary of the Treasury to prescribe and issue all rules and regulations needed for enforcement of the Code. Regulations can be classified into three groups: (1) legislative (Treasury Regulation), (2) interpretive (Tax Court opinion), and (3) procedural (United States Supreme Court opinion).

Which of the following is NOT an advantage of the cash basis method of accounting? A) Taxpayers can keep simple records. B) Constructive receipt serves to defer income. C) Taxpayers have more control over each year's income and expenses. D) Income is reported when it is received.

Constructive receipt, which serves to accelerate income, is not considered an advantage of the cash method of accounting.

LMN Corporation owns 5% of the outstanding stock of the STU Corporation. During the current year, LMN receives $10,000 of dividends from STU Corporation. Both corporations are domestic corporations. Which of the following correctly identifies the tax treatment of the dividends received by LMN? A) The dividends may be 80% excluded. B) The dividends may be 50% excluded. C) The dividends are fully included. D) The dividends received are fully excluded.

Fifty percent of the dividends received from a qualifying corporation may be excluded if the recipient corporation owns 20% or less of the distributing corporation; there is a 65% exclusion with 20% to 80% ownership; and there is a 100% exclusion if the recipient corporation has over 80% ownership.

Which of the following are includible in an individual's gross income for income tax purposes? Gambling winnings Inheritances Interest collected by the taxpayer on federal obligations Scholarships and fellowships in degree programs A) I and III B) I only C) I, III, and IV D) III and IV

Gambling winnings and interest on federal obligations are includible in an individual's gross income for income tax purposes. The other items are not subject to income taxation.

George, whose wife died last November, filed a joint tax return for last year. He did not remarry after his wife's death and has continued to maintain his home for his two dependent children. In the preparation of his tax return for this year, what is George's filing status? A) Head of household B) Married filing separately C) Single D) Qualifying widower

George filed a joint return in the year of his wife's death. He can file as a qualifying widower (also known as surviving spouse) for the two years following his wife's death if he continues to maintain a home for his dependent children.

Which of the following statements regarding alimony paid under a 2022 divorce agreement is CORRECT? A) The divorced couple may be members of the same household at the time the alimony is paid. B) Alimony is deductible by the payor spouse, and includible in income by the payee spouse, to the extent that the payment is contingent on the status of the divorced couple's children. C) Alimony may be paid in either cash or property. D) Alimony payments must terminate on the death of the payee spouse.

Payments to former spouses are no longer deductible and are considered alimony only if: - the payments are made in cash (and not property); - the decree does not specify that the payments are not alimony for federal income tax purposes; - the payor and payee are not members of the same household at the time that the payments are made; and - there is no liability to make the payments for any period after the death of the payee.

Philip, a professor, earned a salary of $140,000 from a university in the current year. He received $35,000 in dividends and interest during the year. In addition, he incurred a loss of $25,000 from an investment in a passive activity. Assuming Philip's at-risk amount in the activity at the beginning of the current year was $15,000, what is his AGI for the current year? A) $160,000 B) $175,000 C) $150,000 D) $115,000

Philip's AGI, after considering the passive investment (and loss), is $175,000. This consists of $140,000 of active income and $35,000 of portfolio income. Philip cannot deduct the passive loss of $25,000 against either active or portfolio income. In addition, he is further restricted to a total possible loss of only $15,000 because of the at-risk rules.

Which one of the following is a publication of specific taxpayer guidance from the IRS? A) Regulations B) Private Letter Rulings C) Revenue Rulings D) Revenue Procedures

Private Letter Rulings are taxpayer guidance from the IRS that apply only to the particular taxpayer(s) asking for the ruling; they are not applicable to all taxpayers. The primary purpose of the regulations is to explain and interpret particular IRS Code sections.

Your client, Elaine Dell, is near the highest tax bracket and is contemplating several investments. She is, however, concerned about minimization of her federal income tax liability on the income from the investment. Which of the following investments would produce income that would be taxed at the lowest potential tax rate? A) A certificate of deposit B) A zero coupon bond C) A utility stock with a high dividend yield D) A corporate bond fund

Qualified dividends are generally taxed at a 15% rate (or 20% for taxpayers with higher income levels). All of the other options produce interest income, which is taxable as ordinary income, at the marginal rate of the taxpayer.

If Rachel files single with gross income of $90,000 and taxable income of $76,000, what is her marginal tax rate based on the following tax information? A) 22.00% B) 14.47% C) 24.00% D) 17.49%

Rachel's taxable amount of $76,000 puts her in the marginal tax rate of 22%.

The hierarchy of sources published by the IRS from most authoritative to least is as follows:

Regulations Revenue Rulings Revenue Procedures Private Letter Rulings Technical Advice Memorandums Notices Announcements

Which one of the following is NOT a group of the IRS Regulations? A) Legislative B) Interpretive C) Implementative D) Procedural

Regulations can be classified into three groups: (1) legislative, (2) interpretive, and (3) procedural.

Which of these is allowable in the computation of total income on the Form 1040? A) Penalty on the early withdrawal of savings B) Net capital losses of up to $3,000 C) Investment interest expense D) Deductible IRA contributions

Remember that the total income is the starting point for the federal income tax calculation on the Form 1040. The total income is essentially all income that is taxable reduced by several deductions, such as net capital losses up to $3,000, rental losses allowable under the active participation rental real estate provision, Schedule C losses, and other business losses that flow through and are deductible by the taxpayer. It is the amount before the deduction for adjustments to income. The deductible IRA contribution and the penalty on the early withdrawal of savings are adjustments to income. Investment interest expense is an itemized deduction.

Jack and Emily are legally separated on December 31 this year. Jack earned $40,000 this year, and Emily earned $80,000. They live in a common law state and have no dependents. They have come to a tax preparer to determine how they must file their income taxes this year. What does the planner tell them? A) They must file as single, each reporting $60,000 (half of the total of $120,000) in income. B) They must file as single, each one reporting their own income. C) They must file as MFS, each reporting their own income. D) They may file as MFJ as they were not legally separated until the end of the year.

Single (S) filing status is used by an unmarried, legally separated, or divorced individual who does not qualify for any other filing status.

Which of these is the 2022 threshold on earnings for the additional Medicare tax for a single taxpayer? A) $127,200 B) $250,000 C) Unlimited D) $200,000

Single taxpayers are subject to the additional Medicare tax when earned income exceeds $200,000 in the tax year.

Which one of the following is NOT a taxation function of the social objective? A) Charitable deduction B) Excluding life insurance proceeds from taxation C) Renovation of a historic home D) Reduction of taxes to stimulate the economy

Social objectives of the federal taxation system include the charitable deduction, excluding life insurance proceeds from taxation, and renovation of a historic home. The reduction of taxes in order to stimulate the economy is due to the economic objective.

Molly's grandparents gifted her with substantial securities at her birth eight years ago. In 2022, she has dividends of $10,000 and brokers' fees of $800 on the activity in the account her parents manage for her. What is her net unearned income taxed at her parents' rate? A) $7,700 B) $10,000 C) $9,100 D) $8,850

Some of Molly's unearned income is taxed at her parents' rate and is calculated as follows: $10,000 UI - $1,150 (standard deduction) - $1,150 (greater of $1,150 for 2022 or amount of allowable itemized deductions directly connected with the production of the unearned income) = $7,700

Which of the following statements regarding accounting periods is CORRECT? All taxpayers may report taxable income on the basis of a calendar year. If a partner has a different taxable year than that of the partnership, the partner must report his share of the entity's income in the same taxable year within which the entity's taxable or fiscal year begins. A) II only B) I only C) Both I and II D) Neither I nor II

Statement I is correct. All taxpayers may report taxable income on the basis of a calendar year. Such a year is a 12-month period ending on December 31. Statement II is incorrect. If a partner or S corporation shareholder/owner has a different taxable year than the business entity, the owner must report his share of the entity's income in the same taxable year within which the entity's taxable or fiscal year ends.

Ron, age 43, and Sandy, age 41, are married with two children, Michael, age 12, and Victoria, age 8, who has been blind since her birth. Ron is an architect and general partner with XYZ partnership. Sandy is self-employed as an attorney and works out of a home office. Her home office is exclusively and regularly used for business and is her principal place of business. Their information for the tax year 2021 is as follows: Adjusted gross income: $217,300 Itemized deductions (including qualified residential mortgage interest,taxes paid, and charitable contributions): $33,000 Early in the current year, Sandy's father died. Sandy is the sole beneficiary of her father's entire estate. The estate is presently in the probate process. Sandy's mother, Lisa, age 68, has moved in with them but provides her own support. She was married to Sandy's father when he died earlier this year. This is Ron's second marriage. He makes monthly support payments to his former wife and his daughter. Because both Ron and Sandy are considered to be self-employed, they make quarterly estimated tax payments each year to cover both their income tax and self-employment tax obligations. Based on the information provided in the case scenario for Ron and Sandy, which of the following statements regarding Ron's status as a partner with XYZ partnership is CORRECT? 1) Ron has the right to participate in the management and operation of the business. 2) Ron has no personal liability for the acts of the partnership or the other partners.

Statement I is correct. As a general partner, Ron has the right to participate in the management and operation of the business. Statement II is incorrect. As a general partner, Ron has unlimited personal liability for the acts of the partnership and other partners.

Courtney and Della are considering obtaining a home equity line of credit of $50,000. They will use some of the proceeds to make needed improvements to their personal residence. Della is concerned about the deductibility of the interest. Which of the following statements is(are) CORRECT? 1). Home equity interest is not deductible to the extent used for other than home acquisition or improvements. 2). All of the home equity loan interest will be deductible for the couple. A) Neither I nor II B) II only C) I only D) Both I and II

Statement I is correct. Home equity loan interest is not deductible on a taxpayer's income tax return to the extent it is used for other than home acquisition or improvements for the home that secure the mortgage. The interest on the funds used for home improvements is deductible.

Which of the following elements is NOT required for alimony? 1. The divorce agreement must specify that the payments are alimony. 2. The payor and payee are not members of the same household at the time the payments are made. 3. There is no liability to make the payments after the payee's death. 4. The payments are not for the support of the payor's children. A) II, III, and IV B) II and III C) I, II, and IV D) I only

Statement I is correct. It is not necessary to specify that payments are alimony. However, the divorce agreement cannot specify that the payments are not alimony.

The kiddie tax rules apply 1) to the unearned income of a child under age 19 (under age 24 if a dependent full-time student providing less than 50% of own support). 2) to the earned income of a child under age 19 (under age 24 if a dependent full-time student providing less than 50% of own support). A) I only B) II only C) Both I and II D) Neither I nor II

Statement I is correct. The kiddie tax applies only to any unearned income (such as dividends and interest) of a child under age 19 (under age 24 if a dependent full-time student providing less than 50% of own support). For a dependent on another's income tax return at any age, the standard deduction for earned income is the amount of earned income plus $400, but limited to an amount no greater than the standard deduction for a single taxpayer ($12,950 in 2022).

Which of the following itemized deductions would be adjustments to regular taxable income in arriving at alternative minimum taxable income (AMTI)? 1. Casualty losses 2. State income taxes paid 3. Standard deduction 4. Charitable donation made to the local university A) II and III B) I and III C) II, III, and IV D) I and IV

Statement I is incorrect because casualty losses are deductible for both regular tax and AMT; no adjustment is necessary. Statement II is correct because state taxes are not deductible for AMT purposes. Statement III is correct because the standard deduction is a positive adjustment when calculating AMT income. Statement IV is incorrect because charitable contributions are deductible for both regular income tax and AMT; no adjustment is necessary.

Which of the following statements regarding charitable deductions by corporations is CORRECT? 1. The corporate statutes of most states permit corporations to make charitable contributions, and the Tax Code permits a charitable deduction for contributions by a corporation. 2. The charitable deduction is limited to a maximum of 20% of the corporation's adjusted taxable income. In the event the contribution is in excess of 20% of the corporation's adjusted taxable income, the balance can be carried forward for up to five years. A) Neither I nor II B) I only C) II only D) Both I and II

Statement II is incorrect because the charitable deduction is limited to a maximum of 10% of the corporation's adjusted taxable income.

Which of the following statements regarding Section 1033 involuntary conversions is CORRECT? 1) For an owner-user, the replacement property must pass the functional use test. 2) The taxpayer use test provides less flexibility than the functional use test. A) I only B) Neither I nor II C) Both I and II D) II only

Statement II is incorrect. The taxpayer use test provides more flexibility than the functional use test.

Which of the following rules regarding the sale of Section 1231 property is CORRECT? 1) When Section 1231 property is sold for more than the purchase price, the gain is afforded capital gain treatment and taxed using capital gain tax rates. 2) When Section 1231 property is sold at a loss, the loss is treated as a capital loss. A) II only B) I only C) Neither I nor II D) Both I and II

Statement II is incorrect. When Section 1231 property is sold at a loss, the loss is treated as an ordinary loss, not a capital loss.

All of the following are tax avoidance techniques except A) investing in zero-coupon U.S. Treasury bonds. B) investing in tax-free municipal bonds. C) utilizing tax credits such as qualifying child care expenses. D) owning, rather than renting, a residence to benefit from claiming a home mortgage interest deduction.

Taxes must be paid on accrued interest on zero-coupon Treasury bonds even though no cash income is received. The other choices are legitimate tax avoidance techniques.

hich of the following forms does a C corporation file? A) Form 1065 B) Form 1120 C) Form 1120-S D) Form 1120-C

The C corporation files a tax return with the IRS—this is the Form 1120.

Which of these is NOT a goal of the federal income tax system? A) Economic growth B) Monetary policy C) Inflation control D) Price stability

The Federal Reserve System controls monetary policy. Income tax policy is used to influence all of the other goals.

Which one of the following is NOT a goal of the federal income tax system? A) Price stability B) Monetary policy C) Inflation control D) Economic growth

The Federal Reserve System controls monetary policy. Income tax policy is used to influence all of the other goals.

Your client, Mary, is contemplating the formation of an S corporation for her small manufacturing business. What can you accurately tell Mary about forming an S corporation? Mary would have limited liability. The number of shareholders is limited to 100. Mary's death would require reorganization of the corporation. The corporation's items of income, deductions, and tax credits would flow through to Mary's income tax return. A) III only B) I and IV C) I, II, and IV D) II and III

The S corporation form is similar to the partnership in operating as a conduit for tax purposes. It also is similar to a C corporation, since it features limited liability for shareholders. In addition, the death of a shareholder in an S corporation does not create a need to reorganize the business. Finally, unlike a C corporation, an S corporation may have no more than 100 shareholders.

ABC Corporation purchased various items of depreciable tangible personal property, with a total cost of $3.26 million, for use in its business during 2022. Assume that ABC has taxable income (without regard to the Section 179 expense) of $290,500 this year. What is the maximum Section 179 deduction that ABC may claim in the current year? A) $1,080,000 B) $3,260,000 C) $290,500 D) $520,000

The Section 179 election is first reduced by the amount that property placed in service during the tax year exceeds $2.7 million. The $560,000 excess reduces the maximum election to $520,000 ($1.08 million maximum Section 179, reduced by the $560,000 excess). The maximum deduction is then limited to taxable income (or earned income), which in this case is $290,500.

Which of the following tax factors may limit the availability of tax benefits from a limited partnership? I. "Passive loss" rules II. At-risk rules III. Alternative minimum tax A) I only B) I, II, and III C) III only D) I and II

The Tax Reform Act of 1986 has two major provisions that have significantly reduced the benefit of tax-shelter investments: the at-risk rules and the passive activity loss rules limits. The alternative minimum tax is another example of a tax benefit limitation.

If the Taylors rented their condo for 300 days last year, how many days could the Taylors have used it personally and still maintained the classification of the condo as primarily a rental use property? A) 30 days B) 0 days C) 14 days D) 3 days

The Taylors could have used the property personally for a total of 30 days (300 rental days × 10%). If rental property is rented at least 15 days a year and is not used for personal use more than the greater of 14 days per year or 10% of the rental days, it is classified as primarily rental use. This treatment permits the Taylors to deduct expenses associated with the rental (not personal) use on Schedule E of IRS Form 1040.

In April of the current year, Mel sold his principal residence for a total price of $408,000. Mel sold the house due to a job transfer out of state. He received the house as a gift 18 months ago when its fair market value was $375,000, and the donor's basis was $50,000. He completed a room addition for $20,000. He paid realtor commissions of $23,000 on the sale. What amount, if any, must be recognized on the sale of Mel's residence? A) $0 B) $315,000 C) $127,500 D) $13,000

The allowable exclusion is the full exclusion of $250,000 multiplied by 75%—18 months of use divided by the 24-month requirement. The basis is the $50,000, increased by the improvement of $20,000. The donor's basis is used as the carryover basis. The gain recognized is the amount of gain that is subject to tax.

Mary is a single taxpayer, age 67. She has the following itemized deductions: Home mortgage interest (first mortgage)$15,950 State income taxes$3,120 Property taxes$1,480 Charitable contributions$2,000 Gambling losses$1,500 Unreimbursed employee business expenses$4,600 Tax return preparation fee$400 Medical expenses$18,118 Mary's AGI for 2022 is $318,000. Included in the AGI is $500 of gambling winnings. What is the amount of her allowable itemized deductions? A) $23,050 B) $23,280 C) $22,820 D) $44,250

The allowable itemized deductions total $23,280. The unreimbursed employee business expenses are not deductible. The medical expenses are deductible only to the extent that they exceed 7.5% of AGI. That 7.5% of AGI is $23,850. The medical expenses exceed that amount by $230. Home mortgage interest (first mortgage)$15,950State income taxes$3,120Property taxes$1,480Charitable contributions$2,000Gambling losses (to extent of winnings)$500Medical expenses$230Allowable itemized deductions$23,280

Paul died recently. At the time of his death, he had $12,000 in suspended losses from a limited partnership, a passive activity. His basis in the limited partnership was zero, and the step-up in basis at death was $10,000. What is the amount of suspended losses that are deductible? A) $0 B) $12,000 C) $10,000 D) $2,000

The answer is $2,000, the suspended losses ($12,000) minus the basis ($10,000).

Kevin Riley anticipates adjusted gross income of $120,000 for the current tax year. He has made no charitable gifts during the year, but now he wants to give his church a stamp collection with a fair market value of $70,000. Kevin paid $38,000 for the collection five years ago. The collection is appreciated tangible personal property that is unrelated to the church's exempt function. What is the maximum allowable charitable deduction Kevin can receive during the current year if he makes an immediate gift of the stamp collection? A) $24,000 B) $38,000 C) $60,000 D) $36,000

The answer is $38,000. Since this is use-unrelated property, the allowable deduction is limited to his basis.

Mary filed her 2020 income tax return on April 3, 2021. The IRS has recently determined that she worked a part-time job, but the employer failed to provide Mary with a W-2. Mary honestly did not realize that she was required to report the income, because she did not receive a W-2. The IRS has determined that her negligent failure to report the income resulted in an additional income tax liability of $2,000. What is Mary's tax penalty? A) $400 B) $200 C) $100 D) $1,000

The answer is $400. The negligence penalty is 20% of the deficiency due to the taxpayer's negligence. For the $2,000 tax deficiency, 20% results in a negligence penalty of $400. It may be argued that the failure to report the income was fraud, but the fact pattern states that the act was merely negligent.

In 2022, Jim had an AGI of $160,000. What percentage of prior-year income tax must be paid by Jim in 2022 to avoid the imposition of a penalty for underpayment of estimated tax? A) 80% B) 90% C) 110% D) 100%

The common exception for avoiding estimated tax penalties generally is 90% of the current year's tax or 100% of the prior year's tax. However, if the prior year's AGI exceeds $150,000, then the requirement is 90% of the current year tax or 110% of the prior year's tax liability.

Assume that married taxpayers filing jointly have a taxable income of $470,650. What is the taxpayers' effective tax rate? You will need to use the tax rate schedule found in your materials. A) 47% B) 23.8% C) 35% D) 26.8%

The effective income tax rate is the amount of income tax ($112,234) divided by the taxable income of $470,650. This gives us 23.8% (rounded). Taxable income $470,650 Less (from tax rate schedule). (431,900) Amount over $431,900. $38,750 Times (marginal rate, from tax rate schedule). 35% Tax on amount over $431,900 $13,563 Plus (from tax rate schedule) 98,671 Total Tax $112,234

Sam had the following items of income during the 2021 tax year: Sole proprietorship net income: $30,000 Interest income: $2,000 Flow-through of S corporation income: $10,000 What is the amount of self-employment tax Sam owes? A) $4,239 B) $4,400 C) $4,590 D) $6,120

The equation is $30,000 - 7.65% = $27,705 × 15.3% = $4,239. Remember that the first step in computing the self-employment tax is always the 7.65% subtraction. The flow-through of income from an S corporation (or limited partnership income to a limited partner) is not considered to be self-employment income.

Which one of the following is a CORRECT hierarchy of authority of sources published by the IRS from least authoritative to most? A) Technical Advice Memorandums, Private Letter Rulings, Revenue Rulings B) Regulations, Technical Advice Memorandums, Private Letter Rulings C) Technical Advice Memorandums, Announcements, Revenue Procedures D) Revenue Procedures, Revenue Rulings, Notices

The hierarchy of sources published by the IRS from most authoritative to least is as follows: Regulations Revenue Rulings Revenue Procedures Private Letter Rulings Technical Advice Memorandums Notices Announcements

All of the following statements regarding the installment method of reporting gain from a disposition of property are correct except A) an installment sale is a sale of property in which the seller receives at least one payment after the year of sale. B) the installment method permits the seller to spread out the taxable gain over more than one year. C) the payments received under an installment sale may each include capital gains, return of capital, and interest. D) the installment sale method may be used for securities sold in the secondary market.

The installment sale method cannot be used for inventory or securities traded in the secondary market.

Michelle has interest and short-term capital gain income of $9,000 during the current tax year. She paid brokers' commissions of $1,000, investment advisers' fees of $2,200, and had $7,700 of interest expense on funds borrowed to purchase securities. Michelle has an AGI of $105,000. What amount of investment interest expense may be deducted as an itemized deduction? A) $7,700 B) $5,800 C) $6,700 D) $8,200

The investment interest expense ($7,700) is deductible up to the amount of the net investment income. The net investment income is simply the investment income of $9,000. Investment interest expense may be deducted up to the amount of investment income—$7,700 in this situation. The broker's commissions do not enter into this calculation, nor do the advisor's fees. They are not a deductible expense, and they are not part of the investment income.

During the current tax year, Rod purchased a building for exclusive use in his manufacturing business. The cost of the property was $422,000, of which $122,000 was attributable to the land. Which of the following statements identifies the proper treatment of the expenditure? A) The $122,000 must be capitalized and may not be depreciated. B) The $300,000 attributable to the building may be currently deductible. C) The $122,000 must be capitalized and may be depreciated. D) The cost attributable to the building may be deducted under Section 179.

The land may not be depreciated, as only "wasting" assets are subject to depreciation. The Section 179 expense election generally applies to personalty only, and is not available for most real estate. The cost of the building may not be currently deducted; it must be capitalized and depreciated because it has a useful life of over one year.

What is the lowest marginal income tax rate for a taxpayer in 2022? A) 5% B) 10% C) 15% D) 0%

The lowest marginal income tax rate is 10% for all taxpayers in 2022. The 0% rate is the rate at which it is possible for capital gains to be taxed, depending on the taxpayer's income.

Matthew Brady, age 67, purchased a deferred annuity in January 1982 for $50,000. In the current year, when the surrender value was $125,000, Matthew took a nonperiodic distribution of $75,000. Which one of the following statements correctly describes the income tax consequences of the distribution? A) $50,000 is taxable, $25,000 is tax free. B) $75,000 is taxable income. C) $75,000 is tax free. D) $50,000 is tax free, $25,000 is taxable.

The pre-August 14, 1982, annuity retains first-in, first-out (FIFO) treatment. Thus, the basis of $50,000 is treated as being withdrawn first and is tax free. The remaining $25,000 is taxable. If this were a post-August 13, 1982, contract, it would be treated on a last-in, first-out (LIFO) basis.

Which one of the following is primarily used to explain and interpret particular IRS Code sections? A) Private Letter Rulings B) Revenue Procedures C) Revenue Rulings D) Regulations

The primary purpose of the regulations is to explain and interpret particular IRS Code sections. Private Letter Rulings are taxpayer guidance from the IRS that apply only to the particular taxpayer(s) asking for the ruling; they are not applicable to all taxpayers.

Which one of the following is NOT a federal taxation function of the economic objective? A) Restricting spending through greater taxation B) Promoting full employment C) Reduction of taxes during a recession to stimulate the economy D) Charitable deduction

The reduction of taxes in order to stimulate the economy is due to the economic objective, as are restricting spending and promoting full employment. Social objectives of the federal taxation system include the charitable deduction, excluding life insurance proceeds from taxation, and renovation of a historic home.

Carl and Janet are married taxpayers filing a joint tax return. In 2022, their AGI is $360,000, and their net short-term capital gain and dividend income (included in the AGI) is $90,000. They have investment interest expense of $4,000 and state and local income taxes attributable to the investment income of $6,000. What is the amount of Medicare contribution tax that they must pay? A) $4,180 B) $3,420 C) $3,040 D) $3,800

They will pay the 3.8% Medicare contribution tax on $80,000. This is the lesser of the net investment income ($80,000) or the AGI in excess of the threshold amount ($360,000 - $250,000, or $110,000). The net investment income is the investment income of $90,000, reduced by the investment expenses of $10,000. In this situation, the $80,000 of net investment income is subject to the Medicare contribution tax. Carl and Janet will pay a $3,040 Medicare contribution tax (3.8% on $80,000).

Sharon and Oliver South are a married couple filing jointly, with one dependent child. For the 2022 tax year, they have the following items relevant to their income tax situation: Wages. $100,000 Sole proprietorship net income. $10,000 Alimony paid to Oliver's former spouse. $18,000 Child support paid. $12,000 IRA contribution. $6,000 Self-employment tax liability. $1,413 Net capital loss. $4,200 Child tax credit. $2,000 Unreimbursed medical expenses. $17,000 Oliver's divorce decree was finalized in 2015. Neither spouse is an active participant in a company-maintained retirement plan. What is the amount of the Souths' AGI? A) $83,500 B) $70,500 C) $82,293 D) $82,500

Wages. $100,000 Schedule C income 10,000 Net capital loss (3,000) Total income $107,000 Alimony paid (18,000) IRA contribution ($6,000) ½ self-employment tax (707) AGI $82,293 Remember that net capital losses are only deductible up to $3,000 in a given year. CHILD SUPPORT PAYMENTS are NEITHER DEDUCTIBLE by the payor NOR TAXABLE to the recipient. The child tax credit is deducted after computing the income tax (a tax credit is a dollar-for-dollar reduction of the tax liability). Medical expenses are an itemized deduction, which are deducted after computing AGI (a below-the line deduction, or deduction from AGI).

As part of a 2021 divorce decree, Patrick was required to transfer all of his existing life insurance policies to his former spouse, Marlene. She became the owner and beneficiary of the policies. Patrick is now required to continue making the premium payments on all of the policies. How will the life insurance premium payments be treated? A) They are included as taxable income on Marlene's return. B) They are considered alimony. C) They are included as tax deductions. D) They are included as income.

When a taxpayer transfers ownership and control of life insurance policies to a former spouse, any premium payments that the taxpayer makes on those policies are considered alimony to the ex-spouse.

Alternative minimum taxable income (AMTI) is calculated using which of the following methods? A) Adjusted gross income (from IRS Form 1040) + positive AMT adjustments − negative AMT adjustments + AMT preference items = AMTI B) Regular taxable income (from IRS Form 1040) − positive AMT adjustments + negative AMT adjustments + AMT preference items = AMTI C) Regular taxable income (from IRS Form 1040) + positive AMT adjustments − negative AMT adjustments + AMT preference items = AMTI D) Modified adjusted gross income (from IRS Form 1040) + positive AMT adjustments − negative AMT adjustments + AMT preference items = AMTI

regular taxable income (from IRS Form 1040) + positive AMT adjustments − negative AMT adjustments + AMT preference items = AMTI

What percentage, if any, of a taxpayer's self-employment tax may be deductible as an adjustment to income? A) 50% B) 0% C) 75% D) 25%

Only half (50%) of a taxpayer's self-employment tax liability is deductible as an adjustment to income.

If Leslie and Armando file as married filing jointly, with gross income of $176,000 and taxable income of $158,000, what is their marginal tax rate based on the following tax information? Refer to the 2022 tax table provided in your course references. A) 22.00% B) 16.67% C) 14.97% D) 24.00%

The couple's marginal tax rate is 22%.

Which of these statements regarding long-term capital gains rates is NOT correct? A) Net long-term capital gains are generally subject to a maximum rate of 15% or 20%. B) A 25% maximum capital gains rate applies to unrecaptured Section 1250 income. C) A 28% maximum capital gains rate applies to the gain on collectibles held for more than one year. D) The long-term capital gain holding period is one year.

The long-term capital gain holding period is more than 12 months. The other statements are correct.

Which one of the following statements correctly describes the income tax consequences upon the disposition of a passive activity? A) The suspended losses are deductible against any income upon the sale of the entire interest. B) The suspended losses are deductible in full if more than 50% of the interest in the activity is sold. C) The suspended losses are nondeductible. D) If a taxpayer sells 20% of the interest in an activity, then 20% of the suspended losses are deductible against any other income.

The suspended losses are deductible against any income upon the sale of the entire interest.

A taxpayer has Schedule C net income of $60,000. What is the taxpayer's total self-employment tax? Round your answer to the nearest dollar. A) $8,478 B) $4,239 C) $9,180 D) $3,825

The total self-employment tax is $8,478 ($60,000 × 0.9235 = $55,410; $55,410 × 0.1530 = $8,478).

Nicholas and Ashley are married and file a joint tax return. Nicholas earns $80,000 annually, and Ashley earns $90,000 each year. What percentage of the tax liability does each spouse owe? A) 100% for each spouse B) Ashley has the highest salary; therefore, she owes 100% C) An amount based on each spouse's percentage of income D) 50% for each spouse

Under the rule of joint and several liability, each spouse owes 100% of the tax liability, no matter who earned the income or in what proportion.

If Stewart and Hope file married filing separately and Hope has gross income of $300,000 and taxable income of $267,000, what is her marginal tax rate? Refer to the 2022 tax table provided in your course references. A) 35.00% B) 22.75% C) 25.56% D) 37.00%

When using the MFS status, Hope is in the 35% marginal tax bracket.

Kurt and Allison Long are married and file a joint income tax return. Their adjusted gross income (AGI) is $180,000 per year. On last year's tax return, the Longs claimed a $1,200 credit for child care expenses. The Longs are in the 22% marginal income tax bracket. What amount of deductions for AGI would be required to equal the tax benefit of the $1,200 child care credit? A) $1,538 B) $264 C) $5,455 D) $936

$1,200 divided by the 22% marginal income tax bracket gives us $5,455.

In 2022, Lonnie, age 17, is claimed as a dependent on his parents' income tax return and has $3,200 of interest and short-term capital gain income from an UTMA account that was established many years ago. Lonnie's parents, who file jointly, have taxable income of $130,000 (22% MFJ tax bracket). What is Lonnie's income tax liability for 2022? A) $313 B) $505 C) $360 D) $565

$3,200unearned income($1,150)standard deduction($1,150)taxed at child's rate of 10%: $1,150 × 10% =$115$900taxed at parents' top marginal tax rate of 22%: $900 × 22% =$198Total$313 The first $1,150 of unearned income is sheltered by the child's limited standard deduction. The next $1,150 is taxed to the child, at the child's marginal income tax bracket. All unearned income in excess of $2,300 (for 2022) is taxed to the child at the parents' top marginal tax rate. The parents' tax rate is 22% up to $178,150 of taxable income (rate schedule provided in textbook or downloaded from this online course dashboard). The kiddie tax applies to children who are under 19 years of age or who are under 24 if a full-time student. (A full-time student is an individual who is a full-time student for at least five calendar months during the tax year.) The kiddie tax does not apply to a child who is married and files a joint return for the tax year, or if the child has earned income that exceeds half of his support. Also, the kiddie tax applies only where the child has at least one living parent.

Terry and Jan are married taxpayers filing a joint tax return. Their AGI is $310,000, and their net investment income (included in the AGI) is $90,000. What is the amount of the Medicare contribution tax for the current tax year? A) $3,420 B) $2,280 C) $5,180 D) $4,180

$310,000 - $250,000 = $60,000. $60,000 × 3.8% = $2,280. $60,000 < $90,000.

If Leslie and Armando file married filing jointly, with gross income of $176,000 and taxable income of $158,000, what is their effective tax rate? Refer to the 2022 tax table provided in your course references. A) 14.97% B) 24.00% C) 22.00% D) 16.45%

22%($158,000 ‒ $83,550) + $9,615 = $16,379 + $9,615 = $25,994 ÷ $158,000 = 16.45%.

If Jason files single with gross income of $110,000 and taxable income of $91,000, what is his effective tax rate based on the tax rate schedule provided in your course references? A) 22.0% B) 17.2% C) 24.0% D) 14.4%

24% x ($91,000 ‒ $89,075) + $15,214 = $462 + $15,214 = $15,676 ÷ $91,000 = 17.2%.

All of the following are permitted to represent taxpayers before the IRS in an audit proceeding except A) CFP® certificants without additional credentials. B) enrolled agents. C) licensed attorneys. D) certified public accountants.

A CFP® certificant without additional credentials is not permitted to represent taxpayers before the IRS in an audit.

Which of the following statements regarding installment payments pursuant to a divorce is CORRECT? It is usually in the best interest of the payor to spread the amount of payments over as many years as possible to take advantage of the time value of money. It is usually in the payee's best interest to immediately receive as much money as possible rather than receiving payments over a longer period. A) I only B) Both I and II C) Neither I nor II D) II only

A common issue that arises in the settlement of divorce proceedings is how payments are best structured in resolution of marital obligations. As a general rule, it is usually in the best interest of the payor to spread the amount of payments over as many years as possible to take advantage of the time value of money. Alternately, the payee's best interests are served by immediately receiving as much money as possible rather than receiving payments over a longer period.

Michael was divorced after 12 years of marriage and recently died. He had two dependent children, ages four and six, who are cared for by their mother. He was currently, but not fully, insured under Social Security at the time of his death. The benefits that his survivors are entitled to include which of the following? 1. A lump sum death benefit of $255 2. A children's benefit 3. A divorced spouse's benefit 4. A parent's benefit for deceased workers' parents who are over the age of 62 A) I and II B) I, II, III, and IV C) III and IV D) I, II, and III

A lump sum death benefit of $255 is payable to the surviving spouse or children of the deceased worker if he was fully or currently insured. The children's benefit is payable because Michael was either currently or fully insured. The mother of the children would be entitled to a benefit because she is caring for Michael's children who are under the age of 16. However, that is not called the divorced spouse benefit. It is the surviving spouse caring for a dependent child benefit. The parents are not entitled because Michael was not fully insured. Statements III and IV are benefits only available under fully insured status.

Grant and Kelly, who file as married filing jointly, sold Section 1244 stock for a $122,000 loss in the current year. What is the character of this loss for the year? A) $122,000 capital B) $100,000 ordinary and $22,000 capital C) $50,000 ordinary and $72,000 capital D) $122,000 ordinary

A married couple filing jointly who sells Section 1244 stock can treat $100,000 of the loss as ordinary. The remaining loss is a capital loss, which may be netted against capital gains in the year of the loss. Also, in the year of loss, any remaining capital loss after netting against capital gains is deductible against ordinary income as a capital loss to a maximum of $3,000, with any remaining capital loss amounts carried forward to future years and treated as any other capital loss.

Five years ago, Greg Young, age 55, purchased a deferred annuity that is estimated to pay him $900 per month for the rest of his life, beginning at age 65. His investment in the contract was a one-time payment of $150,000. The assumed rate of return on the contract is 4%. Which one of the following is an income tax implication of the deferred annuity for Greg? A) Tax-free nonperiodic distributions are allowed for education or family illness. B) Withdrawals in a nonperiodic distribution are first allocated to the tax-free investment in the annuity. C) Earnings within the annuity are taxable in full each year to Greg as ordinary income. D) The annuitized distribution amount consisting of Greg's investment in the contract is a tax-free return of capital.

A non-periodic distribution or withdrawal from a post-August 13, 1982, annuity contract is treated on a last-in, first-out (LIFO) basis. In other words, to the extent that the cash surrender value exceeds the investment in the contract, taxable interest income is treated as being withdrawn first. There are no provisions that allow for a tax-free withdrawal for education or illness. The earnings on the investment in a commercial annuity are deferred—there is no current taxation on the earnings within the contract as long as an individual is the owner (or treated as an owner) of the contract.

Nine years ago, Claire, age 55, purchased a deferred annuity that is estimated to pay her $850 per month for the rest of her life beginning at age 65. Her investment in the contract is a one-time payment of $50,000. The assumed rate of return on the contract is 3.5%. At this time, Claire is not sure whether she will need to withdraw any of her original investment prior to the starting date of the annuity. Which of these is an income tax implication of the deferred annuity for Claire? A) The distribution amount consisting of interest paid on the investment is taxed as a capital gain. B) Withdrawals in a lump sum are first allocated to the tax-free investment in the annuity (FIFO treatment). C) Earnings on the investment are taxable in full each year to Claire as ordinary income. D) A nonperiodic (lump-sum) distribution will be treated on a last-in, first-out (LIFO) basis.

A nonperiodic distribution or withdrawal from a post-August 13, 1982 annuity contract is treated on a LIFO basis. In other words, to the extent that the cash surrender value exceeds the investment in the contract, taxable interest income is treated as being withdrawn first. The earnings on the investment in a commercial annuity are deferred—there is no current taxation on the earnings within the contract as long as an individual is the owner (or treated as an owner) of the contract.

The partner's tax basis in his interest in a partnership A) remains unchanged until the interest is sold or otherwise disposed. B) is increased by his share of income reported by the partnership. C) is decreased if additional capital is contributed. D) remains unchanged unless additional capital is contributed or distributions are made.

A partner's tax basis in a partnership interest is affected by items of income or loss, which are passed through to the partner on a proportionate basis. Items of income, as well as capital contributions, increase basis; deductions, as well as distributions, decrease basis.

Which of the following statements is CORRECT regarding the use of the installment sale method of accounting for income tax purposes? A) Any cost recovery recapture is recognized in the year of the installment sale. B) It may be used only if the payments are to be received over at least a two-year period. C) The installment sale method is available for use by most dealers. D) A taxpayer may not avoid the use of the installment sale method if there is an installment sale.

A requirement of the installment sale method is that any cost recovery recapture be recognized (taxed) in the year of the sale. Only the remaining gain (the gain in excess of the cost recovery recapture) is eligible for installment reporting. An installment sale is a sale in which a payment on the purchase price is received in a year later than the year of sale. It is possible for the taxpayer to affirmatively elect out of installment sale treatment by making an election when filing the tax return for the year of sale. The use of the installment sale method is not available for sales of property by dealers; dealers are required to recognize all gain from an installment sale in the year of sale, except for sales of farm property, certain timeshares, and residential lots.

John has had a very good year and has over $600,000 of taxable income, including a sizable amount of capital gains. He's thinking of selling a large block of stock to a neighbor at a price significantly below market value solely to recognize the loss. If a court disallows the loss on the sale of the stock because the sale was not bona fide and was made for the sole purpose of realizing a loss, which doctrine is being applied? A) Clear reflection of income doctrine B) Step transaction doctrine C) Sham transaction doctrine D) Assignment of income doctrine

A sale that is not bona fide and made for the sole purpose of realizing a loss from the transaction would be considered a sham transaction.

Tony and Paulette are married and will file a joint return. They have one dependent son, Ben, who was born earlier this year. When Ben was born, Paulette's father gave Tony and Paulette a cash gift of $50,000. Tony is not an active participant in a company-maintained retirement plan. Tony paid child support to a former spouse. They have provided you with the following information. Tony's salary$170,600 Tony's IRA contribution$6,000 Itemized deductions$24,000 Child support paid$12,000 Workers' compensation received$6,000 Based on the information given, what is the couple's taxable income for the 2022 tax year? A) $146,300 B) $138,700 C) $152,300 D) $134,300

A taxpayer is allowed to deduct the greater of the standard deduction ($25,900 for 2022) or total itemized deductions ($24,000). The IRA is deductible because neither spouse is an active participant in a company-maintained retirement plan. The child support payments made are not deductible. The workers' compensation benefits are excluded from income. The gift received is also excluded from income. Salary$170,600 Less: IRA deduction(6,000) AGI$164,600 Less: Standard deduction(25,900) Taxable income$138,700

Which one of the following statements is true regarding self-employment taxes? A) Net earnings from self-employment must be calculated under the accrual method of accounting. B) The wage base is not adjusted annually for cost of living increases. C) Self-employed taxpayers are subject to employer withholding. D) A taxpayer is allowed to deduct one-half of his self-employment tax liability as an adjustment to income.

A taxpayer may deduct one-half of his self-employment tax liability as an "above the line" adjustment to income. The wage base is adjusted annually for cost of living increases. Net earnings from self-employment are determined under the same accounting method as that used for income tax purposes. Self-employed taxpayers are not subject to employer withholding.

Jack was divorced on March 30 of the current year and has not remarried as of the last day of the tax year. He lives alone in his condo. His ex-wife, Mary, has custody of their son Jack Jr. What is Jack's filing status for the current tax year? A) Head of household B) Married filing jointly C) Married filing separately D) Single

A taxpayer who is unmarried, legally separated, or divorced and does not qualify for any other filing status must use the single filing status. Jack does not have custody of his son and does not qualify for head of household status.

Adrian Brown owned 500 shares of XYZ growth and income fund. She has become increasingly dissatisfied with the performance of the fund and, upon the advice of a friend, decided to execute a "telephone transfer" and switch the balance in the fund to the XYZ intermediate bond fund. Which one of the following describes the tax effect of such a strategy? A) No gain or loss will be recognized by the taxpayer, and the basis in the new fund will be the same as that of the old fund. B) Any loss will be recognized by the taxpayer, but any gain will be deferred through a reduction in the basis of the new fund. C) Gain or loss will be recognized by the taxpayer on the redemption of the old fund. D) No gain or loss will be recognized by the taxpayer, but the basis of the new fund will be reduced by any deferred gain or increased by any unrecognized loss.

A telephone transfer is the same as a sale or other taxable redemption of the fund. Therefore, gain or loss will be recognized based on the difference in the redemption proceeds and the basis in the shares redeemed. This is true even if the transfer is made between two funds in the same fund family.

Keith was divorced from Barbara in 2018. Barbara received custody of their two children. Keith was ordered to pay $1,000 per month to Barbara until the youngest child reaches age 18. At that time, the payments are to decrease to $400 per month. What portion, if any, is deductible by Keith as qualifying alimony? A) $1,000 B) $0 C) $500 D) $400

A total of $600 is tied to an occurrence or contingency related to a minor child—the child reaching age 18. Thus, this amount is presumed to be nondeductible child support.

Which of the following statements regarding common tax traps is NOT accurate? A) A transaction must be based on economic reality as well as economic form. B) Tax traps are of particular concern to owners of closely held businesses. C) A transaction is based solely on its economic form. D) Tax traps may result in the unexpected recognition of income.

A transaction cannot be based solely on form; it must also be based on economic reality.

Paul was divorced from his spouse, Patricia, late last year. As part of the property settlement agreement, Paul agreed to transfer his interest in a residential real estate tract to Patricia. Paul's cost basis in this real estate tract was $50,000. The tract was appraised at a fair market value of $100,000 at the time of its transfer to Patricia. Which of the following income tax implications is true of Paul's transfer of the real estate to Patricia? A) Paul is allowed a deduction equal to the excess of the fair market value over the basis in the property. B) Paul must recognize the gain on the real estate at the time of transfer at ordinary income rates. C) Paul's basis in the real estate is carried over to Patricia for income tax purposes. D) Patricia receives a basis in the real estate equal to the fair market value at the time of transfer.

A transfer of property incident to divorce is tax free. The transferee takes a carryover basis in the asset.

A small business with under $20 million in gross receipts and no inventory should most likely use which accounting method? A) Accrual B) LIFO C) Cash D) Hybrid

A typical small business would use the cash method of accounting. Due to cost and complexity, the accrual and hybrid methods would NOT be recommended. A business is not required to use the accrual method until it has at least $27 million in gross receipts (2022). A business that has no inventory would not use LIFO.

Abby, age 16, has earned income of $8,605 and interest income of $750 in 2022. She is claimed as a dependent on her parents' income tax return. What is Abby's taxable unearned income in 2022? A) $2,300 B) $1,150 C) $0 D) $750

Abby's taxable unearned income in 2022 is $0 ($750 unearned income − $1,150 standard deduction for unearned income). The standard deduction cannot create a negative amount.

Which of the following statements correctly defines inside buildup as it refers to life insurance? A) During the insured's lifetime, the accumulations of cash value within a policy grow on a tax-free basis. B) During the insured's lifetime, the accumulations of cash value within a policy grow on a tax-preferred basis. C) During the insured's lifetime, the accumulations of cash value within a policy grow on a tax-annuitized basis. D) During the insured's lifetime, the accumulations of cash value within a policy grow on a tax-deferred basis.

Accumulations of cash value within a life insurance policy grow on a tax-deferred basis during the insured's lifetime.

Which of the following statements best describes a tax benefit associated with an active participation rental real estate investment? A) The investment will generate portfolio rather than passive income. B) Unlimited losses may be used to offset active or portfolio income of any taxpayer. C) Up to $25,000 in losses may be used to offset active or portfolio income for certain taxpayers. D) The first $25,000 of taxable income from the investment is nontaxable.

Active participation rental real estate investment operating losses are deductible against active or portfolio income up to $25,000 per year, assuming the investor's AGI is $100,000 or less.

Which one of the following is a statute of limitations that restricts the IRS in auditing a return? A) Twelve years if 50% of gross income is unreported B) Ten years for failure to file or if a fraudulent return is filed C) Six years if 25% of gross income is unreported D) Seven years from the filing date of the return or due date, if later

After the statute of limitations has passed, except in cases of fraud, the IRS cannot audit a return. The statutes to be aware of are as follows: three years from the filing date of the return or due date if later, six years if 25% of gross income is unreported, and no statute of limitations for failure to file or if a fraudulent return is filed.

Alicia is age 16 and her total income was $6,000 in qualified dividends in 2022. Her parents' marginal tax rate is 12%. What is the tax on the dividends at Alicia's rate? (Round the answer to the nearest dollar.) A) $30 B) $0 C) $100 D) $150

Alicia can use the long-term capital gains tax rate on qualified dividends received. At her parents' marginal rate, the long-term capital gains tax rate is 0%.

Alicia is age 16 and she received $6,000 in municipal bond interest income and $900 in other interest income in 2022. Her parents' marginal tax rate is 24%. What is the total federal income tax due on her income in 2022? A) $1,472 B) $735 C) $90 D) $0

Alicia owes no federal income taxes in 2022. Municipal bond interest income is not taxable. The $900 in other interest income is less than Alicia's $1,150 standard deduction amount (for 2022).

Ted and Sophie are divorcing in 2022 and Sophie is questioning her planner about the tax impact of the financial aspects of the divorce decree. Ted will be paying alimony to Sophie for the next four years unless she remarries first. Ted will pay child support to Sophie for their only son, Mark, until he reaches age 18. Sophie and Mark will continue to live in the family home and Ted's interest in the home will be transferred to Sophie. Which statement concerning the income tax implications of Ted and Sophie is NOT correct? A) The alimony paid by Ted is deductible as an itemized deduction on his income tax return in the year he pays it. B) When the family home is transferred to Sophie by Ted, it does not affect her income. C) In the event that she later sells the home, Sophie must consider Ted's adjusted basis prior to the transfer. D) The child support payments are not taxable income to the recipient and are nondeductible by the payer.

Alimony is not (and never has been) an itemized deduction. For divorce agreements after 2018, alimony is no longer an above-the-line deduction for the payor nor taxable income to the payee.

Which of the following statements is CORRECT regarding a minority non-employee shareholder in an S corporation? Reports income when the corporation has net income for a tax year Votes for the board of directors at the annual shareholders meeting Receives a K-1 annually to prepare a personal income tax return Reports on a personal income tax return a pro rata share of corporate profit or loss A) II and IV B) I, II, and III C) I and III D) I, II, III, and IV

All of the statements are correct. The fact that the shareholder has a minority interest in the corporation has no bearing on the correct answers. The flow-through of a proportionate share of net income is reported on the K-1. The shareholder is allowed to vote for the board.

Several years ago, Allison Colbert purchased a deferred fixed annuity. The cost of the annuity was a single payment of $40,000. The annuity will provide monthly payments of $275. At the time the annuitized distributions are to begin, Allison's life expectancy will be 25 years. How much of each payment will be excluded from taxation? A) $206 B) $133 C) $142 D) $57

Allison is expected to receive $82,500 ($275 × 12 × 25). Her investment in the contract ($40,000) is then divided by the total expected return ($82,500) to determine the excludable portion of each payment. The exclusion ratio is the $40,000 divided by $82,500, which equals 48.48%. 48.48% of $275 = $133 excludable from each payment.

As part of their divorce decree, Judy, age 44, was forced to split her IRA with Alex, age 36. Alex received a check in April for $100,000 from the IRA custodian. Alex put the proceeds into a one-year CD account at the local bank. As a result, Alex will A) have to pay ordinary income tax on the $100,000 plus a 10% early withdrawal penalty. B) not have to pay income tax, but Judy will owe gift tax. C) have to pay ordinary income tax on the $100,000. D) not have to pay ordinary income tax on the $100,000.

Although the transfer itself is tax free, Alex should roll the assets distributed from Judy's IRA within 60 days into his own IRA or retirement plan. Because he failed to do so, he owes ordinary income tax on the entire distribution. When incident to a divorce, domestic relations orders (DROs) are however not subject to gift tax or early withdrawal penalties.

In 1992, John Idler purchased a single premium whole life insurance policy. In the current year his medical expenses are $15,000 and his AGI is $75,000. What is the tax implication to John if he borrows the interest from the policy's accumulated cash value to pay his current year's medical expenses? A) John will not be required to report the amount borrowed as income and will not be allowed a medical expense deduction. B) John will be required to report the amount borrowed as income, but he will not be allowed a medical expense deduction. C) John will not be required to report the amount borrowed as income, but he will be allowed a medical expense deduction. D) John will be required to report the amount borrowed as income and will be allowed a medical expense deduction.

Amounts borrowed on a single premium whole life policy issued on or after June 21, 1988 (a MEC), are taxable on a last-in, first-out basis; thus, the earnings would be taxable. A medical expense deduction will be allowed regardless of the source of the funds, since the payment would be for a valid medical expense.

Which of the following statements about S corporations is CORRECT? A) S corporations may have nonresident aliens as shareholders. B) S corporation status is automatic if there are fewer than 100 shareholders. C) S corporations are prohibited from having more than one class of stock. D) S corporations are prohibited from earning passive income.

An S corporation may have only one class of stock, although differences in voting rights are allowed within that one class of stock. An S corporation may have no more than 100 shareholders; however, the corporation must elect S status by filing a Form 2553—the treatment is not automatic. An S corporation may have passive income, although excess passive income may trigger the sting tax if the corporation had been a C corporation with earnings & profits (E&P). Nonresident aliens may not be shareholders in an S corporation.

Which of the following businesses may be considered a specified service business (SSB) for purposes of the qualified business income (QBI) deduction? A) An automobile repair shop B) A medical practice C) A furniture manufacturer/wholesaler D) An engineering firm

An SSB is any trade or business involving the performance of services in health, law, accounting, actuarial science, performing arts, consulting, athletics, financial services, brokerage services, or any trade or business where the principal asset of such trade or business is the reputation or skill of one or more of its employees; or, any trade or business that involves the performance of services that consist of investing and investment management, trading, or dealing in securities, partnership interests, or commodities. Thus, the medical practice would be an SSB, but none of the other businesses would. Engineering and architecture are specifically excluded from the definition of SSB.

Which of the following must be true in order for an adult child or elderly parent to be claimed as a dependent for another taxpayer? A dependent may not have more than $4,400 (2022) of gross income. The taxpayer must provide over 75% of the dependent's support. A person who dies during the year may be identified as a dependent. Social Security payments are always included in the dependent's gross income. A) III and IV B) I and II C) I and III D) II and IV

An adult dependent (i.e., not a "qualifying child" may not have more than $4,400 (2022) of gross income. Social Security income is excluded from the test if that is the elder's only source of income. The taxpayer must also provide over 50% of the dependent's support to claim them. Coincidentally, as long as all the tests are met, a person who dies during the year may still be identified as a dependent.

Which of the following statements regarding the 0.9% additional Medicare tax is NOT correct? A) The tax is calculated on all income in excess of the taxpayer's threshold based on filing status. B) If a taxpayer expects to have income above the threshold for the taxpayer's filing status when combined with a spouse's income and/or any self-employment income, the taxpayer may ask for additional federal income tax withheld by the employers. C) The tax is paid by the employee only; it has no employer contribution. D) The tax is in addition to the 2.9% Medicare tax that is assessed on the taxpayer's compensation and self-employment income.

An individual is liable for the additional Medicare tax if the individual taxpayer's wages, other compensation, or self-employment income (combined with a spouse if filing as married filing jointly) exceeds the thresholds for the taxpayer's filing status.

Which one of the following summarizes code sections in layman's terms, but have only short-term value? A) Private Letter Rulings B) Notices C) Technical Advice Memoranda D) Announcements

Announcements often summarize code sections in layman's terms or notify taxpayers of impending deadlines; they have only short-term value. Technical Advice Memoranda normally take place during an audit or during the appeals process of the audit; they give both the taxpayer and the revenue agent an opportunity to resolve a dispute over a technical question. Private Letter Rulings are taxpayer guidance from the IRS that apply only to the particular taxpayer(s) asking for the ruling; they are not applicable to all taxpayers. A Notice is a public pronouncement that contains official guidance about regulations or interpretations of the Code.

Which one of the following objectives of the federal taxation system would include the addition of bonus depreciation to the tax code? A) Economic objective B) Social objective C) Revenue raising D) Writing tax code

As a social objective, JGTRRA and TCJA significantly increased the Section 179 expense limit and the amount of depreciation deductions (bonus depreciation) that may be claimed in the first year in an attempt to stimulate purchases of business assets. The first, and perhaps most important, goal of the economic objective is price stability. Revenue raising through corporate, individual, and payroll taxes is an important objective of the federal taxation system.

When filing as married filing separately, which of the following are true? 1. If one spouse itemizes deductions, the other spouse must also itemize. 2. The couple avoids joint and several liability for the other spouse's return. 3. Married filing separately is usually more advantageous for a couple than married filing jointly. A) II only B) I, II, and III C) III only D) I and II

As an alternative to filing jointly, married individuals may file separate returns. This is known as married filing separately (MFS), and, in a few cases, the individuals may derive some tax benefit. However, the filing of returns MFS is usually not advantageous for married individuals.

Louisa's 12-year-old son was killed in an auto accident on May 15, 2022. Louisa is single. If her AGI in 2022 is $113,000, how much of a child tax credit can she take in her 2022 tax return for her son? A) $1,000 B) $2,000 C) $500 D) $0

As long as all other tests are met, a full child tax credit of $2,000 in 2022 (without reduction) can be taken for a person who dies during the year.

Anthony has come to you for tax help. He and his wife divorced over a year ago. She lives in Puerto Rico. He lives in Florida. Their four-year-old daughter, Julia, lives with her mom all year. Anthony explained that his wife will file a tax return in Puerto Rico but is not required to file a Form 1040 with the IRS. He therefore wants to claim their daughter as a dependent on his return so he can take a head-of-household deduction and also receive the child tax credit. He pays Sofia substantial alimony and child support. He insists that he can claim Julia as a dependent this reason. Which of these is correct about Anthony's tax situation? A) He can claim his daughter as a dependent because no one else is claiming her for federal tax purposes. B) He cannot claim his daughter because his wife is a non-citizen. C) He cannot claim his daughter as a dependent because she does not live with him. D) He can claim his daughter as a dependent because he provides over half of her support.

Anthony is mistaken. In order to be claimed as a dependent, the child must live with the taxpayer for more than half the year. Although Julia meets many of the requirements of a qualifying child, she does not meet the residency test. Julia did not live with Anthony at all during the year and thus fails the residency test. An exception ​may ​apply to divorced couples. The custodial parent could conceivably sign a waiver allowing the non-custodial parent to claim the child, but there is no indication such a document has been signed. If a taxpayer is required to file a state return, they must also file a federal return. Residents of Puerto Rico are US citizens and are ordinarily required to file Form 1040 in addition to a Puerto Rican (i.e., state) tax return.

A taxpayer purchases a new computer for use in his consulting business. He incurs sales tax and shipping charges in connection with the purchase. Which of the following correctly describes the treatment of the sales tax and shipping charges? A) The sales tax is capitalized, and the shipping charges are currently deductible. B) Both are capitalized. C) Both are currently deductible. D) The sales tax is currently deductible, and the shipping charges are capitalized.

Any cost associated with the acquisition of an asset must be capitalized (added to basis). This would include both the shipping charges and the sales tax. Any cost associated with the acquisition of an asset may not be currently deducted.

What, if any, is the primary difference in tax treatment between a general partnership and a limited partnership? A) The limited partners are treated only as capital investors, whereas the active partners receive both ordinary and capital distributions. B) Limited partnerships are taxed as corporations, while general partnerships are taxed as partnerships. C) The limited partners only receive capital distributions, while the general partners receive only ordinary income distributions. D) None of these.

As long as the limited partnership is classified as a partnership (and not a C corporation) for tax purposes, the taxation is no different than it would be if the organization were a general partnership. That is, the partnership issues a K-1 to all of the partners for their distributive share of items of income and loss. Limited partners generally must treat net income or loss from the partnership as passive.

Colin and Lucy are meeting with their financial planner, Mark, before the tax year ends to discuss changes in their situation. In March of this year, Lucy's mother, Joyce, suffered a stroke and required placement in a nursing home. She did not have the funds to pay for it and it fell to the couple to support her stay until her death in November of this year. They would like to know what tax relief they may have for supporting Joyce this year. What should Mark tell the couple? A) The couple can take a partial tax credit for Joyce this year. B) The couple is assured of at least being able to list Joyce on their tax return this year. C) Mark should ask for documentation to establish Joyce's income, the amounts spent on her own support, and how much the couple spent before he provides any recommendations. D) Because they paid nursing home expenses for Joyce, the couple is entitled to the dependent care credit on their tax return.

B

Which one of the following types of investors derives the greatest tax benefit from investing in preferred stocks? A) Mutual funds investors B) Government investors C) Corporate investors D) Nonprofit institutional investors

Because 50% of the preferred dividends received by a corporation are exempt from federal income taxes, a corporation gains a tax advantage. The government and nonprofit organizations pay no income taxes. Mutual funds are also exempt from taxation.

Several years ago, John established a 2503(c) trust for his niece, Cindy. John named his attorney as trustee and gave him broad investment authority. Cindy turned age 21 this year and acquired the right to revoke the trust, but she chose to allow the trust to continue until she is age 25. Which of the following correctly identifies the taxpayer, if any, who must pay tax on the trust income? A) John, because this is required by law B) Cindy, because she allowed the trust to continue past her 21st birthday C) The attorney as trustee, because of the broad investment authority D) No one, because there is no tax on the trust income until it is distributed

Because Cindy waited past her 21st birthday, when she had the right to revoke the trust, she is responsible for taxes on the trust given to her.

Jasper invested $20,000 for a 30% interest in a nonpublicly traded limited partnership. Jasper is not a material participant. The partnership has a loss this year and Jasper's share is $15,000. This is the only nonpublicly traded limited partnership Jasper owns an interest in, but Jasper did have portfolio income of $9,000 in the same year in addition to his salary of $150,000. How much of the loss can Jasper deduct this year? A) $9,000 B) $15,000 C) $6,000 D) $0

Because Jasper has no passive income to offset the passive loss, the loss is suspended under the passive activity loss rules.

Alex established a 2503(c) trust for his daughter, Julie, when she entered college four years ago. Alex decided to name his attorney as trustee and give Julie the right to revoke the trust at age 23, when she finished college. Julie did not revoke the trust and chose to allow the trust to continue until she is age 30. Which of the following correctly identifies the taxpayer, if any, who must pay tax on the trust income? A) The attorney as trustee B) Alex, because this is required by law C) Julie, because she allowed the trust to continue past age 23 D) The trust, because it is irrevocable and a separate taxable entity

Because Julie waited past age 23 when she had the right to revoke the trust, she is responsible for taxes on the trust given to her.

Macy, age 22 and disabled, has been living with her older brother, Leon, since last December when their parents died in an auto accident. Leon has been providing all of her support as her share of the life insurance benefit was put into a trust for her college education. Leon is asking his planner about Macy's status as a qualifying relative because her life insurance benefit was $250,000. He hopes to use the head of household status this year. What does the planner tell him? A) Because Leon provided all of Macy's support this year, he may claim her as a dependent. B) Even though Leon supported Macy, Leon cannot list her as a dependent because her insurance benefit was so large. C) Only the trust can list Macy as a dependent on its return. D) Leon cannot list Macy as a dependent and claim head of household status.

Because Leon provided all of Macy's support this year, he can claim her as a dependent and claim head of household status.

Ron, age 43, and Sandy, age 41, are married with two children: Michael, age 12, and Victoria, age 8, who has been blind since her birth. Ron is an architect and general partner with XYZ partnership. Sandy is self-employed as an attorney and works out of a home office. Her home office is exclusively and regularly used for business, and the home office is her principal place of business. Their information for the current tax year is as follows: Adjusted gross income: $217,300 Itemized deductions (including qualified residential mortgage interest, taxes paid, and charitable contributions): $33,000 Early in the current year, Sandy's father died. Sandy is the sole beneficiary of her father's entire estate. The estate is presently in the probate process. Sandy's mother, Lisa, age 68, has moved in with them but provides her own support. She was married to Sandy's father when he died earlier this year. This is Ron's second marriage. He makes monthly support payments to his former spouse and his daughter. Because both Ron and Sandy are considered to be self-employed, they make quarterly estimated tax payments each year to cover both their income tax and self-employment tax obligations. Based on the information provided in the case scenario, which of the following statements regarding Lisa's income tax filing status is CORRECT? A) Lisa must file married filing separately. B) Lisa may file married filing jointly. C) Lisa must file a single return. D) Lisa may file as head of household

Because Lisa's spouse died earlier in the year, she may use married filing jointly status. She cannot use the head of household filing status because she does not maintain a household for a qualifying child or relative.

Which of the following married couples may NOT file their federal income tax return using the married filing jointly (MFJ) status? A) Sara has income for the tax year but Jack does not. B) Mark and Beth are both self-employed and have different fiscal years to accommodate their businesses. C) Terry and Edie moved into separate homes in November of this year but have not yet started legal proceedings to end their marriage. D) Paul and Josie were married this year on New Year's Eve.

Because Mark and Beth have different fiscal years to accommodate their businesses, their tax years do not begin on the same date and they may not file as MFJ. The other three couples are eligible to use the MFJ filing status.

Which of the following married couples may file their federal income tax return using the married filing jointly (MFJ) status? A) Mark and Beth are both self-employed and have different fiscal years to accommodate their businesses. B) Paul and Josie will be married next year on New Year's Day. C) Terry and Edie divorced last year but did not move into separate homes until November of this year. D) Sara has income for the tax year but Jack does not.

Because Mark and Beth have different fiscal years to accommodate their businesses, their tax years do not begin on the same date, and they may not file as MFJ. In order to use the MFJ status, a couple must be legally married on the final day of the tax year.

Ron, age 43, and Sandy, age 41, are married with two children, Michael, age 12, and Victoria, age 8, who has been blind since her birth. Ron is an architect and general partner with XYZ partnership. Sandy is self-employed as an attorney and works out of a home office. Her home office is exclusively and regularly used for business, and the home office is her principal place of business. Their information for the tax year 2022 is as follows: AGI: $217,300 Itemized deductions (including qualified residential mortgage interest, taxes paid, and charitable contributions): $33,000 Early in the current year, Sandy's father died. Sandy is the sole beneficiary of her father's entire estate. The estate is presently in the probate process. Sandy's mother, Lisa, age 68, has moved in with them but provides her own support. She was married to Sandy's father when he died earlier this year. This is Ron's second marriage. He makes monthly support payments to his former spouse and his daughter. Because both Ron and Sandy are considered to be self-employed, they make quarterly estimated tax payments each year to cover both their income tax and self-employment tax obligations. Based on the information provided in the case scenario for Ron and Sandy, which of the following statements regarding Sandy's home office deduction is CORRECT? A) The amount Sandy may deduct this year for qualifying home office expenses is unlimited. B) Sandy may deduct her qualifying home office expenses when calculating AGI. C) The net income from the business use of Sandy's home must equal or exceed the business expenses (including depreciation). D) Sandy is not eligible to deduct her home office expenses.

Because Sandy is self-employed, she can deduct her qualifying home office expenses when calculating AGI (as a line item on Schedule C). The amount of her home office expense deduction this year is limited to the net income from her business. A profit from the business is not one of the tests for qualification of a home office for tax deduction purposes. Rather, it is one of the limitations on the deductible amount after it qualifies.

Yetunde has been selling sports memorabilia online for six years. It is not her primary employment, but she has been doing well with sales. While Yetunde will be itemizing her deductions this year, she believes she can report her income and expenses on Schedule C. Her gross sales from the sports memorabilia this year are $55,000. Her total expenses are $37,600, including cost of goods sold of $28,000. Yetunde has kept detailed records since she began selling memorabilia and can track her profit and loss for each year. In Year 1, she had a loss of $5,000; in Year 2, there was a loss of $2,000; in Year 3, she had a profit of $9,000; in Year 4, Yetunde had another loss of only $500; and Year 5 was a good year with a profit of $12,000. How should Yetunde report her sports memorabilia business for Year 6? A) Only cost of goods sold can be expensed on Schedule C; the rest of the business expenses are miscellaneous itemized deductions. B) Because this is not Yetunde's primary employment, the business is deemed a hobby and she should use hobby rules to report the income and expenses. C) Yetunde can use Schedule C to report the expenses of her business but must report the $55,000 as other income on the front of the Form 1040. D) Because Yetunde has a profit from the business in three of the last five years, the business is presumed not to be a hobby and she will report income and expenses using Schedule C.

Because Yetunde had a profit in Years 3, 5, and 6, the hobby rules do not apply and Yetunde can use Schedule C to report income and expenses from her business.

Lacey is a single taxpayer. Lacey's aunt, Blanche, has come to live with her after it was determined she could no longer live independently. She is physically challenged and needs full-time care. Blanche's parents set up a trust for her that supplies all of her support, including paying for in-home care and all medical bills. Lacey wants to know how this will affect her income tax situation. What does the planner tell her? A) Lacey may list Blanche as a dependent on her income tax return. B) Blanche is a qualifying relative. C) Lacey's filing status changes to head of household. D) Lacey and Blanche will both file as single.

Because the trust pays for all of Blanche's support, Lacey's income tax return is unaffected. Both Lacey and Blanche will file as single.

Lindsey is age 2 and her total income was $4,000 in qualified dividends in 2022. Her parents are in the 12% marginal tax bracket. What is her total tax on the dividends? A) $0 B) $600 C) $585 D) $216

Because these are qualified dividends, they qualify for long-term capital gains rates. Lindsey's taxable unearned income is calculated as follows: $4,000 − $1,150 (2022) standard deduction − $1,150 taxed at the child's rate = $1,700 of taxed at her parents' marginal rates. Therefore, her tax rate on the qualified dividends received is 0%.

Barbara and Morgan were divorced last year and have a child age 13. The court ruled that Barbara must pay Morgan $2,000 per month to cover both alimony and child support. The divorce decree states that 70% of each payment is allocated for child support and the payments must last for five years. Based on this information, which of the following statements is CORRECT? A) Barbara can deduct $1,400 per month as child support. B) Barbara can deduct $600 per month as child support. C) Barbara cannot take a deduction. D) Barbara can deduct $1,400 per month as alimony.

Beginning in 2019, alimony payments are no longer deductible by the payor.

What is Bobby's total self-employment tax on his Schedule C net income of $50,000? Round your answer to the nearest dollar. A) $7,650 B) $3,533 C) $7,065 D) $3,825

Bobby's total self-employment tax is $7,065 ($50,000 × 0.9235 = $46,175; $46,175 × 0.1530 = $7,065).

Which of the following parties may be subject to IRS penalties? A) Tax preparer only B) The individual taxpayer when the tax preparer has closed their business C) Both the individual taxpayer and the tax preparer D) Individual taxpayer only

Both clients and tax professionals alike may be held accountable for tax reporting errors. Financial planners should also be aware that a tax return preparer might be subject to certain penalties. These penalties include failure to provide a taxpayer with a copy of their return, failure to keep a copy of all returns prepared for at least the last three years or to maintain a list of returns prepared, and failure to sign a return as preparer and give their tax identification number on the return.

Darryl owns two houses he has rented to tenants. He has hired Property Managers, Inc., to collect rents, provide maintenance, and pay mortgages on the properties. His approval is required for all disbursements and any repairs or changes made to the houses. Which of the following statements regarding Darryl's rental activities are CORRECT? 1.) Darryl is a small investor who meets the definition of active participation in the rental activity. 2.). Subject to AGI limitations, Darryl may deduct up to $25,000 in real estate losses against active and/or portfolio income annually. A) Neither I nor II B) II only C) I only D) Both I and II

Both statements are correct. Darryl meets both of the tests for an active participant in real estate activities and qualifies for the $25,000 deduction. The tests are as follows: The small investor must actively participate in the activity; note that active participation is a lesser standard than that of material participation and requires only that the investor participate in management decisions regarding the real property. The investor must own at least 10% in value of all interests in the activity during the taxable year.

Which of the following statements regarding the kiddie tax is CORRECT? 1. The kiddie tax provision limits income shifting by preventing families from transferring large amounts of unearned income to children and making the shift effective for income tax purposes. 2. If a child under the age of 19 has unearned income above a specified amount, the excess is taxed at the parents' marginal tax rates for the year, rather than at the child's marginal rate. A) Both I and II B) Neither I nor II C) I only D) II only

Both statements are correct. The kiddie tax applies to unearned income for children under the age of 19 and full-time students until they reach age 24.

Your clients, Joseph and Jane, have read many articles in financial publications about the alternative minimum tax (AMT) and are concerned that some of their investments and activities may cause AMT problems. Which of the following are preference items or adjustments for purposes of the individual AMT? 1. Interest from qualified private-activity municipal bonds issued in 2008 2. Bargain element on the exercise of an incentive stock option 3. Excess of percentage depletion over the property's adjusted basis 4. Cost depletion deductions A) I and IV B) II, III, and IV C) I, II, III, and IV D) I, II, and III

By definition, the only listed item that is not an AMT preference item or adjustment is the cost depletion deduction. Note that interest on private-activity municipal bonds issued in 2009 and 2010 is not a preference item for the AMT.

Which of the following statements concerning alimony is CORRECT? A) No payments except cash can be considered alimony. B) Cash payment of the payee spouse's mortgage made by the payor spouse as required by the divorce or separation instrument qualify as one-half alimony. C) Payments to maintain property used by the payee spouse but owned by the payor spouse do not qualify as alimony. D) Payments made with respect to jointly owned property are considered full alimony.

Cash payment of the payee spouse's mortgage, rent, tuition, or tax liability made by the payor spouse as required by the divorce or separation instrument may qualify as alimony. Payments to maintain property used by the payee spouse, but owned by the payor spouse, do not qualify as alimony, even if required under the instrument. Payments made with respect to jointly owned property are considered one-half alimony. These property-related expenditures may include mortgage payments, real estate taxes, and homeowners insurance.

Phillip's personal automobile was almost destroyed in an accident. The insurance company paid $6,000 on the claim. The auto's fair market value before the accident was $16,000, and the value after the accident was $1,000. His basis in the automobile was $12,000. Phillip's AGI is $42,500. What is the amount of Phillip's deductible casualty loss? A) $1,750 B) $6,000 C) $0 D) $1,650

Casualty losses are only deductible for damages sustained within a federally declared disaster area. Thus, there is no deduction for this loss. If the loss had been incurred in a federally declared disaster area (as a result of the disaster), the deductible casualty loss computation would begin with the lesser of the decrease in fair market value ($15,000 decrease in FMV) or the adjusted basis in the property. In this situation, the adjusted basis of $12,000 must be reduced by a $100 floor, the insurance of $6,000, and further reduced by 10% of the adjusted gross income. Thus, $12,000 reduced by $100, $6,000 insurance, and further reduced by $4,250, equals $1,650.

Justin was divorced in February 2022. As part of the divorce decree, Justin is paying $3,000 per month in child support and alimony to his ex-spouse, Casey. When his youngest child becomes 18, this payment will be reduced to $1,000. Justin would like to know the tax implications of the child support and alimony he is paying. After evaluating Justin's circumstances, what should you tell Justin regarding the tax treatment of these payments? 1. None of the payments are taxable income to his ex-spouse. 2. Justin may deduct the alimony but not the child support. A) II only B) Neither I nor II C) I only D) Both I and II

Child support is not deductible by the payor or taxable to the payee, and, beginning in 2019, the tax treatment is the same for alimony.

Christopher's wife, Sarah, died last year and he has been living alone in their home since then. What filing status should he use when filing his income tax return for this year? A) Head of household B) Qualifying widower (surviving spouse) C) Single D) Married filing jointly

Christopher may only file as a single taxpayer. He has no dependent children or other dependents in his household and does not qualify for either qualifying widower or head of household status.

Lois, age 29, and Clark, age 31, recently divorced. Lois is taking custody of their three children ages four, two, and one. Clark will pay alimony and child support of $1,000 per month. They live in a state that requires alimony payors to obtain a life insurance policy. Clark obtained a group term life policy for $50,000 through his employer to meet this obligation. Clark's lawyer claims this satisfies his legal obligation. Which of the following should a financial planner recommend to Lois? A) She should inquire if she can take loans against the cash value for future college tuition payments. B) She should accept the settlement as adequate. C) She should demand his employer increase the coverage. D) She should demand an increase in coverage and request a permanent policy.

Clark is underinsured given his legal obligations. Lawyers are not financial planners and generally are not trained to calculate life insurance coverage needs. Consequently, they often settle for an amount of life insurance that is too low. Term policies do not have cash value. If Clark leaves his employer, the term policy will most likely be canceled, which would compel Lois to take Clark back to court.

Which of these is NOT a step in the tax research process? A) Obtaining a letter ruling from the IRS if the issue is novel B) Diagnosing the problem from the facts C) Obtaining all the facts D) Communicating the answer

Communicating with the IRS is not a typical step in the research process.

Mary has owned her principal residence for over six years. Two years ago, she married John, who immediately moved into the residence. John has never used the Section 121 exclusion. If Mary sells the residence this year and John and Mary file a joint return, which of the following statements is CORRECT with respect to the availability of the Section 121 exclusion? A) The maximum exclusion is $250,000 because John is not an owner of the residence. B) The maximum exclusion is $500,000 because Mary has at least two years of ownership, and both spouses meet the use requirement. C) The maximum exclusion is $250,000 because that is the maximum exclusion for an individual who was single when the residence was purchased. D) The maximum exclusion is $500,000 because that is the amount always available for married taxpayers who file jointly.

Currently, Section 121 allows for a gain exclusion, of up to $500,000 for taxpayers married filing jointly, to any taxpayer who satisfies certain tests, known as the ownership test and the use test. To satisfy the ownership test, the home must have been owned and used as a principal residence for at least two of the five years preceding the date of sale. (Note: These years do not have to be consecutive; they only have to add up to at least two years.) Either spouse can meet the ownership test, but both must meet the use (two-out-of-five-year) test. This is likely not difficult for most married couples (applies even to those living in the house and then getting married), but it can be burdensome for individuals who are divorced or in the process of a divorce.

During the current year, Peter has Section 1231 gains totaling $8,000. He also has $1,000 of Section 1231 losses. Four years ago, Peter reported a net Section 1231 loss of $2,000. These are the only two years in which Peter has had Section 1231 gains or losses. What is the amount and character of the current year's Section 1231 gains and losses? A) $5,000 ordinary income, $2,000 long-term capital gain B) $7,000 long-term capital gain C) $7,000 ordinary income D) $2,000 ordinary income, $5,000 long-term capital gain

D

Which of the following statements describes the constructive receipt doctrine? 1) Darrell was issued notification in December of this year that his bonus for the current year would be $10,000 and the check would be issued in January next year. 2) Martha was notified that a check would be issued to her on December 31 for royalties on her song used in a local commercial; the check was available for pick up that day. A) I only B) Both I and II C) Neither I nor II D) II only

Darrell was only notified in December of the amount of the bonus that would be paid. No check was issued and the funds were not available until January of the following year. The check to Martha constitutes constructive receipt, whether or not she picks up the check in December.

Derrick is about to begin receiving payments from a deferred fixed annuity that he purchased many years ago. His investment in the annuity contract was $202,500. He is to receive $1,500 per month for the rest of his life. His current life expectancy, based on IRS tables, is 15 years. What amount, if any, of each monthly payment is taxable to Derrick? A) $375 B) $0 C) $1,500 D) $1,125

Derrick is expected to receive $270,000 ($1,500 × 12 × 15). His investment in the contract ($202,500) is then divided by the total amount expected to be received ($270,000) to determine the excludible portion of each payment. $202,500 ÷ $270,000 = 75% exclusion ratio × $1,500 = $1,125 excludible from each payment. Thus, the remaining $375 of each payment is taxable. $202,500 investment/$270,000 expected Return=75% 75%×$1,500 payment=$1,125 excluded

Disclosing or using client information given to a tax preparer may result in which one of the following maximum penalties? A) Closure of business B) 50% tax penalty C) Imprisonment for up to one year D) 100% penalty

Disclosing or otherwise using any client information given to a tax return preparer pursuant to the preparation of a tax return is a crime for which the preparer may be imprisoned for up to one year.

Barney had an adjusted basis of $15,000 in an antique automobile that he had purchased as an investment. He sold the auto for $45,000. This year, the buyer made a down payment of $12,000 and made the first of three annual installment payments of $11,000. What amount, if any, of installment sale income must Barney recognize in the current year? A) $7,334 B) $15,333 C) $23,000 D) $7,667

Dividing the profit by the total contract price yields the gross profit percentage, or ratio. In this case, the gross profit was $30,000 and the contract price was $45,000. The profit of $30,000 divided by $45,000 equals a gross profit percentage of 66.67%. This tells us that 66.67% of each dollar received is profit. This percentage is multiplied by the $23,000 received during the year to yield a taxable amount of $15,333.

Dwight has an active participation rental real estate activity. In 2021, he had losses of $20,000 from the active participation real estate and his AGI was $140,000. He was allowed to deduct $5,000 of the losses against other income. The remaining $15,000 loss was carried forward into 2022. In 2022, Dwight has an AGI of $90,000 and only $6,000 of current losses from his real estate rental activity. What amount of loss, if any, may Dwight deduct in 2022? A) $6,000 B) $21,000 C) $0 D) $15,000

Dwight is allowed to deduct $21,000 of losses in 2022. The $15,000 carryforward losses are treated as if they occurred in 2022. The $6,000 current losses plus the $15,000 carryover losses total $21,000. This loss would be fully deductible as the AGI is under $100,000.

Pat's son, Alex, has been severely disabled since he was a small child. He has just turned 22 and Pat has provided all of his support until this January when he was placed in an extended care facility. With the increased expense not covered by insurance, Pat must split his support with his paternal grandmother and his aunt who have provided the other two-thirds equally. Because this is a new situation this tax year, Pat has come to her financial planner to understand who can list Alex on their tax return for the purposes of filing status and possible credits. What does their planner tell the Pat? A) No one may list Alex on their return because no one has paid more than half of Alex's support. B) The three relatives are free to agree among themselves who will list Alex on their return. C) Only Pat, as his mother, can list Alex on her return. D) While one of the taxpayers may list Alex as a dependent in an agreed upon year, no tax credits would be available to that taxpayer.

Eligible taxpayers are generally free to agree among themselves (via a multiple support agreement) who will list an individual as a dependent and who also will claim any available credits when an individual can be claimed by more than one taxpayer.

What is the amount and character of the penalty for an employer who fails to withhold Social Security and federal income taxes from employee paychecks? A) 100%, and the responsible party needs to pay back what is owed plus a 100% penalty B) 100%, but the responsible party needs only to pay back what is owed with no additional penalty C) 50%, and the responsible party needs to pay back what is owed plus a 50% penalty D) 50%, but the responsible party needs only to pay back 50% of what is owed with no additional penalty

Employers are required to withhold amounts from an employee's paycheck for Social Security taxes and federal income taxes. If the employer fails to do so or fails to pay such amounts to the IRS, they or any other responsible person will be subject to the 100% penalty, which is simply having to pay 100% of the amount they should have collected, accounted for, and paid; they are not subject to any additional penalty.

For purposes of the accumulated earnings tax (AET), what amount may a regular corporation retain in accumulated earnings before having to prove a reasonable business need for the retained funds? A) $25,000 B) None of these C) 15% of the current year's earnings D) $250,000

Every regular corporation can accumulate up to $250,000 in retained earnings without having to prove a reasonable business need.

Which one of the following statements is incorrect regarding investment interest expense? A) Investment interest expense is deductible up to the amount of the net investment income. B) Excess investment interest expense cannot be carried forward into succeeding tax years. C) Interest paid or accrued to purchase or carry tax-exempt investments is not deductible. D) Net investment income is the taxpayer's investment income—typically interest, nonqualified dividends, and short-term capital gains.

Excess investment interest expense can be carried forward into succeeding tax years. Investment interest expense is deductible up to the amount of net investment income. The interest on funds borrowed to purchase tax-exempt investments is not deductible. The net investment income is typically interest, nonqualified dividends, and short-term capital gains. Long-term capital gains and qualified dividends may be included at the taxpayer's election, but the taxpayer must forgo the preferential tax rates on these items.

Amy bought a small print shop for $40,000. In the current year, as a result of depreciation deductions taken on previous income tax returns, her adjusted basis for the print shop was $38,000. Also in the current year, she added a new wing to the print shop, which cost $40,000. What is her adjusted basis for the building in the current year after the addition to the building? A) $78,000 B) $80,000 C) $78,000, less the actual cost of construction of the new addition D) $80,000, less the actual cost of construction of the new addition

Explanation The Tax Code requires that certain items must be included when calculating adjusted basis. Two of the items included are expenditures affecting the capital account (i.e., the cost of an addition) and any depreciation deductions taken. When Amy first bought the building, her basis was $40,000. Her adjusted basis without the new addition, but including the $2,000 depreciation, would be $38,000. The cost of the new addition is not deductible from income, but is instead a capital expenditure that must also be depreciated. The $40,000 basis of the addition is added to the original basis of the building, resulting in an adjusted basis of $78,000.

Raphael has Schedule C net income of $55,000 for the current year. In addition, he has a flow-through of interest and dividends of $27,000 from an investment partnership. What is Raphael's self-employment tax for 2022? Round your answer to the nearest dollar. A) $6,755 B) $8,415 C) $7,771 D) $11,586

Explanation The flow-through of interest and dividends is not subject to the self-employment tax. (Answers may vary slightly due to rounding.) Actual earnings$55,000.00Less 7.65%(4,207.50)Net earnings from self-employment$50,792.50× 15.3%Self-employment tax$7,771.25

All of the following are examples of self-employment income for purposes of the self-employment tax except A) net Schedule C income. B) income from a sole proprietorship. C) net Schedule F income. D) salary paid to an S corporation shareholder.

Explanation Wages and salary paid to an S corporation shareholder are not self-employment income.

In the current tax year, Fay has short-term capital loss carryovers of $5,000 and long-term capital loss carryovers of $40,000, both carried over from the previous year. Her net short-term gain for this year is $6,000, and her net long-term gain for this year is $5,000. How much of her gain for this year will be taxable? A) $1,000 B) $5,000 C) $0 D) $6,000

Fay can apply her short-term capital loss carryover to all current short-term capital gains, which results in a net short-term capital gain for this year of $1,000 ($6,000 gain − $5,000 carryover). She is then left with a net long-term capital loss of $35,000 ($5,000 gain − $40,000 carryover). To calculate net capital gains for the year, aggregate the long-term and short-term gains or losses, which in this case equals $35,000 long-term loss − $1,000 short-term gain, or a $34,000 net capital loss. She has no net gain and, as such, pays no taxes on any of the capital transactions she made this year.

Irwin, a young financial planner, operates his practice as an LLC. His LLC net income is $100,000, and the taxable income on his single return is $140,000. The LLC paid no wages, and has no depreciable property. What is the amount of Irwin's qualified business income (QBI) deduction? A) $28,000 B) $0, because he has no depreciable property C) $0, because he paid no wages D) $20,000

For a specified service business (SSB), if the taxpayer's taxable income is below the phaseout range of $170,050 to $220,050 (Single 2022), the QBI deduction is fully available. An SSB involves performance of services in the fields of health, law, accounting, actuarial science, performing arts, veterinary services, consulting, athletics, financial services, or brokerage services. For pass-through entities, the owner/taxpayer may be allowed a deduction equal to 20% of the lesser of the QBI, or 20% of the taxpayer's taxable income (reduced by net capital gain). Thus, the available deduction is 20% of the QBI of $100,000, or $20,000.

Fred runs a small business that has been in the family for over 50 years. The business has always used the cash basis method of accounting. Which of the following would be considered income to Fred last year? 1) A bonus he declared for himself on December 31 of last year that is not payable until January 2 of the following year 2) Dividends received on December 30 of last year 3) Salary received during last year 4) Interest on his money market account last year, but not posted until January 2 of the next year A) I, II, and III B) II, III, and IV C) II and III D) I and II

For the cash method of accounting, all income actually received during the tax year is included in gross income (e.g., dividends, interest, wages). Income received constructively during the year, even though actual receipt is delayed, is includible for the tax year. Bonuses declared but not yet payable until the next tax year are not includible for the prior tax year because there was no constructive receipt.

Claudia has $130,000 in gross income in 2022. Investment income comprised $19,000 of her income and the rest was salary. She has made a $4,000 contribution to her company Section 401(k) plan and has a deduction for mortgage loan interest. She will itemize deductions. Claudia's filing status is single. Which IRS tax form can Claudia use to report her income in 2022? A) IRS Form 1120 B) IRS Form 1040 C) IRS Form 1041 D) IRS Form 1065

Form 1065 is for partnerships. 1120 is for C Corporations. Form 1041 is used by estates and trusts. Claudia may only use Form 1040.

Fred, age 59, is a single taxpayer. He has wage income of $90,000 for the current tax year. Fred is not an active participant in a company-maintained retirement plan. In addition, he has the following: Long-term capital gains$4,000 Short-term capital losses$9,000 Loss from active participation rental real estate$3,700 Alimony paid to ex-wife$5,200 Gambling winnings$7,100 Gambling losses$4,100 Interest income$3,500 Sole proprietorship (Schedule C) income$2,000 Self-employment tax liability$283 Qualified home mortgage interest$6,890 Real estate tax paid$1,840 Investment interest expense$4,925 Charitable contributions (cash)$2,975 Total medical expenses$4,217 State and local income taxes$1,625 Consumer interest$2,180 Unreimbursed employee business expenses$1,560 IRA contribution$6,000 Fred's divorce was finalized in 2014. What is the amount of Fred's AGI? A) $86,200 B) $86,341 C) $84,559 D) $83,559

From the $95,900 of total income, we would subtract the adjustments to income. The adjustments to income total $11,341. This is comprised of the $5,200 of alimony payments, one-half of the self-employment tax liability ($141), and the IRA contribution of $6,000. Note that the IRA contribution is deductible in this situation because Fred is not an active participant in a company-maintained retirement plan. Subtracting the $11,341 from the total income of $95,900 leaves us with $84,559. The total income is computed by starting with the wage income of $90,000. To that $90,000, we add the interest of $3,500, the self-employment income of $2,000, and the gambling winnings of $7,100. Of the $9,000 of short-term capital losses, the first $4,000 would offset the long-term capital gains. Of the remaining $5,000 net capital loss, only $3,000 would be deductible in computing the current year total income. The $3,700 active participation real estate loss is fully deductible, as the AGI (computed without regard to the active participation loss) is less than $100,000. Thus, the total income is $95,900. Total (gross) income$95,900 Adjustments to income Alimony paid$5,200 ½ self-employment tax$141 IRA deduction$6,000 Total adjustments$11,341 Adjusted gross income$84,559

Greg, Justin, and Mel are all engineer-owners of GJM, Inc., a C corporation that is a qualified personal service corporation (PSC). Historically, their gross receipts have been as follows: $4.75 million annually in Years 1-4; and $5.75 million in Year 5. This year, gross receipts are expected to be $6.1 million. Can GJM use the cash method of accounting this year if it clearly reflects income? A) Yes, the cash method is available to GJM because it is a PSC. B) No, GJM is a C corporation and must use the accrual method. C) Because GJM provides services to the construction industry, it must use the long-term contract method. D) No, GJM must use the accrual method because the average earnings for the preceding three years are more than $5 million.

GJM, Inc., is a PSC and may use the cash method of accounting.

An office building with an adjusted tax basis of $120,000 was destroyed by fire on January 2 of last year. On January 15 of the current year, the insurance company paid the owner $200,000. The owner reinvested $190,000 in a new office building. What is the basis of the new building under Section 1033 (the involuntary conversion rules)? A) $120,000 B) $190,000 C) $110,000 D) $180,000

Gain realized equals $80,000, and gain recognized equals $10,000 ($200,000 amount realized − $190,000 amount reinvested). This is a net postponed gain of $70,000 ($80,000 − $10,000). The basis of the new building equals $120,000 ($190,000 − $70,000).

How can passive activity losses be deductible from other taxable income? A) Passive activity losses are deductible against portfolio gains. B) Passive losses can only be offset by passive gains. C) Passive losses can only be carried forward against future passive gains. D) Passive losses can offset passive gains, and a phased-out $25,000 deduction ($12,500 for MFS) for rental real estate applies.

Generally, passive activity losses can only be used to offset passive activity income. An exception for the deduction of passive losses is for rental real estate. A phased-out deduction of $25,000 ($12,500 for MFS) of rental real estate losses is allowed against a taxpayer's other nonpassive income. If AGI is greater than $100,000 ($50,000 for MFS), however, there is a reduction of 50 cents for each dollar over $100,000 ($50,000 for MFS), which then terminates at $150,000 ($75,000 for MFS) of AGI.

How can passive activity losses from an ongoing nonpublicly traded partnership, such as a RELP, be deductible from other taxable income? A) Passive losses from one partnership can only be offset by passive gains from the same partnership. B) Passive losses can offset passive gains. C) Passive activity losses are deductible against portfolio gains. D) Passive losses can only be carried forward against future passive gains.

Generally, passive activity losses can only be used to offset passive activity income. Losses from nonpublicly traded partnerships (non-PTP) can be used to offset gains from other nonpublicly traded partnerships. Losses from master limited partnerships can only offset gains from the same master limited partnership. Losses from a non-PTP may be carried forward, but it is not a requirement. Passive losses cannot be used to offset portfolio income.

Dan and his spouse, Gina, are getting divorced. Dan has an IRA with a balance of $400,000. Part of the balance was rolled over from his 401(k). He claims the IRA is a gift because it was funded predominantly with cash gifts from his parents during their 15-year marriage. Gina does not have a retirement account. Which of the following is correct? A) Gina can only claim a portion of the IRA if they live in a community property state. B) Qualified plans are exempt from property settlements in a divorce. C) Because the IRA was funded with gift money, it is treated as separate property. Gina has no claim to the funds. D) Gina can request a domestic relations order and, if granted, claim a portion of the IRA funds.

Gina can request a domestic relations order and submit a claim for a portion of the IRA funds. By law, IRAs and 401(k)s can only be funded with earned income. For this reason, Dan's claim that the IRAs were funded with gift money is an implausible defense. Qualified plans and IRAs may be divided in a property settlement, whether the parties reside in a community property or common law state.

Don and Paul are married. They adopted an infant daughter in December of last year. They have consulted you, a CFP® professional, for advice on how to proceed when filing their federal income tax return this year. What should you recommend as their filing status this year for their federal return? A) Married filing separately B) Head of household C) Single D) Married filing jointly

Having a dependent does not change the filing status for a married couple.

James has suspended losses from a nonpublicly traded partnership of $17,000. In 2022, he has income from a nonpublicly traded partnership of $15,000. What amount of suspended losses, if any, may James deduct in 2022? A) $0 B) $2,000 C) $17,000 D) $15,000

He may deduct (under the at-risk limitations) his share of the partnership loss to the extent of his investment in the partnership. With a $17,000 loss and only $15,000 in income, James can only deduct $15,000 of the loss.

Denise earns $155,000 annually. The following summarizes her other income tax information for the current tax year: Made a $6,000 IRA contribution Received $1,700 interest from a private-activity municipal bond Received $300 for jury duty Received $12,000 of alimony from her former spouse (divorced in 2016) Received child support of $24,000 Denise has a 401(k) available through her employer, but neither she nor her employer has ever contributed to the account. Based on the information given, how much total income does Denise have for tax purposes? A) $167,300 B) $161,800 C) $149,500 D) $192,700

Her total income (the starting point for the federal income tax calculation) is composed of the $155,000 wages received, the $12,000 alimony received, and the $300 of jury duty fees. The private-activity municipal bond interest generally is an AMT preference item, but is excluded for regular income tax purposes. The child support received is also excluded from income. The IRA contribution would be deductible, as she is not an active participant, but it is deductible in arriving at the adjusted gross income, not the total income.

If Phoebe files single with gross income of $86,000 and taxable income of $75,000, what is her marginal tax rate? Refer to the 2022 tax table provided in your course references. A) 24.00% B) 22.00% C) 14.47% D) 17.49%

His marginal tax bracket is 22%.

Your clients, Jane and Mark, are contemplating the purchase of a condominium to use as a rental property. They would manage the property themselves and anticipate that it would generate losses for the first few years, at least. Which of the following statements are CORRECT with respect to active participation rental real estate? 1) The interest may be held through a limited partnership. 2) A deduction-equivalent tax credit of up to $25,000 is available. 3) The taxpayer must hold a 10% or greater ownership interest in the property. 4) The taxpayer must participate in the management of the property in a bona fide sense. A) III and IV B) I and II C) I, II, and III D) II, III, and IV

III and IV are the only choices that are correct, by definition. The interest in the property may not, by definition, be held through a limited partnership, and up to $25,000 refers to losses that may be deducted, not credits that may be taken. The deduction-equivalent tax credits mentioned in option II are relevant with respect to low-income housing and historic rehabilitation passive activities.

Frank Swanson anticipates adjusted gross income of $80,000 during the current tax year. He is considering making a gift of real estate to the public university he attended. Frank's adjusted basis in this real estate is $50,000. The real estate has a current fair market value of $70,000. Frank has owned the real estate for 19 months. If Frank donates the real estate, what is the maximum allowable charitable deduction Frank can receive for the current tax year? A) $50,000 B) $40,000 C) $24,000 D) $70,000

If Frank makes a 50% election, he must utilize the basis of the property but may deduct up to 50% of AGI. This yields a $40,000 current-year deduction with a $10,000 carryforward. If no 50% election were made, the deduction would be based on the fair market value of the property but would be limited to 30% of AGI, which is $24,000, with a $46,000 carryforward.

What important information could you tell your nonresident client (married to a U.S. citizen) about electing to be treated as a resident alien for tax purposes? You will need to pay taxes only on your U.S. income. You will need to pay taxes on your worldwide income. This will create an immigration benefit. You should obtain an individual taxpayer identification number (ITIN) or Social Security number. A) II and IV B) II, III, and IV C) I only D) I and IV

If married to a U.S. citizen or resident alien, the nonresident alien can elect to be treated as a resident alien for tax purposes only. Tax status does not necessarily reflect immigration status. If this election is made, the couple must pay U.S. taxes on their worldwide income. In this situation, the nonresident alien spouse should obtain an Individual Tax Identification Number (ITIN). Depending on their individual situation and intentions, they may apply for a Social Security number with the Social Security Administration.

If a vacation home is rented for 14 days or less during the year, which one of the following statements is CORRECT? A) The full amount of home mortgage interest is permitted as an itemized deduction. B) A portion of the rental income may be nontaxable. C) Repair expenses attributed to the rental activity are deductible. D) Typically, only a small amount of cost recovery deductions is allowed for the year.

If property is rented fewer than 15 days per year, the full amount of home mortgage interest, taxes, and casualty losses are permitted as an itemized deduction (not expenses, though); in addition, rental income can be excluded from gross income.

Three years ago, Sam received a gift of 100 shares of common stock from his uncle. The fair market value of the stock on the date of the gift was $12 per share. His uncle had purchased the stock four years earlier at $5 per share. Sam sold this stock for $17 per share last week. What was Sam's per-share basis in the stock when it was sold? A) $12 B) $17 C) $22 D) $5

If the fair market value on the date of the gift is greater than the donor's adjusted basis, the donor's adjusted basis is used as the recipient's basis. Note that the donor's holding period would be tacked to the donee's holding period.

In 2022 Mason is working with his attorney and his CFP® professional on the final documents concerning his divorce from Kristen. The sources for the cash and property for Kristen have been identified and are to be received by Kristen over eight years. If alimony payments are specified to continue after Kristen's death, which of the following statements regarding such payments is CORRECT? A) The payments are not considered alimony. B) The alimony is both deductible by Mason and is taxable income to Kristen. C) The alimony is a deductible expense. D) The total future amounts become taxable income.

If the payments are scheduled to be paid beyond the death of the payee, the payments are generally not deductible and are no longer considered alimony but a disguised property settlement. Other requirements that must be met for the alimony to be deductible are that the payments be in cash and pursuant to a divorce or separation agreement.

Joy is transferring ownership in a $250,000 whole life insurance policy on herself to her former spouse Jack, pursuant to their divorce decree. At the time of transfer to Jack, Joy's basis in the policy was $47,500 and the cash value was $60,000. What is Jack's basis in the life insurance policy on Joy? A) $60,000 B) $12,500 C) $0 D) $47,500

Jack's basis in the policy is the same as Joy's basis: $47,500. A transfer of assets between spouses incident to a divorce is income tax free and the transferee assumes the transferor's basis in the property on the date of transfer.

Kumail created an irrevocable trust for the benefit of his dependent children. Kumail named the local bank as trustee of the trust and authorized it to invest in stocks, bonds, and negotiable certificates of deposit. Included in the investment authority is the right to use trust income to purchase insurance on Kumail's life. This year, the trustee uses all of the income from the trust to pay premiums on life insurance on Kumail. All funds are currently invested in high-yielding bonds paying 4% semiannual interest on a face value of $100,000. Which taxpayer must pay tax on the income of the trust, and why? A) The trust, because it is irrevocable with no benefits to the grantor B) Kumail, because the income is (or may be) used to purchase insurance on Kumail's life C) The children, because they are the designated beneficiaries D) The bank, because of its broad authority as trustee

If the trust income is, or may be, used to purchase insurance on the life of the grantor or the grantor's spouse, then the trust is a grantor trust. This is unlike a typical ILIT, where trust corpus, not income, is used to pay the premiums.

Ron, age 43, and Sandy, age 41, are married with two children: Michael, age 12, and Victoria, age 8, who has been blind since her birth. Ron is an architect and general partner with XYZ partnership. Sandy is self-employed as an attorney and works out of a home office. Her home office is exclusively and regularly used for business, and the home office is her principal place of business. Their information for the tax year 2022 is as follows: Adjusted gross income: $217,300 Itemized deductions (including qualified residential mortgage interest, taxes paid, and charitable contributions): $33,000 Early in the current year, Sandy's father died. Sandy is the sole beneficiary of her father's entire estate. The estate is presently in the probate process. Sandy's mother, Lisa, age 68, has moved in with them but provides her own support. She was married to Sandy's father when he died earlier this year. This is Ron's second marriage. He makes monthly support payments to his former spouse and his daughter. Because both Ron and Sandy are considered to be self-employed, they make quarterly estimated tax payments each year to cover both their income tax and self-employment tax obligations. Ron's divorce decree specifies that the payment made to his former spouse is $300 per week until his former spouse dies, at which point payments will continue to be made to her estate until her daughter is age 18. Based on the information provided in the case scenario for Ron and Sandy, what amount per week is considered to be alimony? A) $150 B) $0 C) $300 D) $50

If there is an obligation to continue to make payments after the death of the ex-spouse, the payments are not treated as alimony.

Which form should a nonresident alien file, if they earned income during the taxable year and if not electing to be a resident? A) Form 1040 B) Form 706 C) Form 1041 D) Form 1040NR

If they earned U.S. income during the year, nonresident aliens must file Form 1040NR. However, if married to a U.S. citizen or resident alien, the nonresident alien can elect to be treated as a resident alien for tax purposes only. Tax status does not necessarily reflect immigration status. If this election is made, the couple must pay U.S. taxes on their worldwide income. In this situation, the nonresident alien spouse should obtain an Individual Tax Identification Number (ITIN). Depending on their individual situation and intentions, they may apply for a Social Security number with the Social Security Administration.

Imputed loan interest is usually taxable and occurs when A) a company loan is provided to an employee. B) the loan is secured by specific real property. C) a loan is provided by a related person at below-market terms. D) bonds are gifted by an employer.

Imputed loan interest is usually taxable and occurs when a loan is provided by a related person at below-market terms.

Which one of the following amendments to the U.S. Constitution established federal taxation in the U.S.? A) Fifteenth Amendment B) Eighteenth Amendment C) Sixteenth Amendment D) Seventeenth Amendment

In 1913, the required number of states ratified the Sixteenth Amendment, and taxation of income began on March 1 of that year. Since that time, the federal income tax system has played an important role in an individual's investment decisions.

Your client, Hal Meyer, will receive a deductible loss of $15,100 from a working oil and gas interest. Hal is in a 35% marginal income tax bracket and has asked you the approximate amount of tax savings that this will generate. What is the approximate amount, if any, of tax savings generated by this loss? A) $10,400 B) $5,200 C) $15,600 D) $0

In a 35% tax bracket, a $15,100 loss deduction will save $5,285. Thus, $5,200 is the closest answer.

Ed was divorced from his spouse, Julie, in 2018. Julie was granted custody of their only child, Sally, age 5. Ed was ordered to pay $2,500 per month to Julie until Sally reaches age 18. At that time, the payments are to decrease to $1,200 per month. What portion of each payment, if any, is deductible by Ed as qualifying alimony? A) A total of $1,300 of each payment is deductible by Ed. B) None of each payment is deductible by Ed. C) A total of $1,200 of each payment is deductible by Ed. D) All of each payment is deductible by Ed.

In a divorce decree, any amount tied to the happening of a contingency or an event related to a minor child is deemed to constitute nondeductible child support. In this situation, $1,300 is tied to Sally turning 18. Thus, only the remaining $1,200 is considered to be alimony.

A) For a gift loan, the amount of the imputed interest is treated as a gift from the lender to the borrower. B) In a compensation-related loan, the employer has interest income and compensation expense in the amount of the imputed interest, but the employee has received a gift from the employer in the same amount. C) If the borrower's net investment income for the year does not exceed $1,000, no interest is imputed on loans of more than $100,000. D) For gift loans greater than $10,000 and less than or equal to $100,000, a minimum of $1,000 of interest must be imputed.

In a gift loan, a gift has been made by the lender to the borrower in the amount of the imputed interest. In a compensation-related loan, the employer has interest income and compensation expense for the amount of the imputed interest. The borrower will have compensation income and interest expense, which may or may not be deductible, for the same amount. For gift loans greater than $10,000 and less than or equal to $100,000, if the borrower's net investment income for the year does not exceed $1,000, no interest is imputed. For a gift loan of more than $100,000, the prevailing federal rate of interest will be imputed.

Imputed interest on a below-market loan (with the IRS providing accepted loan rates) will be paid, unless the gift loan is A) less than $10,000 and the gift loan recipient has less than $1,000 in interest income. B) from a corporation to a shareholder. C) less than $5,000 and the loan recipient has no interest income. D) between friends.

In a gift loan, the amount of the imputed interest constitutes a gift from the lender to the borrower. For gift loans greater than $10,000 and less than or equal to $100,000, no interest is imputed if the borrower's investment income for the year does not exceed $1,000. For a gift loan of more than $100,000, the prevailing federal rate of interest will be imputed.

Charley lent his friend, Richard, $17,000 for a down payment on a home in a no-interest loan early in the current year. Charley had investment income of $750, and Richard had investment income of $1,200 in the same year. The federal interest rate is 3.5%. Richard has been making payments each month. What recommendations do you make for accounting for the loan made to Richard by Charley? A) Because this is a gift loan greater than $10,000 but less than or equal to $100,000, no interest will be imputed to the loan. B) Charley must develop an amortization schedule using the federal rate of 3.5% to account for Richard's payments of principal and interest. C) Imputed interest is calculated on the loan to Richard and is considered a gift to Richard from Charley. D) Because Charley's investment income is less than $1,000 this year, no interest is imputed to the loan.

In a gift loan, the amount of the imputed interest constitutes a gift from the lender to the borrower. For gift loans greater than $10,000 and less than or equal to $100,000, no interest is imputed if the borrower's investment income for the year does not exceed $1,000. For a gift loan of more than $100,000, the prevailing federal rate of interest will be imputed. For this loan, Richard's investment income exceeds $1,000 and interest will be imputed.

Which of the following statements is accurate with respect to a like-kind exchange? A) No gain will be recognized unless the taxpayer receives boot. B) No gain will be recognized on the exchange of inventory. C) Gain recognized is equal to the gain realized on the exchange plus the boot received. D) The amount of gain recognized will reduce the taxpayer's basis in the property received.

In a like-kind exchange, the gain recognized is always the lesser of the gain realized or the boot received. If there is no boot received, there is no gain recognized. Inventory is not eligible for like-kind exchange treatment—thus, gain would be recognized. The basis in the acquired property is the FMV of the acquired property, reduced by the gain realized but not recognized (the deferred gain).

Elliot is an accountant with his own business, E&J Tax Service. The business was organized and is currently operated as a sole proprietorship. Elliot is presently in the maximum marginal income tax bracket. As a tax planning device, he is considering reorganizing the tax service into an S corporation with himself and his 18-year-old son, Bill, as shareholders. Elliot is hopeful that Bill's status as a corporate shareholder will encourage him to eventually join the business. What is one income tax implication of the proposed S corporation as an intrafamily transfer? A) All of the income will be taxed to Elliot as an improper assignment of personal service income. B) A proportion of the income will be taxed to Bill under the family attribution rules in tax law. C) Bill will owe self-employment tax on his share of the income. D) A proportion of the income will be taxed to Bill as a percentage owner of the corporation.

In order for an S corporation (or partnership) to function as an income-splitting device, capital must be a material income-producing factor. In the case of a tax service, the performance of personal service by Elliot is the material income-producing factor. Thus, the income would be taxed to Elliot. The flow-through of income from the S corporation is not subject to the self-employment tax.

Which of the following applies to the at-risk rules, as related to passive loss restrictions for partners? 1) It is the maximum deductible loss for an investment limited to the amount of risk that the taxpayer has at the end of the current year. 2) Determining the amount at risk includes the adjusted basis of other property contributed to the partnership. 3) The inclusion of nonrecourse financing is essentially the only difference between the basis in a partnership and the amount at risk. A) II only B) I only C) I and II D) I, II and III

In the Tax Code, the at-risk rules are defined as the maximum deductible loss for an investment limited to the amount that the taxpayer-investor has at risk at the end of the current year (i.e., the amount of potential economic loss). A partner may deduct losses only to the extent of the amount that they have "at risk." The amount at risk equals the sum of the following: -The money invested (except to the extent the money invested was borrowed and was secured only by the investment) -The adjusted basis of other property contributed to the partnership -Amounts borrowed for use in the activity, but only to the extent that the partners are personally liable for repayment of the debt (recourse indebtedness) -The partner's share of income, less the partner's share of losses or withdrawals from the partnership -The proportionate share of qualified nonrecourse financing in a real estate activity ONLY Essentially, the only difference between the amount at risk and the basis in a partnership interest is the treatment of nonrecourse financing.

Susan received 100 shares of stock as a gift from her uncle, Carl. Carl purchased the stock 15 years ago for $12 per share. Susan received the stock from Carl two months ago, when the fair market value of the stock was $15 per share, and she sold the stock this week for $19 per share. What is the amount and character of Susan's gain from the sale of the stock? A) $400 short-term capital gain B) $700 short-term capital gain C) $700 long-term capital gain D) $400 long-term capital gain

In the case of an asset received as a gift, where the fair market value on the date of the gift is greater than the donor's adjusted basis, the recipient has a carryover basis. In this case, Uncle Carl had purchased the stock for $12 per share and had gifted it to Susan when the fair market value was $15 per share. Susan subsequently sold the stock for $19 per share. Thus, the carryover basis from Uncle Carl would be $12 per share. In a situation where the recipient of the gift takes the donor's basis, the holding period is tacked. In other words, the donor's holding period is added to the donee's holding period. Thus, Susan is treated as holding the stock for over 15 years.

Dave owns equipment that has an adjusted basis of $10,000 and a fair market value of $75,000. Through an exchange, he acquires new equipment from Rachel that has a fair market value of $60,000 and an adjusted basis of $35,000. In the exchange, Dave receives $15,000 from Rachel. What is the amount of gain or loss, if any, recognized by Dave in the exchange? A) $15,000 B) $10,000 C) $65,000 D) $50,000

In the exchange, Dave received new equipment with a fair market value of $60,000 and cash of $15,000. He gave up an adjusted basis in his property of $10,000. The difference between $75,000 and $10,000 is the gain realized, $65,000. Because this is not realty, it is not eligible for 1031 exchange treatment. Thus, the transaction is treated as a sale and subsequent purchase. The entire gain of $65,000 is recognized and taxable.

Your client, Joe, has active income of $300,000 per year and substantial unused passive losses from a nonpublicly traded limited partnership. He would like to find an investment that would allow him to utilize his passive losses. Which of the following are the most appropriate investments for Joe? 1) An active participation rental real estate activity generating income 2) A master limited partnership (MLP) generating income 3) Certificates of deposit generating portfolio income 4) A nonpublicly traded limited partnership generating income A) I, II, and IV B) I and II C) I and IV D) III and IV

Income from active participation rental activities is considered passive income. The nonpublicly traded limited partnership losses may not offset income from a publicly traded partnership (the MLP) or portfolio income.

On April 1 of the current tax year, Susan sold her principal residence for a total price of $501,000; $301,000 was in cash, with the buyer assuming a $200,000 mortgage on the house. Susan purchased the house 15 years ago for $290,000. She has not made any improvements to the house. To assist in the sale of the residence, she incurred costs of $1,500 for repairs three weeks before the sale occurred. Realtor commissions of $31,000 resulted from the sale. On May 1 of the current tax year, Susan bought a new residence for $260,000. Assume that Susan is considering renting out her new residence for two weeks (14 days) during the upcoming tax year. However, she is unsure of the income tax consequences. Which one of the following statements is CORRECT? A) The rental income is includible in income, but mortgage interest and property taxes allocable to the rental are deductible for AGI. B) The rental income is includible in full in gross income. C) The rental income is not includible in income. D) The rental income may or may not be includible in income, depending on the amount.

Income from rentals for fewer than 15 days during the year are not required to be included in gross income. However, no deductions attributable solely to the rental are allowed, either. The home mortgage interest and property taxes are still deductible in full as itemized deductions.

Maxine, an individual taxpayer, donated $100,000 in cash to a qualified public charity in Year 1. Her adjusted gross income was $150,000 in Year 1 and $150,000 in Year 2. She makes no donations to charity in Year 2. How much of a tax deduction will she be allowed for this gift in each of the two tax years? A) Year 1: $45,000; Year 2: $0 B) Year 1: $75,000; Year 2: $5,000 C) Year 1: $45,000; Year 2: $35,000 D) Year 1: $90,000; Year 2: $10,000

Individual cash donations to qualified public charities are limited to 60% of adjusted gross income, but excess amounts may be carried over in subsequent tax years. Sixty percent of Maxine's Year 1 AGI was $90,000. In Year 1, she can deduct $90,000 of the gift. In Year 2, she can deduct the remainder of $10,000.

Victoria Glass has $6,500 of investment interest expense and net investment income of $6,000 in the current tax year. She paid broker's commissions of $1,000 during the tax year. How much investment interest expense, if any, may Victoria deduct in the current tax year? A) $5,000 B) $5,500 C) $6,500 D) $6,000

Investment interest expense is deductible only to the extent of net investment income.

Carol has investment interest expenses of $11,000 from her margin account. She had municipal bond interest income of $6,000, corporate bond interest income of $5,000, and qualified dividends of $2,000. She will not forgo the 15% tax rate on the qualified dividends. What is her investment interest expense deduction? A) $7,000 B) $13,000 C) $5,000 D) $11,000

Investment interest expense is deductible up to the amount of investment income. Carol is limited to a $5,000 investment interest deduction or only the amount of bond taxable bond interest, which is her investment income. She may not treat qualifying dividend income for which she uses the 15% long-term capital gain rate as investment income, nor can she include nontaxable municipal bond interest as investment income.

Tom Bell has investment income (interest) of $8,000 in the current year. He paid $1,200 in investment adviser fees and had $7,000 of investment interest expense. His AGI is $35,000. What amount of investment interest expense may be deducted in the current year as an itemized deduction? A) $8,000 B) $6,800 C) $6,500 D) $7,000

Investment interest expense is deductible up to the amount of investment income. The investment income is the interest income of $8,000. However, the deduction cannot exceed the actual investment interest expense of $7,000.

For the current tax year, Bob Phillips, an individual taxpayer filing a joint return, has $50,000 of investment interest expense and $20,000 of net investment income (interest and dividends). Bob's AGI is $200,000. How much investment interest expense, if any, may Bob deduct in the current tax year? A) $0 B) $50,000 C) $21,000 D) $20,000

Investment interest expense is deductible up to the amount of net investment income. The problem tells us that the net investment income is $20,000; thus that is the maximum deduction. The fact that the dividends are included in the net investment income indicates that the taxpayer elected to include them in investment income and is forgoing the preferential rates associated with qualified dividends. The AGI has no bearing on the answer.

Bruce and Melissa Parish, married taxpayers filing jointly, have the following items related to their investments during the current tax year: Investment interest expense $5,000 Interest income $2,500 Short-term capital gains $1,000 Investment adviser's fees $1,250 Commissions paid on stock purchase $200 Adjusted gross income $60,000 What is the Parishes' allowable investment interest expense deduction for the current year? A) $3,500 B) $5,000 C) $3,450 D) $3,250

Investment interest expense is limited to the taxpayer's net investment income of $3,500.

Which one of the following is the best description of itemized deductions? A) Trade or business expenses deductible from adjusted gross income B) Personal expenses deductible from adjusted gross income C) Personal expenses deductible in arriving at total income D) Trade or business expenses deductible in arriving at total income

Itemized deductions are generally personal expenses (e.g., home mortgage interest, medical expenses) that are specifically allowed as a deduction from AGI in arriving at taxable income. They are not deductible in arriving at total income.

Which of the following is a tax preference item for the purpose of calculating the alternative minimum tax? 1) Tax-exempt interest from a private-activity bond issued in 2008 2) Cash contributions to charitable organizations 3) Cash flows from limited partnerships 4) Personal-service income in excess of tax losses A) I and II B) I, III, and IV C) II only D) I only

Items II, III, and IV are not tax preference items. However, other AMT tax preference items include the part of the deduction for certain depletion that is more than the adjusted basis of the property and the excluded gain on the sale of certain small business stock (Section 1202).

Which one of the following employee benefits is NOT currently deductible by a C corporation? A) Group legal services B) Health and accident insurance C) Group term life insurance of $100,000 D) Qualified employee discounts

Items that are currently deductible by a C corporation would include group term life insurance, group legal services, and health and accident insurance. Qualified employee discounts do not generate a deduction for the corporation—the discount merely results in less sales revenue being brought into the organization.

Jane owns a printing business. She wants to trade her old copiers for new fax machines. In the contemplated exchange, Jane will pay $750 in cash. Additional information related to the transaction is given as follows: The copiers have an adjusted basis of $1,500. The copiers have a fair market value of $1,000. The fax machines have a fair market value of $1,750. What is Jane's recognized gain or loss in this exchange? A) ($250) B) $500 C) $0 D) ($500)

Jane is paying $750 plus the adjusted basis of $1,500 ($2,250); compared to the fair market value of the property received of $1,750, thus yielding a $500 loss. Because this is not real estate, this is not a like-kind exchange. This exchange is simply treated as a sale of the asset. A loss on a Section 1231 asset may be recognized in the year of the loss. Personalty does not qualify for like-kind exchange treatment.

Jane, age 35, whose filing status is single, earned a salary of $55,000 in 2022. She also made a $2,000 contribution to her Roth IRA for 2022. Jane had a capital loss of $3,000 during the year. Her uncle, Charles, gave her $100,000 in municipal bonds for which she earned interest of $3,500. In her employment as a sales representative for her company, Jane incurred $650 of unreimbursed business expenses. What is Jane's adjusted gross income (AGI)? A) $53,800 B) $56,800 C) $52,000 D) $53,300

Jane's AGI is $52,000 ($55,000 ‒ $3,000). Jane's $3,000 capital loss is a deduction for calculating AGI. Roth IRA contributions are never deductible from gross income. Municipal bond interest is not included in income. The unreimbursed business expenses are not deductible.

Jean's mother dies and leaves her house to Jean this year. The house is valued at $40,000. Jean rents the house to tenants for $1,000 per month following her mother's death. What amount must Jean include in her annual gross income for this year? A) The lesser of the FMV of the house or the amount of rental income she receives B) $40,000 plus the amount of rental income she receives C) The amount of rental income she receives D) $40,000

Jean receives the house on a tax-free basis because inheritances and gifts are not taxable to the recipient. However, any income generated by the house, such as rental income, is subject to taxation.

In February 2022, Jerry's manufacturing equipment was completely destroyed in a fire. Jerry determined that the adjusted basis of the equipment was $30,000. The original cost of the equipment was $200,000. In November 2022, he received a check of $200,000 from the insurance company. When does the replacement period end for the involuntary conversion? A) December 31, 2023 B) December 31, 2022 C) December 31, 2024 D) November 30, 2024

Jerry has until December 31, 2024, to purchase the replacement property because the replacement period for a casualty or theft ends on the last day of the second taxable year following the year in which the gain was realized.

Jim owns an apartment building with a fair market value of $225,000 and an adjusted basis of $85,000. He wants to acquire Frank's duplex, which has a fair market value of $240,000 and an adjusted basis of $130,000. In the exchange, Jim will pay Frank $15,000 in cash. What is Jim's substitute basis in the acquired duplex? A) $100,000 B) $140,000 C) $225,000 D) $240,000

Jim is receiving an FMV of $240,000 for the duplex. He is giving up an adjusted basis of $85,000 plus $15,000 cash. The difference between the $240,000 received and the $100,000 given up is the realized gain of $140,000. The gain recognized (the taxable amount reported on the income tax return) in a like-kind exchange is the lesser of gain realized ($140,000) or boot received ($0). The substitute basis in an asset acquired in a like-kind exchange is the FMV of the qualifying property received ($240,000) reduced by the gain realized, but not recognized ($140,000 - $0 = $140,000). Thus, $240,000 - $140,000 = $100,000.

On December 20, 2005, Jody moved into a condominium that she owns and had rented to tenants since July 1, 1998. Her cost basis in the condo was $238,440. Jody took depreciation deductions totaling $54,000 for the period that she rented the property. After moving in, she used the residence as her principal residence. Jody sells the property on August 1, 2022, for $538,000. Jody is in the highest marginal income tax bracket for the current year. What is the amount and character of the recognized gain resulting from the sale? A) $54,000 of ordinary income; $49,560 of "regular" long-term capital gain B) $54,000 of unrecaptured Section 1250 income; $299,560 of "regular" long-term capital gain C) $54,000 of unrecaptured Section 1250 income; $49,560 of "regular" long-term capital gain D) $353,560 "regular" long-term capital gain

Jody's gain realized (the actual economic gain) from the sale is $353,560 ($538,000 of sales proceeds reduced by the adjusted basis of $184,400). Of this $353,560 of gain, the first $54,000 is recognized as unrecaptured Section 1250 gain, taxed at 25%. Unrecaptured Section 1250 gain is the gain created by the straight-line depreciation. This leaves $299,560 of gain to account for. Jody used the condo as her principal residence for two full years—thus, she is eligible to exclude $250,000 under Section 121. This leaves $49,560 of long-term capital gain to be recognized at a 20% rate (because she is in the highest marginal income tax bracket, her taxable income exceeds the $445,850 breakpoint for the 20% LTCG rate). The recognized gain is the gain on which Jody will pay taxes. Note that the nonqualified use provision does not come into play here as there was no nonqualified use after 2008.

Joe has $60,000 in wages, $13,000 in deductions, and $5,000 in passive losses this year. The passive loss did not arise from rental property or real estate. What is Joe's taxable income this year? A) $60,000 B) $47,000 C) $48,000 D) $44,000

Joe's taxable income is $47,000 ($60,000 − $13,000). Passive losses may only offset passive income so his $5,000 loss is not allowed, except for rental and real estate property. Rental and real estate passive losses are allowed up to $25,000 and are subject to the AGI phase out limitations.

Lindsey is age 2 and she received $6,000 in municipal bond interest income and $900 in other interest income in 2022. What is the total federal income tax due on her income in 2022? A) $90 B) $1,495 C) $0 D) $1,400

Lindsey owes no federal income taxes in 2022. Municipal bond interest income is not taxable. The $900 in other interest income is less than Lindsey's $1,150 standard deduction amount.

Kate is an investor in a nonpublicly traded limited partnership. She invested $50,000 in cash and is personally liable for $5,000 of the partnership's debt. What is Kate's capital at risk in the partnership? A) $0 B) $5,000 C) $50,000 D) $55,000

Kate's at-risk amount in the nonpublicly traded limited partnership is $55,000. This includes the $50,000 in cash she invested and the $5,000 in partnership debt for which she is personally liable (recourse financing).

During the current tax year, Jim purchased a warehouse for exclusive use in his manufacturing business. The cost of the property was $620,000, of which $100,000 was attributable to the land. Which of the following statements identify the proper treatment of the expenditure? 1) A portion of the cost attributable to the building may be deducted under Section 179. 2). The $100,000 attributable to the land must be capitalized and may not be depreciated. 3). The $520,000 attributable to the building must be capitalized and depreciated. 4) The entire $620,000 must be capitalized and depreciated. A) I and II B) IV only C) II only D) II and III

Land is not a depreciable asset—only "wasting" assets are subject to depreciation. The building must be capitalized and depreciated over a period of 39 years. Section 179 generally does not apply to realty; it applies to tangible personalty used in the active conduct of a trade or business.

Legally married spouses may file a joint return A) even if their tax years do not begin on the same date. B) if neither spouse is a nonresident alien at any time during the year. C) only if both spouses have income or deductions. D) if separated by a divorce decree, but the divorce is not final.

Legally married spouses may file a joint return (even though one spouse has no income or deductions) if their tax years begin on the same date; they are not legally separated under a decree of divorce or separate maintenance on the last day of the calendar year; and neither is a nonresident alien at any time during the year.

Samantha received the following dividends in 2022 from her portfolio: Ordinary dividends from HOT stock, a publicly traded company Dividends from Sky High Realty and Trust, a publicly traded REIT Life insurance dividends from her whole life policy Qualified dividends from BET stock, a publicly traded company Which of the above is NOT considered taxable? A) II and IV B) IV only C) III only D) I and II

Life insurance dividends are considered a return of premium paid (provided the cumulative dividends received over the life of the policy do not exceed the basis in the policy) and thus are not taxable. The other choices listed are taxable. Qualified dividends are eligible for long term capital gains rates. REIT dividends may qualify for a QBI deduction but nonetheless will still be taxable.

Lindsey is age 2 and her total income was $2,300 in qualified dividends in 2022. What is the tax on the dividends at Lindsey's rate? A) $95 B) $0 C) $115 D) $30

Lindsey is in the 10% marginal income tax bracket. She can use the long-term capital gains tax rate on qualified dividends received. At her income and filing status, that capital gain tax rate is 0%.

Ethel had the following from securities transactions during the current year: Long-term capital gain: $6,400 Long-term capital loss: $2,200 Short-term capital gain: $2,300 Short-term capital loss: $5,500 Which of the following describes the net capital gain or loss reportable by Ethel for the current tax year? A) $1,000 net long-term capital loss B) $900 net long-term capital gain; $100 net short-term capital loss C) $1,000 net long-term capital gain D) $4,200 net long-term capital gain; $3,200 net short-term capital loss

Long-term transactions are netted together, as are short-term transactions. The net long-term capital gain is $4,200 ($6,400 - $2,200). The net short-term capital loss is $3,200 ($2,300 - $5,500). The net short-term capital loss is netted with the net long-term capital gain ($4,200 - $3,200) to result in a net long-term capital gain of $1,000.

Jim, a single taxpayer, sells Section 1244 stock for a $250,000 loss during the tax year. The stock had been held for two years. Which one of the following is CORRECT regarding the treatment of the loss? A) $100,000 is treated as an ordinary loss; $150,000 is treated as long-term capital loss. B) $250,000 of the loss is fully deductible as a capital loss. C) $50,000 is treated as an ordinary loss; $200,000 is treated as long-term capital loss. D) $50,000 is treated as an ordinary loss; $200,000 is treated as a Section 1244 carryforward.

Loss on the sale of Section 1244 stock is deductible as an ordinary loss up to $100,000 per year on a jointly filed return, or $50,000 on any other return (which applies to Jim as a single taxpayer). Any Section 1244 loss in excess of the annual limits is treated as a capital loss—long term or short term, depending on the holding period. The $100,000/$150,000 option would be correct if Jim were a married taxpayer filing jointly.

Jim, a married taxpayer filing a joint return, owns Section 1244 stock. Jim paid $195,000 for the stock many years ago. This year, the corporation filed for bankruptcy and the stock became worthless. What is the nature and character of the loss on the stock? A) $100,000 ordinary loss; $95,000 long-term capital loss B) $195,000 ordinary loss C) $50,000 ordinary loss; $145,000 long-term capital loss D) $0 loss, as worthlessness of the stock is not a recognition event

Loss on the sale, exchange, or worthlessness of Section 1244 stock is deductible as an ordinary loss up to $50,000 per year, and $100,000 per year on a jointly filed return. Any excess loss in a given year is treated as a capital loss. In this case, the capital loss is long term, due to the more than one-year holding period.

John has the following items from four separate investments during the current tax year: -Passive income from a publicly traded limited partnership: $8,000 -Passive loss from a publicly traded limited partnership: $10,000 -Passive income from a nonpublicly traded limited partnership: $17,000 -Passive loss from a nonpublicly traded limited partnership: $9,000 -What is the total amount, if any, of passive losses that may be deducted during the current year? A) $9,000 B) $19,000 C) $17,000 D) $0

Losses from a non-PTP may be deducted up to the amount of income from a non-PTP. In this situation, the passive loss of $9,000 may be deducted in full against the $17,000 of passive income. The income from a PTP may not be offset by passive losses arising from any other source, and the losses from a PTP must be held in suspense until that same partnership generates income.

Marcus purchased a diamond ring for $15,000 10 years ago. It was stolen in March this year. The ring was purchased to celebrate achieving a significant promotion at work. The FMV at the time of the theft was $20,000. The ring was insured, and after the deductible, Marcus received $19,000 from the insurance company. Marcus replaced the ring with a new one for $20,000. Under Section 1033, what is Marcus's new basis in the replacement ring? A) $19,000 B) $20,000 C) $16,000 D) $15,000

Marcus's deferred gain on the new ring is $4,000. His new basis is the FMV of the property at acquisition minus the deferred gain ($20,000 − $4,000 = $16,000).

Max and his mother, Lucy, live together in his home. Lucy is bedridden, and Max must pay a caregiver to provide meals and other aid during the day so he can leave the house and work. He pays $5,000 annually for this service. His mother has no income, and he is her full support. What tax relief may be available to Max for the expenses of caring for his elderly mother? A greater standard deduction amount Child and dependent care credit A) Neither I nor II B) II only C) Both I and II D) I only

Max is entitled to both the standard deduction amount for the head of household filing status and the child and dependent care tax credit, subject to the limits based on Max's AGI.

Which of the following are adjustments to gross income (above-the-line deductions)? Medical expenses Capital losses Deductible IRA contributions A) II and III B) I and II C) I and III D) II only

Medical expenses are an itemized (below-the-line) deduction.

Harriet builds remote-control cars as a hobby. Which hobby expenses must she deduct from hobby income first? A) None are deductible B) Operating expenses C) Cost of goods sold D) Cost recovery deductions

Most hobby expenses are not deductible with the notable exception of cost of goods sold.

A business fails and the owners lose their investment in the unincorporated business, along with their personal residences, automobiles, and other personal property. What type of business entity does this suggest? A) Preferred shareholders in a corporation B) Common shareholders in a corporation C) General partners in a general partnership D) Limited partners in a limited partnership

Multiple ownership of an unincorporated business is characteristic of a general partnership. More than one owner personally liable indicates a general partnership business form. Common shareholders, preferred shareholders, and limited partners do not have personal property at risk; only their investment in the company is at risk.

Jeffrey and Karen have given cash gifts to their children over the years. In addition, in 2022 - Mark, age 13, earns $2,500 in salary. - Jennifer, age 19, who attends community college for approximately three months per year, earns $2,300 in dividends and capital gains. - Nancy, age 12, earns $2,950 in dividends and interest. - Steven, age 10, earns $900 in dividends and interest. Whose income is subject to the tax at the parents' marginal rate? A) Steven's B) Jennifer's and Nancy's C) Nancy's D) Nancy's and Mark's

Nancy is the only child up to and including age 18 with unearned income in excess of $2,300 for 2022. Earned income is not subject to taxation at the parental rate. Jennifer is not subject because she is not a full-time student. The kiddie tax applies to children under 19 years of age. It also applies to children under age 24 if they are full-time students. The kiddie tax does not apply if the child's earned income exceeds one-half of the child's support. A full-time student is an individual who is a full-time student for at least five calendar months during the tax year.

Neil McElroy is an engineer for Causley Computer Inc. In addition, Neil operates a janitorial service that cleans several local office buildings. Neil was divorced in 2019, and his wife received custody of their two children. He has assembled the following information for preparation of his tax return for the current tax year. Neil's salary$71,500Interest income$9,500Monthly alimony paid to ex-spouse$1,500Monthly child support$500Purchase of equipment for use in janitorial service$10,000IRA contribution$6,000 Based on the information given, which of the following are fundamental methods of managing Neil's tax liability? Tax credit: Neil could take an investment tax credit for purchases of qualifying business equipment. Deductions for AGI: Neil may deduct alimony payments of $18,000 made to ex-spouse. Deductions for AGI: Neil may deduct child support payments of $6,000. Exclusions: Neil could have invested in municipal bonds to receive tax-free income. I and II B) IV only C) I, II, and IV D) II, III, and IV

Neil may not deduct alimony paid to his former spouse because the deduction is disallowed for alimony under divorce decrees in 2019 and thereafter. He may invest in municipal bonds to receive tax-free income. There is no investment tax credit for equipment purposes. Some students confuse this with the Section 179 expense election, but that provision provides a deduction, not a credit. Child support payments are specifically nondeductible.

Mary is an active participant in an employer-sponsored retirement plan, but her husband, Frank, is not. Their combined adjusted gross income (AGI) is $230,000 for 2022. They each contributed $6,000 to an IRA for the current year. Which of the following statements is CORRECT regarding the deductibility of the IRA? A) Frank may deduct his IRA contribution, but Mary may not. B) Neither Mary nor Frank may deduct the IRA contributions. C) Both Frank and Mary may deduct the IRA contributions. D) Mary may deduct her IRA contribution, but Frank may not.

Neither Mary nor Frank may deduct their IRA contributions. The active participant spouse is subject to a MAGI (AGI without the IRA) phaseout between $109,000 and $129,000 in 2022. The spouse who is not an active participant but whose spouse is an active participant may take a deduction for the contribution, subject to a phaseout between $204,000 and $214,000. Because the AGI exceeds $214,000, neither spouse may deduct the IRA contribution. Remember that those phaseouts apply only if the taxpayer (and/or spouse, if married) is an active participant in a company-maintained retirement plan. These phaseouts will be provided on the exam.

Which of the following statements regarding replacement property in a Section 1033 exchange is CORRECT? 1) In the taxpayer use test, the taxpayer's use of the replacement property and of the involuntarily converted property must be the same. 2) For the functional use test, the owner-investor's properties must be used in similar endeavors as the previously held properties. A) II only B) Both I and II C) Neither I nor II D) I only

Neither statement is correct. In the functional use test, the taxpayer's use of the replacement property and of the involuntarily converted property must be the same. In the taxpayer use test, the owner-investor's properties must be used in similar endeavors as the previously held properties.

Which one of the following statements is CORRECT concerning capital gains and losses? A) Net capital gains are always taxed at a flat rate of 15%. B) Net capital losses are deductible up to $3,000 annually. C) Net capital gains are always taxed at a maximum rate of 28%. D) Excess capital losses are carried forward for up to five years.

Net long-term capital gains (LTCG) (from other than unrecaptured Section 1250 income and collectibles) are taxed at rates of 0%, 15%, or 20%. For married couples filing jointly, the 0% long-term capital gain rate ends at $83,350 of taxable income. For long-term capital gains falling between the $83,350 breakpoint and $517,200 of taxable income (again, for married couples filing jointly), the rate is 15%. For long-term capital gains falling into taxable income levels above $517,200 (MFJ), the rate is 20%. The table shows the breakpoints for LTCG and qualified dividend preferential rates. LTCG Rates Based on Taxable Income Filing Status 0% rate 15% rate 20% rate Single:Under $41,675 $41,676-$459,750 Over $459,750 Head of household: Under $55,800 $55,801-$488,500Over $488,500 Married filing jointly: Under $83,350$83,351-$517,200Over $517,200 Estates and trusts: Under $2,800$2,801-$13,700Over $13,700 Special rates apply to the sale of real estate or collectibles—25% (the maximum rate for gain attributable to straight-line depreciation on real estate), or 28% (maximum rate in the case of collectibles). Net capital losses, the capital losses remaining after netting against capital gains, are deductible up to $3,000 per year with an indefinite carryforward.

The effective tax rate is obtained by dividing the amount of tax paid by A) the average tax rate. B) taxable income C) the amount of deductions and credits. D) the correct tax bracket.

The effective tax rate is found by dividing total tax by taxable income.

Patty has a $10,000 passive loss carryforward from Beta limited partnership, which is publicly traded. She also has a $15,000 passive loss carryforward from Alpha limited partnership, which is nonpublicly traded. In the current year, she has $6,000 of income from Beta. She also has $11,000 of income from Gamma LP. Gamma is not publicly traded. What is the total amount of passive losses that Patty may deduct during the current year? A) $25,000 B) $11,000 C) $6,000 D) $17,000

Of the $10,000 passive loss carryforward from the Beta limited partnership, only $6,000 may be utilized in the current year due to the $6,000 of current year passive income. A total of $11,000 in losses from the Alpha limited partnership may be utilized against the $11,000 of income from the Gamma limited partnership in the current year because both are nonpublicly traded. Thus, the total of passive losses that are allowed for the current year is $17,000.

During the current year, Chuck has Section 1231 gains totaling $14,000. He also has $3,000 of Section 1231 losses. Four years ago, Chuck reported a net Section 1231 loss of $7,000. These are the only two years in which Chuck has had Section 1231 gains or losses. What is the amount and character of the current year's Section 1231 gains and losses? A) $4,000 ordinary income, $7,000 long-term capital gain B) $11,000 long-term capital gain C) $4,000 long-term capital gain D) $7,000 ordinary income, $4,000 long-term capital gain

Of the current year's $11,000 Section 1231 gain, $7,000 is treated as ordinary income. This is due to the $7,000 of unrecaptured Section 1231 losses during the five-year lookback period. The remaining $4,000 of current Section 1231 gain is treated as long-term capital gain.

To which of the following do the passive activity loss rules apply? 1)Individuals 2) C corporations that are not closely held 3) Closely held C corporations 4)Estates A) II and IV B) I and III C) I, III, and IV D) I only

Of the listed choices, option II is the only option to which the passive activity loss rules do not apply. The passive activity loss rules specifically do not apply to C corporations that are not closely held.

Under the imputed interest rules, which of the following statements is CORRECT? A) Tax avoidance loans must be less than $10,000 to avoid the imputed interest rules. B) A loan between a lender-employer and a borrower-employee follow the same rules as gift loans between individuals. C) On gift loans between individuals in the amount greater than $10,000 and less than or equal to $100,000, the imputed interest cannot exceed the borrower's net investment income. D) If the borrower's net investment income for the year does not exceed $1,000, then $1,000 is the maximum interest that can be imputed on loans of $100,000 or less.

On gift loans between individuals in the amount greater than $10,000 and less than or equal to $100,000, the imputed interest cannot exceed the borrower's net investment income. If the borrower's net investment income for the year does not exceed $1,000, then no interest can be imputed on loans of $100,000 or less. A loan between a lender-employer and a borrower-employee is considered a compensation-related loan and the $10,001-$100,000 exception does not apply. If the principal purpose of a loan is tax avoidance none of the exceptions to the imputed interest rules apply.

Which of the following statements comparing a C corporation and a general partnership is CORRECT? Both a C corporation and general partnership can have more than one owner. Both a C corporation and general partnership are regarded as distinct entities for tax purposes. Profits are divided equally in both business forms. Neither a C corporation nor general partnership pays federal tax on income. A) I and III B) I and IV C) II only D) I only

One similarity of a partnership and a C corporation is that they may both have more than one owner. However, only a C corporation is regarded as a distinct, separate entity for tax purposes. Partnerships are flow-through entities. C corporations may retain profits. Partnerships are not required to distribute profits equally. C corporations pay tax on income.

Which of the following are qualified interest expense deductible in arriving at an individual's AGI? Student loan interest Mortgage interest on a loan acquired for a personal residence in 2022 for $600,000 Interest on home equity loan indebtedness to buy an automobile Investment interest expense A) I, II, III, and IV B) IV only C) I only D) II and III

Only item I, student loan interest, is deductible in arriving at an individual's AGI. Statements II and IV are incorrect and are deductions from AGI. Statement III is not a deductible expense. Home equity loan interest is not deductible for AGI and is only deductible from AGI to the extent the proceeds are used for home acquisition or improvement and not personal expenses.

Ted is 21 and is preparing to file his annual federal income tax return. He received the following during the calendar year: Autographed guitar won from a radio station Cash received for cutting grass in the summer Tuition scholarship to Private University Which items are excluded from gross income on his tax return? A) II and III B) III only C) I only D) I, II, and III

Only the tuition scholarship is excluded from gross income. The prize and the cash received for cutting grass are included.

Which of these are allowable itemized deductions for purposes of computing the alternative minimum tax (AMT)? Charitable deductions Qualified housing interest Medical expenses in excess of 7.5% of AGI Real estate taxes A) I and II B) II and III C) I, II, and III D) II, III, and IV

Option IV, real estate taxes, is the only itemized deduction listed that is not allowed for AMT purposes.

Which of the following are requirements for a taxpayer who materially participates in a real property trade or business to be able to deduct any losses from the business? 1) More than 50% of the individual's personal services during the tax year are performed in the real property trades or businesses in which the individual materially participates. 2) More than 10% of the individual's net assets at the end of the tax year are invested in the real property trades or businesses in which the individual materially participates. 3) The individual performs more than 750 hours of service in the real property trades or businesses in which the individual materially participates. A) I and II B) I and III C) I only D) II and III

Options I and III are specific requirements that must be met for a taxpayer to be able to deduct any losses from a real estate trade or business. The amount of the individual's net assets used in the business is not relevant.

Which of the following are requirements in order for a taxpayer who materially participates in a real property trade or business to be able to deduct any losses from the business? 1)More than 50% of the individual's personal services during the tax year are performed in the real property trades or businesses in which the individual materially participates. 2) More than 10% of the individual's net assets at the end of the tax year are invested in the real property trades or businesses in which the individual materially participates. 3) The individual performs more than 750 hours of service in the real property trades or businesses in which the individual materially participates. A) I only B) I and II C) II and III D) I and III

Options I and III are specific requirements that must be met in order for a taxpayer to able to deduct any losses from a real estate trade or business. The amount of the individual's net assets used in the business is not relevant.

Which of the following apply to the passive activity loss rules? 1) Deductible passive losses are limited to the passive gains in other passive activities. 2)Any unused passive losses may be carried forward against future passive gains. 3) When the passive activity property is disposed of, any unused passive losses can be deducted against passive gains, portfolio, or active income. 4) Passive loss rules apply only to real estate transactions. A) II and IV B) II, III, and IV C) I and III D) I, II, and III

Passive loss rules apply to all passive activities, not just real estate transactions. Passive losses may be deducted against passive gains. If the investor has excess passive losses, the losses are carried forward and may be used in future years to offset future passive gains.

Your client has a salary of $80,000, dividends of $20,000, and limited partnership income of $15,000. This year, she invested in an equipment-leasing partnership. Her initial investment included $50,000 cash and a nonrecourse note for $60,000. What is the maximum tax deduction your client may take from this equipment-leasing investment this year? A) $35,000 B) $45,000 C) $15,000 D) $50,000

Passive losses are only deductible against passive income. She has passive income of $15,000 from the limited partnership, thus she could deduct up to $15,000 of passive losses from the equipment-leasing limited partnership.

Which of the following benefits provided by an employer is taxable to its employees? A) Undergraduate and graduate education assistance, up to a maximum of $5,250. B) Expenses for qualifying work-related education of the self-employed. C) Meals provided on the employer's premises that allow employees to attend a job related education seminar. D) A company car is used for personal mileage by an employee.

Personal use of a company car is a taxable fringe benefit. The other education benefits listed are specifically excluded by the Code.

Your client Sally, age 30, is designing an educational investment program for her 8-year-old son. She expects to need the funds in about 10 years when her AGI will be approximately $70,000. She wants to invest at least part of the funds in tax-exempt securities. Which of the following investment(s) may yield tax-exempt interest on her federal return if the proceeds were used to finance her son's education? 1) Treasury bills 2) EE bonds 3) GNMA funds 4) Zero coupon Treasury bonds A) II and III B) I, III, and IV C) III and IV D) II only

Proceeds from EE savings bonds may be exempt if the proceeds are used for qualified higher-education expenses of the taxpayer, spouse, or dependent. There is an AGI phaseout, which is $85,800‒$100,800 (2022) for a single taxpayer. (The actual phaseouts are provided on the exam.) All the other options generate currently taxable income. The Treasury bills and GNMA funds both produce taxable income on the federal return (Treasury bill interest would typically be tax exempt on her state return). The zero Treasury also produces taxable income each year as the amortized discount is added to taxable income, even though no cash income is received.

During January 2012, Jacinto purchased an interest in a nonpublicly traded limited partnership that will generate a $10,000 passive loss for the current tax year. He also owns an interest in a publicly traded limited partnership that will generate passive income of $6,000 for the current tax year. How much of this passive loss, if any, is deductible by Jacinto during the current tax year? A) $0 B) $10,000 C) $4,000 D) $6,000

Publicly traded limited partnership income may not be offset by other passive losses. Thus, the $6,000 income is simply taxable in the current year. The $10,000 passive loss is carried forward, and, assuming Jacinto buys no other partnerships, may be used when that same partnership generates income, or is fully deductible upon a taxable disposition of the partnership.

Paul has the following items: Carryforward of prior year passive loss from: XYZ limited partnership (publicly traded)$(10,000)ABC limited partnership (nonpublicly traded)$(6,000) Current year passive income and loss from: XYZ limited partnership (publicly traded)$12,000 GHI limited partnership (publicly traded)$(9,000) JKL limited partnership (nonpublicly traded)$18,000 RST limited partnership (nonpublicly traded)$(14,000) What is the total amount of passive losses that Paul may deduct during the current year? A) $28,000 B) $30,000 C) $18,000 D) $14,000

Publicly traded partnership (PTP) income may only be offset by prior year losses from the same partnership. Thus, the $10,000 XYZ carryforward is deductible. Nonpublicly traded income ($18,000) may be offset by current losses ($14,000) or carryforward losses ($6,000) from any nonpublicly traded activities. Thus, the $10,000 XYZ loss and the $18,000 nonpublicly traded loss total $28,000.

Which one of the following is NOT subject to the Medicare contribution tax? A) Qualified Roth distributions B) Qualified dividends C) Income from a nonperiodic distribution from an annuity D) Long-term capital gains

Qualified Roth distributions are not subject to the Medicare contribution tax. Only taxable items, such as net capital gains, net rental income, annuity income and dividends, for example, are subject to the Medicare contribution tax.

Which one of the following statements regarding education provisions is CORRECT? A) Qualifying expenses for the American Opportunity Tax Credit (AOTC) include tuition, books, supplies, and room and board. B) Higher-education expenses for the Section 529 plan may include expenses for a computer, software, and internet access. C) The annual contributions to a Coverdell account may not exceed $2,000 per donor. D) The American Opportunity Tax Credit applies to undergraduate and graduate courses at an accredited, Title IV institution.

Qualifying expenses for the AOTC do not include room and board. The AOTC may only be used for the first four years of undergraduate or vocational courses. Qualified higher-education expenses for Section 529 purposes include tuition, books, fees, and equipment for enrollment at an eligible educational institution; expenses for special needs services; and room and board costs for students who are at least half-time. In addition, expenses for the purchase of computer or peripheral equipment (printer, modem, etc.), computer software, or internet access and related services may be treated as qualifying expenses if the equipment, software, or services are to be used primarily by the beneficiary during any of the years the beneficiary is enrolled at an eligible educational institution. The annual contributions to a Coverdell account may not exceed $2,000 per beneficiary.

Which of these is CORRECT regarding the Lifetime Learning credit? A) The credit only applies during the first four years of postsecondary school. B) There is a phaseout between $160,000 and $180,000 of AGI for married taxpayers filing jointly. C) The credit is equal to 100% of the first $2,000 and 25% of the next $2,000 of qualifying higher education expenses. D) Qualifying expenses include tuition, books, supplies, and room and board.

Qualifying expenses generally include only tuition. Amounts paid for books and supplies may be included only if required to be paid to the educational institution as a condition of enrollment. The credit is available annually for an unlimited number of years. The credit is equal to 20% of qualified tuition expenses up to $10,000. All of the listed options, except for the phaseout limits, accurately describe the American Opportunity tax credit.

Which one of the following is an application of the administrative powers of the Internal Revenue Service and not of the powers of Congress? 1) Regulations 2) Revenue Rulings A) I only B) Neither I nor II C) II only D) Both I and II

Regulations are a direct extension of the lawmaking powers of Congress, whereas revenue rulings are an application of the administrative powers of the Internal Revenue Service.

Which one of the following is allowable in the computation of total income? A) Charitable contributions B) Net capital losses of up to $5,000 C) Loss from a sole proprietorship D) Tax credits

Remember that the total income is the amount shown on the front of the Form 1040. It is the amount before the deduction for adjustments to income. Certain deductions are allowed in the computation of total income, such as the deduction for sole proprietorship losses or net capital losses up to $3,000. Charitable contributions are an itemized deduction.

Max is a widower who provides a home for himself and his dependent six-year-old daughter, Lucy. He has hired an individual to pick his daughter up from school each day, bring her home, cook dinner, and perform some housekeeping services until he gets home four hours later. He pays $1,600 per month for the service. How will this affect Max's income tax return? Max may be entitled to a child and dependent care credit. Max qualifies to list Lucy on his income tax return as a dependent. Because Lucy attends school during the day, the child or dependent care credit is not available. Max must allocate the $1,600 per month between child care and housekeeping services. A) I and II B) II, III, and IV C) I and IV D) II only

School attendance does not affect the availability of the credit. The $1,600 in expenses incurred each month to enable Max to work outside the home do not have to be divided between child care and housekeeping services.

Which of the following statements is CORRECT? A) A shareholder who receives a Schedule K-1 from an S corporation must calculate and pay self-employment tax on the income. B) A sole proprietor must pay the 0.9% Additional Medicare Tax on all net self-employment income. C) Self-employed individuals, such as a sole proprietor or general partner in a partnership, generate self-employment income and must pay both portions of the FICA (Federal Insurance Contributions Act) payroll tax. D) A general partner reports the income from the Schedule K-1 form provided by the partnership on Schedule C of Form 1040.

Self-employed individuals, such as a sole proprietor or general partner in a partnership, generate self-employment income. In turn, such individuals must pay both portions of the FICA (Federal Insurance Contributions Act) payroll tax. A general partner reports the income from the Schedule K-1 form provided by the partnership on Schedule E of Form 1040. A shareholder who receives a Schedule K-1 from an S corporation does not pay self-employment tax on the income. The Additional Medicare Tax of 0.9% also applies to self-employed individuals who have a combined income greater than $200,000 if single and $250,000 if MFJ. The tax is levied on the net earnings from self-employment of the sole proprietor or partner and consists of: -- the gross income derived from any trade or business, less allowable deductions attributable to this trade or business (generally Schedule C); or -- the taxpayer's distributive share of the ordinary income or loss of a partnership (not an S corporation) engaged in a trade or business (Schedule K-1).

Peter, a CPA, has a standard package of services for which he normally charges $600. Peter donated this package to a charitable auction. The package contains three hours of Peter's services for which the CPA normally charges $200 per hour. How much, if any, can Peter take as a charitable contribution deduction? A) $200 B) $400 C) $0 D) $600

Services donated to charity are not deductible. Therefore, Peter cannot deduct any amount under the charitable contribution income tax rules.

Clare is a single taxpayer. In 2022, her AGI is $235,000, including a net long-term capital gain of $50,000. What is the amount, if any, of Medicare contribution tax that she must pay? A) $1,900 B) $0 C) $570 D) $1,330

She will pay the 3.8% Medicare contribution tax on $35,000. This is the lesser of the net investment income ($50,000) or the AGI in excess of the threshold amount ($235,000 - $200,000, or $35,000). In this situation, only $35,000 of the net investment income is subject to the Medicare contribution tax. Clare will pay a $1,330 Medicare contribution tax (3.8% on $35,000).

A taxpayer invested in a real estate limited partnership several years ago. There is a special allocation in effect in the partnership. He is concerned about the deductibility of the losses that are flowing from the partnership. Which of the following rules or doctrines may limit the availability of income tax benefits from his limited partnership investment? 1) Direct participation program 2) Special allocation rules 3) At-risk rule 4) Passive activity loss rule A) I and IV B) II, III, and IV C) II and III D) I and III

Special allocation rules, such as the substantial economic effect doctrine, limit the ability to utilize special allocations in a partnership. The at-risk rule limits the ability to utilize leverage by attacking the use of nonrecourse financing. The passive activity loss rule limits the ability to deduct losses from activities in which the taxpayer does not materially participate. The direct participation program simply refers to a tax conduit.

All of the following statements regarding income tax filing status are CORRECT except A) unmarried people who maintain a household for a qualifying child or relative may be eligible for head of household status. B) if a spouse dies during the tax year, the surviving spouse may use married filing jointly status for that year. C) spouses may not file a joint return if one spouse has no income or deductions. D) it is usually advantageous for married couples to file a joint return.

Spouses may file a joint return even if one spouse has no income or deductions.

Max is selling a truck that he uses in his business. He has taken $5,000 of depreciation on the truck and wants to use the installment sale method to sell the truck to Jerry for a down payment and an installment note over 36 months. He paid $40,000 for the truck and is selling it for $38,000. What are the tax consequences of this transaction? 1) Max must recapture $3,000 of the Section 1245 depreciation taken as ordinary income in the year of the sale. 2) Max has $5,000 of depreciation recapture taxed at the 25% tax rate. A) II only B) Neither I nor II C) Both I and II D) I only

Statement I is correct. Gain recaptured under Section 1245 (depreciable personal property used in a trade or business) is taxed as ordinary income and is not eligible for installment sale treatment. Therefore, these amounts are fully recognized (taxable) as ordinary income in the year of sale. Unrecaptured Section 1250 depreciation occurs only on depreciable real property (real estate) used in a trade or business.

Now that their parents have died, Joseph is assuming the responsibility for the care of his 25-year-old disabled sister. Joseph has told his financial planner there is a trust for his sister to provide funds sufficient for her to live in an assisted-living facility or to reimburse him for the cost of an at home caregiver and his sister's other expenses if she stays in his home. He would prefer for his sister to live with him but does not want to cause her any problems with tax issues on any income she gets if she does. His sister is adamant that she does not want to be anyone's dependent. What advice can the planner give to Joseph? If his sister's income from the trust is paying for all of her needs, then she is not a dependent on anyone's income tax return. If Joseph's sister lives in an assisted-living facility, she cannot be a dependent of Joseph. A) I only B) Both I and II C) II only D) Neither I nor II

Statement I is correct. If all of his sister's support is paid for by funds she receives from the trust, she cannot be a dependent of Joseph. Statement II is incorrect. Simply not living with Joseph does not preclude him from listing his sister as a dependent. If Joseph provided more than half of the support for his sister, he would be allowed to list her as a dependent and take any tax credits that may be available.

Jack bought publicly traded stock seven years ago for $6,000. Its current value on the securities market is $11,000. He has donated this appreciated stock to a charity that provides housing for the homeless. What must Jack do to take the donation as a charitable deduction? 1. Jack must have documentation from the charity substantiating the amount of the donation, the date donated, and the name of the charity. 2. All donations of stock must have a qualified appraisal of the stock attached to the donor's income tax return. A) Both I and II B) Neither I nor II C) II only D) I only

Statement I is correct. In additional, the taxpayer must be in receipt of this documentation by the due date of the return or when the return is filed. Statement II is incorrect because it reads "all" donations of stock must have a qualified appraisal. A qualified appraisal is not required for closely held stock if the amount donated is less than $10,000. The appraisal itself is not attached to the tax return.

Which of the following statements regarding passive activity losses is CORRECT? 1) When determining the amount of suspended loss that may be used against income, the at-risk rules are applied before the passive activity loss rules. 2) If a loss is not allowed because of the at-risk limitations, the loss is a suspended loss eligible for deduction as a disposition of a passive activity. A) II only B) Both I and II C) I only D) Neither I nor II

Statement I is correct. Statement II is incorrect. If a loss is not allowed because of the at-risk limitations, the loss is a suspended loss and is not eligible for deduction as a disposition of a passive activity.

As part of the property settlement after Lori and Gordon divorced, Gordon transferred ownership of a life insurance policy to Lori. Lori is the beneficiary of the policy, and Gordon is the insured. Which of the following statements regarding the property settlement is CORRECT? The transfer of the life insurance policy is subject to the transfer-for-value rule. The death proceeds of the policy will be income tax free at Gordon's death. A) Both I and II B) I only C) II only D) Neither I nor II

Statement I is incorrect; the transfer-for-value rule does not apply when a life insurance policy is transferred from one spouse to the other under a property settlement incident to divorce. Statement II is correct.

Which of the following statements regarding the disposition of passive activities with suspended losses is NOT correct? 1) After using losses to offset gain on the sale of the activity, any remaining losses are still passive losses. 2) Suspended losses from the activity are first offset against any gain on the sale of the activity. A) Both I and II B) Neither I nor II C) II only D) I only

Statement II is a correct statement. Statement I is incorrect. After using losses to offset gain on the sale of the activity, any remaining losses are classified as nonpassive losses.

Which of the following statements regarding the alternative minimum tax is CORRECT? 1). If the regular income tax after credits equals or exceeds the individual AMT, then no individual AMT payment is required. 2) If the regular income tax after credits is less than the individual AMT, then the AMT is not due. A) II only B) I only C) Neither I nor II D) Both I and II

Statement II is incorrect. If the regular income tax after credits is less than the individual AMT, then the AMT must be paid with this difference, referred to as the AMT payable.

Brandon failed to file a federal income tax return for last year's tax liability by April 15 of the current year. He filed his tax return October 30 of the current year and remitted the tax that was due. Which of the following statements is CORRECT? 1) Brandon will owe interest to the IRS on the unpaid tax liability that runs from April 15 of the current year until the tax liability is paid in full. 2) Because he took so long to file, the 90%/100% payment criteria does not apply. 3) Brandon will be assessed the failure-to-file penalty. 4) Depending on the facts and circumstances surrounding the return filed, Brandon could be assessed a negligence penalty. A) III and IV B) I only C) I, II, III, and IV D) I, III, and IV

Statement II is incorrect. The 90%/100% underpayment criteria will still apply even though Brandon did not file in a timely manner. All of the other statements are correct.

Robin and Marian, who are both age 33, are trying to calculate their taxable income. Which items from the IRS Form 1040 are deducted from the couple's adjusted gross income (AGI) to arrive at their taxable income? 1.) 100% of self-employment taxes paid 2.) Ordinary and necessary business expenses 3.) Student loan interest 4.) Medical expenses in excess of 7.5% of AGI A) IV only B) II only C) I, II, III, and IV D) I and III

Statement IV is correct. The medical expenses are part of the itemized deductions. Only the greater of standard or itemized deduction are deductible from AGI to arrive at taxable income. One-half of the self-employment tax and student loan interest are items deducted from gross income to arrive at adjusted gross income. The ordinary and necessary business expenses (Schedule C expenses) are deducted in arriving at total income.

Gary has just divorced. He is asking his planner, Ruth, for recommendations of amending his financial plan given his newly single status. Which of the following recommendations should Ruth make? Gary should review the beneficiary designations on his life insurance policies to be certain the beneficiaries are in line with his wishes post-divorce. Ruth should ask for any documentation on property settlements and other court-ordered financial transactions. Ruth should inquire whether a qualified domestic relations order was issued by the court and obtain a copy to ascertain its effect on Gary's financial plan. Ruth should inform Gary that the transfer-for-value income tax rule makes the transfer of any life insurance policies a taxable event. A) III only B) I, II, and III C) II and IV D) I and IV

Statement IV is incorrect. The transfer-for-value income tax rule does not apply in situations when a life insurance policy is transferred from one spouse to another as a result of a property settlement.

Five years ago, Tom bought 10,000 shares at $10 per share in an intermediate-term bond fund. Today, the shares are worth $200,000 and are paying a nonqualified dividend of $8,000 per year. Tom feels that the stock will continue to appreciate at a rate of 5% per year, including the dividend. Tom wants to establish a college education fund for his two daughters, ages 18 and 9. Neither child has any earned income. Which of the following statements is true? 1. If Tom gives 2,500 shares to his 18-year-old daughter, all income from the 2,500 shares will be taxed in her income tax bracket. 2. If Tom gives 2,500 shares to his 9-year-old daughter, all dividends from the 2,500 shares will be taxed at her marginal rate. 3. Two years from now, if Tom's older daughter sells her 2,500 shares at $30 per share, Tom will need to report the gain as a long-term capital gain on his personal income tax return. 4. All interest income received by his 9-year-old daughter that exceeds $2,200 in 2021 will be taxed at the parents' marginal tax rate. A) II and III B) I and II C) I and IV D) IV only

Statement IV is the only correct statement. The kiddie tax applies to children under 19 years of age. It also applies to children under age 24 if they are full-time students. The kiddie tax does not apply if the child's earned income exceeds one-half of the child's support. Thus, I and II are incorrect. There is no requirement that the proceeds of a future sale be reported on the donor's return.

Which of the following are allowable itemized deductions for purposes of computing the alternative minimum tax? 1. Charitable deductions 2. Qualified housing interest 3. Gambling losses to the extent of gambling winnings 4. Property taxes A) I and III B) II, III, and IV C) I and II D) I, II, and III

Statement IV, property taxes, is the only itemized deduction listed that is not allowed for AMT purposes. State and local income taxes are also disallowed.

Taxpayers owning certain resources may recover the cost of exhausting that natural resource (cost recovery) using which of the following methods? 1) Cost depletion 2) Percentage depletion 3) Amortization over 15 years A) II only B) I and II C) II and III D) I and III

Statements I and II are correct. Statement III is incorrect. Amortization is used for Section 197 intangible assets.

Which of these statements regarding the kiddie tax is NOT correct? 1.) The standard deduction for a child with both earned and unearned income is always earned income plus $400. 2.) The child's tax rate is 10% for all income received. 3.) The excess of unearned income above $2,300 is tax to the child at the parent's marginal tax rate. 4.) The kiddie tax provision limits income shifting, wherein families are prevented from transferring large amounts of unearned income to children and making the shift effective for income tax purposes. A) II only B) III and IV C) I and II D) I, II, and III

Statements I and II are incorrect. The standard deduction for a child with both earned and unearned income is the greater of $1,150, or earned income plus $400 but limited to the standard deduction for a single taxpayer, $12,950 in 2022. The child's tax rate is determined by the amount of taxable income the child has. Statements III and IV are correct.

Which of the following taxpayers may owe the additional Medicare tax of .9% in 2022? 1) Brad and Jane file jointly and have combined wages of $288,000. 2) Terry's only income in 2022 is from his investments and totals $290,000. 3) Jack has earned $150,000 in compensation from his employment at Bland Foods Inc. 4) Lisa, whose filing status is head of household, is self-employed and has self-employment income of $225,000. A) I, II, and III B) I only C) IV only D) I and IV

Statements I and IV are correct. The additional Medicare tax rate is .9%. An individual is liable for the additional Medicare tax if the individual taxpayer's wages, other compensation, or self-employment income (combined with a spouse if filing as married filing jointly) exceeds the thresholds for the taxpayer's filing status of a combined income greater than $200,000 if single/head-of-household or $250,000 if married filing jointly. Choice II is incorrect. Because Terry has no wage income, the Additional Medicare tax of .9% cannot apply. However, Terry is subject to the separate Net Investment Income tax of 3.8%.

Your clients, John and Mary Voight, spoke recently to their insurance agent regarding the purchase of a single premium whole life policy. The agent indicated that the policy would be a modified endowment contract. The Voights were unsure what that meant. Which of the following describe a modified endowment contract? 1) Meets the requirements of a life insurance contract under state law 2) Was entered into (or substantially modified) on or after June 21, 1988 3) Fails to meet the "seven pay test" 4) Meets the guideline premium and corridor test or the cash value accumulation test A) I, II, and III B) II, III, and IV C) I, II, III, and IV D) II and III

Statements I through IV are all elements of the IRC definition of a modified endowment contract. The contract fails to meet the seven-pay test if the cumulative amount paid under the contract at any time in the first seven years is greater than the seven net level annual premiums that would have been paid under a seven-pay, paid-up contract.

Which of the following adjustment/preference items is also an exclusion item for the purposes of the alternative minimum tax (AMT)? 1) ISO bargain element 2) Exclusion of gain from Section 1202 qualified small business stock 3) Percentage depletion of oil and gas properties in excess of the taxpayer's adjusted basis at year-end A) III only B) II and III C) II only D) I and III

Statements II and III are correct. Statement I is incorrect. The ISO bargain element is not an exclusion item; it is a deferral item.

Which one of the following is NOT a main source of federal tax revenue? A) Private company taxation B) Payroll taxation C) Individual income taxation D) Corporate income taxation

The three main sources of federal tax revenue are individual income taxes, corporate income taxes, and payroll taxes; private company taxation is not a main source of federal tax revenue.

Which of the following activities are considered passive activities? 1) A real estate broker spending 1,500 hours on her real estate activities, this being 80% of her personal services for the year 2) A taxpayer who owns an apartment building using a property management to assist him in managing the property 3) A taxpayer investing in a real estate limited partnership (RELP) 4) A taxpayer owning 8% of an inherited condo that is rented to others during the year A) I, II, III, and IV B) II, III, and IV C) II and III D) I only

Statements II, III, and IV are correct. Only the real estate professional's activities are not considered passive. In statement IV, the taxpayer owns less than 10% of the activity, so it is considered passive. This taxpayer is barred from using the real estate exception for losses because the taxpayer owns less than 10% of the activity.

Which of the following statements is CORRECT? 1) Targeted program audits make up approximately 25% of each year's total of returns audited. 2) The general statute of limitations for audits is three years from the filing date of the return, but six years if 25% of gross income is unreported. A) II only B) Both I and II C) I only D) Neither I nor II

Targeted program audits make up approximately 25% of each year's total of returns audited. The general statute of limitations for audits is three years from the filing date of the return (or due date, if later), but six years if 25% of gross income is unreported.

Which one of the following steps occurs in the tax calculation process? A) Total tax liability minus itemized deductions plus additional taxes owed, equals total tax liability B) Total withholding is adjusted on Form I-9 C) Total tax liability equals refund or tax owed D) Tax liability minus tax credits equals refund or tax owed

Tax liability minus tax credits equals total tax liability or refund.

Which of the following tax preference items are used in calculating the alternative minimum tax (AMT) for an individual? 1. Tax-exempt income from a State of Louisiana general obligation municipal bond 2. Percentage depletion in excess of basis on a mining property 3. Tax-exempt interest on a private-activity bond issued in 2012 4. Exclusion of gain on the sale of certain qualified small business corporation stock A) I, II, and III B) I only C) II, III, and IV D) I, II, III, and IV

Tax-exempt income from a general obligation municipal bond is not a preference for AMT. All of the other items are tax preference items for AMT purposes.

Marvin has all of the following items. All of them are AMT preference items except A) tax-exempt income from a State of Iowa municipal revenue bond. B) tax-exempt interest on certain private-activity bonds. C) percentage depletion in excess of adjusted basis on a mining property. D) exclusion of gain on the sale of certain qualified small business corporation stock.

Tax-exempt income from a municipal revenue bond is not a preference for AMT. All of the other items are preferences for AMT purposes.

Investments that were intended to defer or eliminate taxes for the investor and, possibly, generate tax losses in excess of the taxpayer's basis are called A) at-risk investments. B) tax shelter investments. C) deductible investments. D) passive activity investments.

Tax-shelter investments were intended to defer or eliminate taxes for the investor and, sometimes, generate tax losses considerably in excess of taxpayer's basis. Before Congress restricted the practice, paper losses in excess of the amount of capital invested in an activity (particularly by limited partner investors) allowed a considerable benefit to taxpayers who were otherwise not liable if the investment failed.

Which one of the following is NOT a main source of federal tax revenue? A) Corporate income taxation B) Taxation of taxpayers living abroad C) Individual income taxation D) Payroll taxation

The three main sources of federal tax revenue are individual income taxes, corporate income taxes, and payroll taxes; taxation of taxpayers living abroad is not a main source of federal tax revenue.

Assume that married taxpayers filing jointly have a taxable income of $200,000. Using the tax rate schedule provided in your course references, what is the amount of federal income tax? Round your answer to the nearest dollar. A) $47,367 B) $35,671 C) $42,750 D) $37,466

Taxable income$200,000Less (from tax rate schedule)(178,150)Amount over $178,150$21,850Times (marginal rate, from tax rate schedule)24%Tax on amount over $178,150$5,244Plus (from tax rate schedule)$30,427Total Tax$35,671 If you arrived at another answer, you likely did the calculation correctly but may have used the single filing status rather than the married filing joint return status or used a flat rate of 24%. While 24% is the marginal rate (the rate paid on the last dollar of income), there are dollars that are taxed at the 10%, 12%, and 22% rates as well.

Caroline has the following items of income to report in 2022: Salary of $65,000 from an S corporation for which Caroline is a 10% shareholder Her share of the net income from the same S corporation of $95,000 as reported on Schedule K-1 Schedule C net income of $15,000 from a small side business she owns How much of her income in 2022 will be subject to self-employment taxes? A) $17,000 B) $95,000 C) $110,000 D) $15,000

Taxes are withheld from Caroline's salary at the S corporation in the same manner as any other employee of the S corporation. Income reported on the Schedule K-1 to shareholders of an S corporation is not considered self-employment income. Only the income from the small side business reported on Schedule C is subject to self-employment taxes.

Which one of the following is advice from the National Office of the IRS requests that can give both the taxpayer and a revenue agent an opportunity to resolve a dispute? A) Notice B) Announcement C) Technical Advice Memoranda D) Private Letter Rulings

Technical Advice Memoranda normally take place during an audit or during the appeals process of the audit; they give both the taxpayer and the revenue agent an opportunity to resolve a dispute over a technical question. Private Letter Rulings are taxpayer guidance from the IRS that apply only to the particular taxpayer(s) asking for the ruling; they are not applicable to all taxpayers. A Notice is a public pronouncement that contains official guidance about regulations or interpretations of the Code. Announcements often summarize code sections in layman's terms, or notify taxpayers of impending deadlines; they have only short-term value.

Terry and Jan are married taxpayers filing a joint tax return. In 2022, their AGI is $310,000, and their net investment income (included in the AGI) is $90,000. What is the amount of their Medicare contribution tax for 2022? A) $3,420 B) $2,280 C) $4,180 D) $0

Terry and Jan will pay the 3.8% Medicare contribution tax on $60,000. This is the lesser of the net investment income ($90,000) or the AGI in excess of the threshold amount ($310,000 - $250,000, or $60,000). In this situation, only $60,000 of the net investment income is subject to the Medicare contribution tax and calculates to $2,280 ($60,000 × 0.038).

During the current tax year, Jamie has a $13,000 short-term capital loss and a $14,000 long-term capital gain, both from the sale of securities. Jamie also has a $10,000 long-term capital gain from the sale of collectibles. Jamie, a single taxpayer, is in the 32% marginal income tax bracket. Which of these accurately describes the result of these transactions? A) $14,000 long-term capital gain and a $3,000 net capital loss carryforward B) $1,000 long-term capital gain, taxed at 15%, and $10,000 collectibles gain, taxed at 28% C) $11,000 long-term capital gain, taxed at 20% D) $11,000 long-term capital gain taxed at 15%

The $13,000 short-term capital loss is first used against the collectibles gain—the gain that would be taxed at the highest rate (28%). This eliminates the collectibles gain. The remaining $3,000 short-term capital loss is then used against the $14,000 long-term capital gain from the sale of securities. This leaves $11,000 of long-term capital gain, taxed at 15%. We know the long-term capital gain is taxed at 15%, as the top of the 32% MITB is $219,950 (2022) for a single taxpayer, and the 20% LTCG rate doesn't kick in until $459,750 for a single taxpayer.

Jeff Munroe has an annual salary of $140,000 and is not an active participant in a company-maintained retirement plan. He had the following financial transactions during the current tax year: Received a $100,000 cash inheritance due to the death of his brother Received unemployment compensation of $2,000 Had a Schedule C loss of $10,000 (assume material participation) Made an IRA contribution of $6,000 Paid qualified student loan interest of $2,000 What is Jeff's total income for the current tax year? A) $132,000 B) $126,500 C) $124,500 D) $142,000

The $140,000 salary is reduced by the $10,000 self-employment loss and increased by the unemployment compensation of $2,000. The inheritance is excluded. The IRA contribution is a potential adjustment to income, as is the student loan interest. Thus, those items do not affect the total income. Remember that total income is the e figure from which allowable adjustments to income are subtracted.

Brent and Sheila are married and will file a joint return. They have provided you with the following information. Brent's salary$80,000Sheila's salary$65,650Alimony payments to Brent's ex-wife$20,000Brent's child support paid$12,000Itemized deductions$13,000 Brent's divorce decree from his previous marriage was finalized in 2016. Based on the information given, what is the couple's taxable income for the 2022 tax year? A) $88,550 B) $99,750 C) $111,550 D) $111,750

The $145,650 in salaries is reduced by the alimony payment of $20,000 to give an AGI of $125,650. The AGI is reduced by the greater of the itemized deductions ($13,000) or the standard deduction ($25,900 in 2022), to equal $99,750. The child support paid is not a deductible item.

Troy and Myrna are married and file a joint income tax return. They have two dependent children who attend day care so that Troy and Myrna can work outside the home. Their adjusted gross income was $140,000 last year. On last year's tax return, the couple claimed a $960 credit for child care expenses. They are in the 22% marginal income tax bracket. What amount of deductions would be required to equal the tax benefit of the $960 child care credit? A) $1,230 B) $749 C) $211 D) $4,364

The $960 credit is divided by the marginal income tax rate of 22% to equal $4,364. The other way to look at this is that a $4,364 deduction in a 22% tax bracket will save the taxpayer $960.

Marco had an individual AMT credit two years ago and has not yet used it. Which of the following is correct regarding Marco's AMT credit? A) The credit can be carried forward indefinitely to be applied against a future alternative minimum tax liability. B) The credit must be applied against the AMT liability in the next year there is AMT to be paid. C) The credit must be applied against the regular income tax liability in the year the credit is created. D) The credit can be carried forward indefinitely to be applied against a regular income tax liability.

The AMT credit created in any one year may be used as a credit against regular income tax in a future year. It may be carried forward indefinitely.

When does the alternative minimum tax (AMT) apply to a taxpayer? A) When the AMT calculation results in a tax liability that is greater than that resulting from the regular income tax calculation B) When the taxpayer has losses from passive activities or other business functions C) Anytime a person is involved in passive activities D) When the taxpayer has not withheld or prepaid a sufficient proportion of actual income tax liability

The AMT is paid when the AMT tax calculation produces a higher amount of tax than the regular tax calculation and the AMT payable is the difference between the two.

Eight years ago, Joan Allen, a married taxpayer filing jointly, purchased U.S. Series EE savings bonds for $6,000. She titled the bonds jointly with her husband, Hank. During the current year, when Joan was 35 years old, they redeemed the bonds to help pay for Joan's graduate school tuition. The accrued value at the time of redemption was $8,000. Their AGI for 2022 is estimated to be $100,000. Assume Joan incurs $8,000 of tuition expenses during the year. What are the tax consequences upon the redemption of the bonds? A) The interest is taxable at both state and federal levels. B) All accrued interest is taxable in the current year. C) A portion of the interest may be excluded. D) All the interest may be excluded.

The EE bond exclusion (for educational purposes) is phased out (for married couples filing jointly) between $128,650 and $158,600 of AGI in 2022. There is no exclusion available when AGI exceeds $158,650. It is not necessary to memorize the exact phaseout amounts because they will be provided on the exam. To qualify for the exclusion, the bonds must be purchased by an individual age 24 or older and held in that person's name, or jointly with a spouse. EE bonds are not taxable at the state level.

If an individual taxpayer files an Extension Form 4868 on a timely basis in 2022, the 2021 federal income tax return is due (ignoring weekends or holidays), at the latest, by A) October 15, 2022. B) April 15, 2022. C) July 15, 2022. D) August 15, 2022

The Form 4868 provides an automatic six-month extension of time to file the Form 1040. The return would be due April 15 without regard to extensions. With the six-month extension, the due date is October 15.

Which of the following statements regarding the alternative minimum tax (AMT) or AMT planning are CORRECT? 1. The AMT reduces the tax benefits from certain types of deductions and tax preferences allowable for regular income tax purposes. 2. The starting point for determining alternative minimum taxable income (AMTI) is AGI as reported for regular income tax purposes. 3. It is generally advantageous to defer the payment of real estate taxes to a future year when AMT will be paid in the current year. 4. It is generally advantageous to accelerate ordinary income into years when AMT will be paid. A) I, III, and IV B) III and IV C) I, II, and IV D) I and III

The IRS notes that the starting point for determining AMTI is taxable income as reported for regular income tax purposes on a taxpayer's IRS Form 1040. Because real estate taxes are not deductible for AMT purposes, it is generally advantageous to defer the payment of such taxes to a year when AMT will probably not be payable. Also, if AMT will be payable in the current year, it is generally advantageous to increase the amount of regular taxable income (e.g., by accelerating ordinary income into the current year) because the total tax payable will likely not be affected by doing so.

Which of the following correctly describes the Targeted Program audit process? A) Voluntary compliance program designed for selected transactions or occupations B) Computes the norms and averages that are used in the Discriminant Functions System Program C) Screens individual tax returns and ranks them in order of auditworthiness D) Matches information documents (e.g., W-2 or 1099 forms) to individual taxpayers' income tax returns

The IRS uses a targeted programs approach for approximately 25% of all returns audited. Targeted Program audits are not so much a revenue-raising device as they are a voluntary compliance device. When a particular problem area is singled out, the word quickly spreads, and voluntary compliance is greatly increased.

Which one of the following types of audits is conducted on a random basis? A) Document Matching Program audit B) Targeted Program audit C) Discriminant Functions System Program audit D) National Research Program audit

The National Research Program audit is the only random audit that the IRS conducts. The NRP is, in effect, the replacement for the old Taxpayer Compliance Measurement Program audit. The Discriminant Functions System Program (DIF) compares the information on each return to a set of norms, weighs each item, and ranks the return for audit-worthiness. The Document Matching Program allows the IRS to detect discrepancies between the amounts reported on a tax return and the amounts shown on information documents (e.g., W-2 forms, 1099 forms). Under the Targeted Program Audit, the IRS "targets" particular taxpayers in particular situations.

Glen and Debbie both have significant net worth and are currently in the highest marginal income tax bracket. They have developed a process that allows them to neutralize toxic chemical waste. They want to form a business that will protect their net worth in case the business fails or it becomes involved in lawsuits. They expect the business to produce significant profits immediately, as they have agreements in the works with several large chemical companies. Also, they would like to share ownership with other family members as they get closer to retirement. Which business form would be most appropriate for Glen and Debbie at this time? A) Limited partnership B) General partnership C) S corporation D) C corporation

The S corporation (and the partnership entities) are conduits, and would cause the income to be added to their other significant individual income. The use of the C corporation would allow for protection from lawsuits or business failure. Also, the general partnership and the limited partnership both would cause a flow-through of the income, and the general partnership would not provide protection from personal liability. There is no indication that either party wants to be a general partner in a limited partnership. The use of a C corporation would be appropriate since it is a separate taxable entity, and the profits would not flow through. They could receive a reasonable salary, and the qualified dividends that are paid out would be subject to long-term capital gain rates.

The effective tax rate is obtained by A) finding the tax bracket of total income. B) finding the tax bracket of the taxable income amount. C) dividing the calculated tax by taxable income. D) dividing taxable income by the marginal tax rate.

The effective tax rate is calculated by DIVIDING THE CALCULATED TAX BY TAXABLE INCOME. The marginal tax rate is found by finding the tax bracket that contains the taxable income amount; it is the amount at which all subsequent taxable amounts will be taxed (until entering the next tax bracket).

Which these statements is CORRECT regarding the Section 121 exclusion? A) The exclusion may not be used if the residence was acquired in a like-kind exchange within the last five years. B) A taxpayer must be age 55 or older to qualify for the exclusion. C) The maximum exclusion for a married couple filing jointly is $250,000. D) In order to qualify for a full exclusion, the exclusion may not be used more than once in a three-year period.

The Section 121 exclusion may not be used if the residence was acquired in a like-kind exchange within the last five years. The maximum exclusion for a married couple filing jointly is $500,000. There is no age limit for the Section 121 exclusion. The exclusion may be used every two years.

During 2022, Gabe purchased equipment for his manufacturing business (a C corporation) for $2.81 million. His business will have taxable income of $990,000 (without regard to the Section 179 expense) for the year. What is the maximum Section 179 expense election for Gabe in the current year? A) $1.08 million B) $990,000 C) $970,000 D) $0

The Section 179 expense election is limited based on the dollar amount of qualifying property placed in service during the tax year. To the extent that more than $2.7 million (for 2022) of qualifying property is placed into service during the tax year, the benefit of the Section 179 election is reduced on a dollar-for-dollar basis. In this situation, there was $110,000 extra placed into service, so the $1.08 million maximum Section 179 election is reduced by the $110,000, leaving a $970,000 Section 179 election. In this situation, there is sufficient taxable (earned) income to allow this deduction.

During 2022, your client, Bob, purchased several items of equipment with a total cost of $265,000 for use in his sole proprietorship. Bob has taxable (earned) income from his Schedule C business of $112,000 (without regard to the Section 179 expense). He also has wages from a part-time job of $10,000. What is the maximum amount of Section 179 expense that Bob may deduct in the current year? A) $265,000 B) $1,000,000 C) $122,000 D) $112,000

The Section 179 expense election is limited to the taxable (earned) income of the taxpayer. For purposes of Section 179, salary or wages received as an employee, even from a completely unrelated source, are also considered to be from the active conduct of the trade or business. Thus, the total taxable (earned) income in this situation is $122,000. The maximum Section 179 expense election is $1.08 million (for 2022), but for Bob, it is limited to his earned or taxable income of $122,000.

During 2022, Stanton purchased several items of depreciable, tangible personalty—with a total cost of $2.85 million—for use in his business. Stanton has taxable (earned) income from his sole proprietorship of $360,000 (without regard to the Section 179 expense). He also has wages from a part-time job of $55,000. What is the maximum Section 179 expense amount that Stanton may deduct in the current year? A) $930,000 B) $415,000 C) $360,000 D) $1,080,000

The Section 179 expense is first limited by the amount of qualifying property placed in service during the tax year. To the extent that the taxpayer purchases and places into service more than $2.7 million (2022) of qualifying property during the tax year, the benefit of the Section 179 election is reduced on a dollar-for-dollar basis. In this situation, Stanton placed an excess $150,000 of property into service. This reduces the maximum election from $1.08 million to $930,000—this is the maximum Section 179 expense of $1.08 million reduced by the dollar amount of property placed into service in excess of $2.7 million. The Section 179 expense election is then limited to the taxpayer's taxable, or earned, income. Because the business is a sole proprietorship, for purposes of Section 179, salary or wages received as an employee are considered to be from the active conduct of a trade or business. Thus, the total earned income in this situation is $415,000. The maximum Section 179 expense election is $930,000, but for Stanton, the current year deduction is limited to his earned income of $415,000.

Jena owns and operates a string of retail electronics stores with approximately $30 million of sales annually. Approximately 20% of her sales are with extended credit terms. What method of tax accounting is most appropriate for Jena's business? A) The hybrid method, because the business involves both inventory and service B) The installment sale method, to spread the gain over a longer time frame C) The cash method, because it provides flexibility in the timing of income and expenses D) The accrual method, because inventory is such a large component of the business

The accrual method of accounting generally is mandatory when inventory constitutes a significant income-producing factor. Thus, the cash method is incorrect. The hybrid method is incorrect because there is no indication that service constitutes a significant portion of the business. Also, the installment sale method is not available for sales of inventory, or sales with revolving credit terms. With annual sales of $30 million, the accrual method exception does not apply. If she had annual sales of under $27 million (2022), she could still use the cash method, even though she has inventory.

Kappa Corporation has the following items of income and expense: Taxable income: $310,000 Federal income tax paid: $80,000 Dividends paid: $20,000 Accumulated earnings and profits at the end of the preceding year: $90,000 Kappa is an engineering firm with 100% of the stock owned by its three employee-shareholders. The corporation cannot establish a valid business purpose for excess accumulations. How much accumulated earnings tax is payable by Kappa? A) $46,000 B) $34,000 C) $42,000 D) $30,000

The accumulated earnings tax applies to corporate accumulated earnings in excess of $250,000, generally, for which there is no valid business reason for accumulating the funds. For a personal service corporation (PSC), the accumulations limit is $150,000. Engineering is one of the personal services specifically listed for determining a PSC. Expenses otherwise not allowed in computing the corporate income tax are allowed in computing the accumulated earnings tax, as is summarized here. (Note that the accumulated earnings credit of $60,000 is the difference between the $150,000 safe harbor and the $90,000 of accumulations at the beginning of the year.) The accumulated earnings tax rate is 20%, the highest rate that may apply to qualified dividends.

Cindy has adjusted gross income (AGI) of $350,000. Included in the AGI is passive income of $40,000 and passive losses of $55,000, $40,000 of which she uses to offset the passive income and $15,000 of which is subject to carryforward. Which one of the following activities has the greatest potential for reducing Cindy's tax liability? A) None of these options will reduce Cindy's tax liability B) Investing in an oil and gas limited partnership that is generating losses C) Investing in "active participation" rental real estate that is producing a loss D) Investing in a real estate partnership in which she will not materially participate that is producing passive losses

The active participation deduction is eliminated at $150,000 of AGI. The oil and gas limited partnership and the equipment-leasing limited partnership would produce more passive losses that are nondeductible. Therefore, none of the options are viable.

Gary received a bequest of 100 shares of XYZ stock from a relative who died on March 1, 2022. The relative bought the stock at a total cost of $5,500. The value of the 100 shares of XYZ stock was $5,750 on March 1. Its value rose to $6,250 on July 1, 2022, on which day Gary sold it for $6,250, incurring expenses for the sale of $250. The taxable gain on the sale would be A) a $500 short-term capital gain. B) a $250 long-term capital gain. C) a $250 short-term capital gain. D) a $500 long-term capital gain.

The amount realized on the sale is $6,000 (sale price of $6,250 reduced by the selling expenses of $250). This is compared to the fair market value (FMV) ($5,750) on the date of death, which is the recipient's basis. This generates a $250 gain. The issue then becomes one of holding period. The holding period of an asset acquired from a decedent is deemed to be long term. Thus, Gary has a $250 long-term capital gain.

Paula purchased an interest in a publicly traded partnership and has experienced a current loss of $7,000. If she purchased a nonpublicly traded partnership with $10,000 of passive income, how much of the passive loss may be used to offset Paula's income in the current year? A) $7,000 B) $0 C) $3,000 D) $10,000

The answer is zero. Losses from publicly traded partnerships cannot be offset against income from nonpublicly traded partnerships.

Which of the following may enable a direct participation program to provide specific tax advantages to the investors? A) Passive activity loss rules B) Partnership basis rules C) Special allocations D) At-risk rules

The benefits that flow through from a partnership entity may be enhanced by the potential, under certain circumstances, "special allocation" of certain items of income, expense, gain, or loss. The passive activity loss rules state that passive losses may only be deducted against passive income (not a tax advantage); there are no partnership basis rules; and the at-risk rules are defined as the maximum deductible loss for an investment limited to the amount that the taxpayer-investor has at risk at the end of the current year. None of the other answers are direct participation programs, which concern business organizations that function as tax conduits.

Blake, a sole proprietor, is selling several business assets. He has been told by a friend that the items he is selling are not capital assets and are subject to the ordinary income tax rate. You are his financial planner and tell him that the gains on Section 1231 assets can be treated as capital gains for income tax purposes subject to certain rules. Which of the assets Blake sold are Section 1231 assets? A) A copyright on the theme song Blake's company uses in its advertising that Blake wrote B) The building and land sold when Blake's business moved to a new location C) Blake's inventory of electric guitars his business manufactures D) Accounts receivable

The building and land sold when Blake's business moved to a new location qualify under Section 1231 as depreciable personal or real property used in business or for the production of income. The building portion of the property was depreciable property. While they are not considered capital assets, under Section 1231 they are taxed using capital gain rates, subject to the Section 1245 and 1250 rules for depreciation recapture rules. Losses are always ordinary and not subject to the $3,000 ($1,500 for MFS) ordinary loss limitation. Accounts receivable, inventory, and copyrights and other creative works held by the creator are all ordinary assets that would result in ordinary income tax (not capital gain) if sold at a gain.

Which of the following assets is NOT generally considered a capital asset? A) A personal auto B) A personal residence C) U.S. government securities held for investment D) A computer used in a business

The business computer is Section 1231 property, not a capital asset. A capital asset is any asset that is not a copyright or creative work, accounts or notes receivable, depreciable property used in a trade or business or for production of income (such as a computer), or inventory.

Carol owns and operates a retail electronics store with annual sales of approximately $2 million. The store also specializes in repairing computers and other small electronics. Approximately 30% of her sales are with extended credit terms. What method of tax accounting may be used for Carol's business? The accrual method, because inventory is such a large component of the business The cash method, because it provides flexibility in the timing of income and expenses The hybrid method, because the business involves both inventory and service The installment sale method, to spread the gain over a longer time frame A) I, II, and III B) I, II, III, and IV C) III only D) I only

The cash method may be used because average annual gross receipts are under $27 million for the prior tax years. The accrual method may always be used. The hybrid method may be used because the business involves both inventory and service. The installment method may not be used, as dealers are not allowed to use the installment method.

MSC, Inc. is a closely held C corporation that manufactures boilers. The company has been in business for over 45 years. MSC has active income this year of $250,000 and no passive or portfolio income. The company also leases equipment that generates passive losses of $120,000 per year. How much of the passive loss can the company use this year? A) $120,000 B) $100,000 C) $0 D) $25,000

The company can deduct the entire $120,000 because it is a closely held C corporation that is not a personal service corporation. Passive losses may be used to offset active income, but not portfolio income.

Jasmine and Luke, a married couple, bought 100 shares of Mutual Fund B for $3,200 on March 2 of this year. On December 19 of this year, they sold the 100 shares of Mutual Fund B for $3,500. They used the proceeds to purchase a trailer for its FMV of $3,500 and immediately donated it to their church to use for special events. What are the tax consequences of this sale? A) The couple has a long-term capital gain. B) There is no recognized gain on the sale because the proceeds were used for a donation. C) The couple has donated appreciated property. D) The couple has a short-term capital gain of $300 on the sale.

The couple have a recognized short-term capital gain of $300. Separately, they have donated property with a value of $3,500 to the church.

During the current tax year, Carolyn has Section 1231 gains of $14,000 and Section 1231 losses of $3,000. Four years ago, Carolyn reported a net Section 1231 gain of $6,000. Three years ago, Carolyn had a Section 1231 loss of $4,000. Which of the following correctly describes the amount and treatment of the gain? A) $11,000 treated as ordinary income B) $7,000 treated as capital gain; $4,000 treated as ordinary income C) $7,000 treated as ordinary gain; $4,000 treated as capital gain D) $11,000 treated as capital gain income

The current Section 1231 gain ($11,000) is treated as ordinary income to the extent of unrecaptured Section 1231 losses during the five-year lookback period. There are $4,000 of unrecaptured Section 1231 losses during the lookback period. Thus, $4,000 is ordinary income, and the remaining $7,000 is treated as long-term capital gain.

Karl anticipates adjusted gross income of $100,000 for the current tax year. He is considering making a gift of appreciated stock to his alma mater, Ohio State University (OSU). His basis in this stock, purchased four years ago, is $18,000. The stock has a current fair market value of $70,000. If Karl gifts the stock to OSU, what is the maximum allowable charitable deduction that Kurt can receive in the current tax year? A) $18,000 B) $21,000 C) $30,000 D) $50,000

The deduction for a donation of LTCG property to a 50% organization (public charity) typically is restricted to 30% of AGI, with the deduction based on the fair market value of the asset. The 50% election would not make sense in this situation, as the deduction would be based on the basis of the stock of $18,000.

Michelle Will has interest income of $23,000 in the current tax year. She paid brokers' commissions of $2,000 on stock purchases and had $40,000 of investment interest expense. What amount, if any, of investment interest expense may be deducted as an itemized deduction? A) $21,000 B) $23,000 C) $33,000 D) $0

The deduction for investment interest expense ($40,000) is limited to net investment income ($23,000). The remaining portion ($17,000), however, can be carried forward into future years. Note that the broker's commissions are not deductible, nor considered an expense when calculating net investment income.

All of the following deductions are allowable in arriving at adjusted gross income except A) alimony paid. B) qualifying contributions to Keogh-qualified and self-employed tax-advantaged plans. C) student loan interest (limited). D) 100% of self-employment tax paid.

The deduction for self-employment tax paid is generally limited to the calculated employer share, or 50%, not 100%.

Which of these is CORRECT regarding the qualified education interest deduction? A) The maximum deduction of $2,500 may be taken by a married taxpayer filing jointly with $200,000 of MAGI. B) The deduction may only be taken as an itemized deduction. C) An individual who is eligible to be claimed as a dependent may take the deduction. D) The deduction may only be claimed by the taxpayer legally obligated to make the loan payments.

The deduction may only be claimed by the taxpayer legally obligated to make the loan payments. If the loan is in the student's name, the parents may not claim the deduction, even if they make the payments. The deduction is taken as an adjustment to income. There is a $145,000 to $175,000 AGI phaseout for a married couple filing jointly. An individual who is eligible to be claimed as a dependent may not take the deduction.

To be considered a responsible person by the IRS, which one of the following is among the important factors? A) Impressive title B) In charge of hiring and firing employees C) Shareholder of a similar company D) No authority to sign checks

The determination of whether someone is a responsible person is a test of facts and circumstances. Some of the questions the IRS usually asks in determining responsibility are as follows: Was the individual an officer or director? Was the individual a shareholder? Was the individual a member of a board of directors? Did the individual have the authority to sign checks? Was the individual responsible for hiring and firing employees? Did the individual have actual authority or merely an impressive title?

Miguel, a shareholder in ABC Corporation, received a cash distribution from the corporation in the amount of $22,000. The corporation had $8,000 of accumulated earnings and profits and $5,000 and current earnings and profits. Miguel had basis in the stock of $6,000. Which one of the following correctly identifies the proper treatment of the distribution from the corporation? A) $13,000 ordinary dividend; $6,000 capital gain; $3,000 return of capital B) $7,000 ordinary dividend; $6,000 return of capital; $9,000 capital gain C) $5,000 ordinary dividend; $6,000 return of capital; $11,000 capital gain D) $13,000 ordinary dividend; $6,000 return of capital; $3,000 capital gain

The distribution from a corporation is determined in a three-step manner. To the extent of current and accumulated earnings and profits, the distribution is an ordinary dividend, subject to short-term capital gain rates. Next, the distribution is treated as a nontaxable return of capital until the basis in the stock is exhausted. Any distribution after that is considered long-term capital gain.

Policyholder dividends from a whole-life insurance policy are generally tax exempt. In which of these situations would the policyholder dividend be tax exempt? A) The policyholder receives dividends that are less than the investment in the contract. B) The dividend is from a MEC and is received in cash. C) The policyholder receives dividends greater than the investment in the contract. D) The dividend is from a MEC and is used to pay a loan.

The dividend from a life insurance policy is typically tax exempt. However, the dividend is taxable to the extent that the dividends received exceed the investment in the contract. The dividend is also taxable if it is from a MEC and received in cash, or is used to pay a loan.

Marion donated a truck to the local food bank to use for picking up food donations. Marion had purchased the truck several years ago for $15,000, and it currently has a value of $3,400. Which of the following statements regarding the documentation Marion must have to support his charitable contribution of the truck is CORRECT? A) An appraisal must be attached to Marion's income tax return for the year of the donation. B) A noncash contribution under $5,000 needs no documentation to support the donation. C) The documentation must have the description of the property, the name of the receiving charitable organization, the date of the contribution, and the amount of the donation. D) A letter from the food bank thanking him for the donation of the truck is sufficient documentation.

The donor-taxpayer must have a canceled check, bank record, or a receipt from the donee organization to substantiate the deduction. The documentation must have the amount of cash or description of property, the name of the receiving charitable organization, the date of the contribution, and the amount of the donation. An appraisal is not required for noncash property over $500 and less than or equal to $5,000. However, taxpayers may wish to get an independent appraisal to support the deduction claimed.

Steven owned a duplex that he rented to tenants. He acquired the property several years ago for $296,000. He used the straight-line method of cost recovery, which totaled $75,000. Steven sold the property in February of the current year for $330,000. Steven is in the 32% marginal income tax bracket. What is the amount and nature of the gain on the sale? A) $75,000 long-term capital gain B) $75,000 Section 1250 recapture C) $34,000 regular long-term capital gain; $75,000 unrecaptured Section 1250 income D) $34,000 regular long-term capital gain; $75,000 Section 1250 recapture

The entire gain of $109,000 is treated as Section 1231 gain. $75,000 of the gain is created by the cost recovery deductions, and is subject to a maximum rate of 25% as unrecaptured Section 1250 income. The remaining $34,000 of gain is created by actual appreciation of the duplex, and is subject to the regular long-term capital gain rate of 15%.

Frank, a single taxpayer, owned a warehouse that he rented as commercial property. He acquired the property several years ago for $196,000. He used the straight-line method of cost recovery, which totaled $35,000. Frank sold the property in February of the current year for $230,000. Frank is single, and has taxable income (not including the real estate gain) of $475,000. What is the amount and nature of the gain on the sale? A) $69,000 ordinary income B) $35,000 unrecaptured Section 1250 gain; $34,000 long-term capital gain C) $34,000 Section 1231 gain; $35,000 ordinary income D) $7,000 ordinary income

The entire gain of $69,000 is treated as Section 1231 gain, because there is no excess depreciation on the use of the straight-line method. So, $35,000 of the gain is subject to a maximum rate of 25%, as unrecaptured Section 1250 income, and the remaining $34,000 of gain is subject to the maximum regular long-term rate of 20%. The 20% long-term capital gain rate applies, as his taxable income is over the $445,850 breakpoint for the 20% rate. Note that Section 1250 recapture (ordinary income treatment) applies only to excess depreciation—in other words, the excess of an accelerated method over what would have been deducted if straight-line had been used. All realty placed in service after 1986 is depreciated using straight-line, and there is NO recapture (ordinary income) where straight-line depreciation was used.

Seven years ago, Karen Price purchased U.S. EE savings bonds for $5,000. During the current year, when Karen was 27 years old, she redeemed the bonds to help pay for her graduate school tuition. The accrued value at the time of redemption was $7,000. Assume Karen incurs $11,000 of tuition expenses in the year. What are the tax consequences upon the redemption of the bonds? A) The income on the bonds is generally subject to state income taxes. B) All the interest may be excluded. C) A portion of the interest may be excluded. D) All accrued interest is taxable in the current year.

The exclusion for EE bond interest redeemed to pay for qualifying higher-education expenses applies only to bonds purchased by an individual age 24 or older, and held in that person's name, or jointly with a spouse. Karen is 27 years old; the bonds were purchased 7 years ago, when Karen was approximately 20. Because Karen does not qualify for the exclusion of the interest income because she was not age 24 or older at the time of purchase. All the interest is taxable in the year the bonds are redeemed. Remember that the interest of EE bonds is deferred until maturity, unless an election has been made to have the interest taxed each year as it accrues. Also, the interest income from EE bonds (and other federal government obligations) is generally not subject to state income tax.

Which of the following statements correctly describes the method for calculating the exclusion ratio for a fixed annuity? A) The number of expected payments is divided by the investment in the annuity contract. B) The investment in the annuity contract is divided by the number of expected payments. C) The total expected return is divided by the investment in the annuity contract. D) The investment in the annuity contract is divided by the total expected return

The exclusion ratio for a fixed annuity contract is not calculated by dividing the total expected return by the investment in the contract. It is calculated by dividing the investment in the contract by the total expected return.

Jerry owns a dry-cleaning business. During the current year, Jerry purchased and placed into service $730,000 of equipment. He had taxable income of $745,000. Jerry is in the highest marginal income tax bracket this year, and expects to be in that bracket for two more years. After that, he plans to semi-retire, but keep the business open for another five years. He expects to drop into the lowest marginal bracket when he semi-retires. What advice would you give Jerry regarding the use of Section 179, bonus depreciation, and cost recovery deductions? A) Forgo Section 179 and bonus depreciation and use the Modified Accelerated Cost Recovery System (MACRS) table. B) Elect the maximum Section 179 and elect the straight-line method. C) Use the bonus depreciation provision. D) Forgo Section 179 and bonus depreciation and elect the straight-line method.

The fact pattern indicates that Jerry is in the highest marginal bracket for three years, and then will be in the lowest marginal bracket after that. It makes sense to maximize the depreciation deduction this year when Jerry is in the highest marginal bracket. By using the bonus depreciation provision, the entire $730,000 may be deducted in the year of acquisition.

During 2022, Judy, a sole proprietor, purchased new equipment (seven-year property) for her manufacturing business at a cost of $600,000. Judy is in a 12% marginal income tax bracket this year, and expects to be in that bracket for two more years. She is extremely confident that she will be in the highest marginal bracket after that. What advice would you give Judy regarding the use of bonus depreciation and cost recovery deductions? A) Use the maximum bonus depreciation and elect the straight-line method. B) Forgo bonus depreciation and use the Modified Accelerated Cost Recovery System (MACRS) table. C) Use the maximum bonus depreciation and use the Modified Accelerated Cost Recovery System (MACRS) table. D) Forgo bonus depreciation and elect the straight-line method.

The fact pattern indicates that Judy is in the lowest marginal bracket for three years, and will be in the highest marginal bracket after that. It makes no sense to maximize the depreciation deduction in years when Judy is in the lowest marginal brackets. By forgoing bonus depreciation and using straight-line, more deductions are pushed into the last five years of the depreciation schedule, when Judy will be in the highest marginal bracket. Remember that because of the half-year convention, seven-year property is depreciated over eight years. Under TCJA, 100% bonus depreciation is allowed for all personalty. In other words, 100% of the cost is deducted in the first year.

Gil owns a portfolio of income-producing real estate. Gil retains ownership of the real estate but directs that the rental income be paid to his son, Kevin. The income is paid directly to Kevin, who reports it as part of his taxable income. Gil does not report the income on his tax return. With which one of the following potential tax traps should Gil be most concerned? A) Ownership attribution rules B) Constructive receipt C) Substance over form D) Assignment of income

The fact that Gil retains ownership of the property and merely assigns the income to someone else is a potential tax trap for him. The assignment of income doctrine serves to tax the person who actually owns the property producing the income. The income cannot merely be assigned to another in order to generate tax advantages.

Gil owns a portfolio of income-producing real estate. Gil retains ownership of the real estate but directs that the rental income be paid to his son, Kevin. The income is paid directly to Kevin, who reports it as part of his taxable income. Gil does not report the income on his tax return. With which of the following potential tax traps should Gil be most concerned? A) Ownership attribution rules B) Substance over form C) Assignment of income D) Constructive receipt

The fact that Gil retains ownership of the property and merely assigns the income to someone else is a potential tax trap for him. The assignment of income doctrine serves to tax the person who actually owns the property producing the income. The income cannot merely be assigned to another to generate tax advantages.

Mike is involved in the business of breeding show horses on a part-time basis as a sole proprietor. He has shown a significant net loss in this business every year for the last six years, and he has used these losses to reduce the tax liability on the salary he has earned from his law practice. With which one of the following potential tax traps should Mike be most concerned? A) Hobby loss B) Passive activity C) Substance over form D) Assignment of income

The fact that this business has not been profitable and is merely a side business increases the likelihood it could be reclassified as a hobby loss. Mike has operated the business only part time, and he has not demonstrated that he is attempting to make it a profitable business. The fact that huge losses have occurred every year overrides the presumption that Mike is in the business to make a profit.

Miranda is involved in the business of breeding show horses on a part-time basis as a sole proprietor. She has shown a significant net loss in this business every year for the last six years, and she has used these losses to reduce the tax liability on the salary she has earned from her law practice. With which of these potential tax traps should Miranda be most concerned? A) Assignment of income B) Substance over form C) Passive activity D) Hobby loss

The fact that this business has not been profitable and is merely a side business increases the likelihood it could be reclassified as a hobby loss. Miranda has operated the business only part time and has not demonstrated that she is attempting to make it a profitable business. The fact that huge losses have occurred every year likely overrides the presumption that Miranda is in the business to make a profit.

In 2022, Sonja, age 16, is claimed as a dependent on her parents' income tax return and has $5,850 of short-term capital gains and interest income from a UGMA account that was established by her grandparents many years ago. Sonja's parents, who file jointly, have taxable income of $400,000. What is Sonja's income tax liability for 2022? A) $862 B) $0 C) $1,396 D) $1,251

The first $1,150 of unearned income is sheltered by the child's limited standard deduction. The next $1,150 is taxed to the child at the child's marginal income tax bracket. All unearned income in excess of $2,300 (for 2022) is taxed to the child at the parents' rates. $5,850unearned income(1,150)standard deduction(1,150)taxed at child's rate of 10%: $1,150 × 10% =$115$3,550taxed at parents' marginal rate 32%$1,136Total$1,251

Which one of the following objectives of the federal taxation system would include keeping prices stable? A) Economic objective B) Social objective C) Writing tax code D) Revenue raising

The first, and perhaps most important, goal of the economic objective is price stability. As a social objective, JGTRRA and TCJA significantly increased the Section 179 expense limit and the amount of depreciation deductions (bonus depreciation) that may be claimed in the first year in an attempt to stimulate purchases of business assets. Revenue raising through corporate, individual, and payroll taxes is an important objective of the federal taxation system.

Margaret purchased a used pickup truck at a cost of $12,400 and sales taxes of $600 to use in her floral business. She purchased the pickup (five-year property) and placed it in service on June 1 of the current tax year. Assume that Margaret opts out of bonus depreciation. Using MACRS, what is the first-year cost recovery deduction that Margaret can claim? A) $1,300 B) $1,240 C) $2,600 D) $5,200

The first-year percentage for five-year property from the MACRS table is 20%. This is multiplied by the basis of $13,000 (including the capitalized cost, the sales taxes of $600) to give a deduction of $2,600. The half-year convention is built into the MACRS table. (The MACRS table will be provided on the end-of-course exam.)

Which one of the following is a tax return preparer failure that would initiate a tax penalty from the IRS? A) Failure to have the clients sign their return B) Failure to maintain a list of all returns prepared for the past 10 years C) Failure to keep a copy of all returns prepared for at least the last three years or to maintain a list of returns prepared D) Failure to provide a taxpayer with a receipt for their services

The following actions would constitute a tax return preparer's failure and be subject to a tax penalty: failure to provide a taxpayer with a copy of their return, failure to keep a copy of all returns prepared for at least the last three years or to maintain a list of returns prepared, and failure to sign a return as preparer and give their tax identification number on the return.

Which one of the following is a tax return preparer failure that would initiate a tax penalty from the IRS? A) Failure to maintain a list of all returns prepared for the past five years B) Failure to provide a taxpayer with a receipt for their services C) Failure to keep a copy of all returns prepared for at least the last 10 years D) Failure to sign a return as preparer and give their tax identification number on the return

The following actions would constitute a tax return preparer's failure subject to a tax penalty: failure to provide a taxpayer with a copy of their return, failure to keep a copy of all returns prepared for at least the last three years or to maintain a list of returns prepared, and failure to sign a return as preparer and give their tax identification number on the return.

Which of the following is NOT a step in the tax calculation process? A) Subtract adjustments to income from total income to get adjusted gross income. B) Subtract adjustments to income and standard or itemized deduction(s) then multiply by total income to get federal taxable income. C) Apply tax credits to tax liability to get total tax liability. D) Deduct the greater of itemized deductions or the standard deduction.

The following are involved in the income tax computation: *** Subtracting adjustments to income from total income to get adjusted gross income, and deducting the greater of itemized deductions or the standard deduction from AGI to arrive at taxable income.*** Subtracting exclusions from AGI is not a step in the tax calculation process. Excluded amounts simply do not show up as income on the return.

Which of the following is NOT a step in the tax calculation process? A) Calculate federal tax on federal taxable income. B) Deduct the greater of itemized deductions or the standard deduction. C) Subtract exclusions from AGI. D) Subtract adjustments to income from total income to get adjusted gross income.

The following are involved in the income tax computation: subtracting adjustments to income from total income to get AGI, and deducting the greater of itemized deductions or the standard deduction from AGI to arrive at taxable income. Subtracting exclusions from AGI is not a step in the tax calculation process. Excluded amounts simply do not show up as income on the return.

Which of the following is NOT a step in the tax calculation process? A) From the final calculation of federal taxable income, find the federal tax owed or refundable. B) Subtract adjustments to income from total income to get adjusted gross income. C) Add in the greater of itemized deductions or the standard deduction. D) Reduce tax owed by allowable credits.

The following are involved in the income tax computation: subtracting adjustments to income from total income to get adjusted gross income, and deducting the greater of itemized deductions or the standard deduction from AGI to arrive at taxable income. The calculation of federal tax is on federal taxable income.

Which of the following is NOT a step in the tax calculation process? A) Total income minus adjustments to income and standard or itemized deduction(s) equals federal taxable income. B) Deduct the greater of itemized deductions or the standard deduction. C) Add adjustments to income to total income to get adjusted gross income. D) From the final calculation of federal taxable income, find the federal tax owed or refundable.

The following are involved in the income tax computation: subtracting adjustments to income from total income to get adjusted gross income, and deducting the greater of itemized deductions or the standard deduction from AGI to arrive at taxable income. The calculation of federal tax is on federal taxable income.

Which of the following is NOT a step in the tax calculation process? A) Calculate federal tax on total income. B) Deduct the greater of itemized deductions or the standard deduction from AGI to arrive at taxable income. C) Claim allowable tax credits. D) Subtract adjustments to income from total income to get adjusted gross income.

The following are involved in the income tax computation: subtracting adjustments to income from total income to get adjusted gross income, subtracting tax withholdings from total tax liability, and deducting the greater of itemized deductions or the standard deduction from AGI to arrive at taxable income. Credits are applied to tax liability. The calculation of federal tax is on federal taxable income.

In February, Jeremy purchased a new computer (five-year property) for use in his business. The cost of the computer was $4,300, while freight and setup charges totaled $600. What is the first-year cost recovery deduction using the straight-line method? A) $490 B) $860 C) $430 D) $790

The freight and setup charges of $600 must be capitalized (i.e., added to the cost of the computer) to give a total basis of $4,900. The straight-line rate for five-year property is 20% (100% divided by 5), but the half-year convention limits the deduction to 50% in the year of acquisition. Thus, $4,900 times 10% equals $490. Note that bonus depreciation is not used, as the fact pattern states to use straight-line.

Maya owns and operates a business as a sole proprietor. Several years ago, she purchased office furniture at a cost of $9,000 to use in her business. She used the straight-line method to recover the cost of the furniture. She claimed $3,214 of cost recovery deductions. She sold the furniture for $10,500. What is the amount and nature (character) of the gain resulting from this disposition? A) $3,214 Section 1245 gain, $1,500 Section 1231 gain B) $1,500 Section 1245 gain, $3,214 Section 1231 gain C) No Section 1245 gain, $4,714 Section 1231 gain D) $4,714 Section 1245 gain, no Section 1231 gain

The gain realized and recognized is the difference between the $10,500 amount realized from the sale and the adjusted basis of $5,786. Thus, the total gain is $4,714. The Section 1245 cost recovery recapture is the lesser of the cost recovery deductions taken ($3,214) or the gain realized ($4,714). Thus, the Section 1245 recapture is $3,214. The remaining $1,500 of gain is attributable to actual appreciation of the asset; therefore, there is $1,500 of Section 1231 gain.

Which of the following would result in higher taxation of one party with a premarital agreement? A) Transfer for premarital is made B) Transfer for consideration is made C) Transfer under the agreement is treated as an estate D) Transfer under the agreement is treated as a gift

The income tax consequences of the premarital agreement depend in large part upon whether the transfer under the agreement is treated as a gift (where income tax is avoided) or as a transfer for consideration (which will probably result in the recognition of significant income by one party).

Several years ago, Ben purchased equipment at a cost of $11,000 to use in his dry-cleaning business. He used the straight-line method of cost recovery. He deducted $7,074 of cost recovery. He sold the equipment earlier this year for $12,000. What is the amount and nature (character) of the gain resulting from this disposition? A) $8,074 Section 1245 gain; no Section 1231 gain B) $1,000 Section 1245 gain; $7,074 Section 1231 gain C) $7,074 Section 1245 gain; $1,000 Section 1231 gain D) No Section 1245 gain; $8,074 Section 1231 gain

The gain realized and recognized is the difference between the $12,000 amount realized from the sale, reduced by the adjusted basis of $3,926. Thus, the total gain is $8,074. The Section 1245 cost recovery recapture is the lesser of the cost recovery deductions taken ($7,074) or the gain realized ($8,074). Thus, the Section 1245 recapture is $7,074. The remaining $1,000 of gain is attributable to actual appreciation of the asset; therefore, there is $1,000 of Section 1231 gain. The Section 1245 income is treated as ordinary income, while the Section 1231 income generally receives long-term capital gain treatment. For Section 1245 property, it makes no difference whether straight-line or accelerated depreciation were used.

In March of the current year, Susan sold her principal residence for a total price of $450,000. Susan purchased the house 15 years ago for $80,000. She has not made any improvements to the house. Real estate commissions of $27,500 resulted from the sale. Sixteen months later, Susan bought a new residence for $225,000. What amount of gain, if any, must be recognized on the sale of Susan's residence? A) $120,000 B) $92,500 C) $342,500 D) $0

The gain recognized is the amount taxable—it is the amount after the Section 121 exclusion. This is computed as follows: Gain realized:Amount realized:Sale price$450,000Selling expenses(27,500)Total amount realized$422,500Less adjusted basis(80,000)Gain realized$342,500Gain recognized:Gain realized$342,500Less exclusion(250,000)Gain recognized$ 92,500 The purchase of the new residence is not relevant to the Section 121 exclusion. If there had been a loss on the sale of the principal residence, it would not be deductible. The loss on a personal-use asset is nondeductible.

Which of the following is subject to the self-employment tax? Distributive share of limited partnership operating income Flow-through of S corporation income Distributive share of general partnership operating income Interest or dividends from investments A) I, II, III, and IV B) I and III C) III only D) II and III

The general partnership operating income is self-employment income. By definition, the other items of income are not subject to the self-employment tax.

Which of the following is subject to the self-employment tax? Distributive share of limited partnership operating income Flow-through of S corporation income Distributive share of general partnership operating income Interest or dividends from investments A) II and IV B) I, II, III, and IV C) III only D) I and III

The general partnership operating income is self-employment income. By definition, the other items of income are not subject to the self-employment tax.

Nathan has a salary of $100,000, dividends of $4,000, and limited partnership income of $10,000. The limited partnership is publicly traded. During January of the current year, Nathan purchased an interest in a nonpublicly traded limited partnership that will generate a $12,000 passive loss during the current tax year. How much of this passive loss, if any, is deductible by Nathan during the current tax year? A) $0 B) $12,000 C) $4,000 D) $10,000

The general rule is that passive losses are deductible only against passive income. However, passive income from a publicly traded partnership cannot be offset by passive losses arising from any other source. Thus, the passive losses from the new partnership will not be deductible.

Jim is planning to make a charitable contribution to a local university, a qualifying charitable organization. He is going to contribute a piece of real estate that he has owned for six years. The fair market value of the property is $80,000, and his basis in it is $55,000. He has an AGI of $120,000. Jim wants to maximize the amount of charitable contribution deductions from the donation of the real estate. What is the amount of charitable contribution deduction that Jim may claim in the current year? A) $40,000 B) $60,000 C) $36,000 D) $55,000

The gift of long-term capital gain (LTCG) property is generally based on the fair market value of the property. The university is a 50% organization, a public charity. LTCG property contributed to a 50% organization involves a 30% of AGI limitation, and 30% of $120,000 is $36,000. There is also a $44,000 carryforward for up to five years. Jim could have made a 50% election to maximize the current-year deduction, but that would have reduced his overall deductions. If Jim had made a 50% election, he could have deducted $55,000 in the current year. By forgoing the 50% election, he is allowed to deduct the full $80,000 fair market value—$36,000 this year and $44,000 over the next several years.

Mark operates a sole proprietorship from his apartment. His gross income for the current tax year is $61,000. Business expenses not associated with his home office total $63,000. Expenses associated with the home office total $4,200. How much of the home office expense, if any, may Mark deduct for the current year? A) $4,200 B) $2,000 C) $275 D) $0

The home office expense deduction is limited to the earned income from the business. In other words, the home office expense deduction can generally neither create nor add to a loss. In this situation, the $61,000 of gross income is reduced by the $63,000 of business expenses not associated with the home office, to leave no earned income. Thus, of the $4,200 of home office expenses, none would be deductible in the current year. Note that the entire $4,200 of home office expenses would be subject to a carryforward.

Sandy operates a hairstyling sole proprietorship, and performs her business dealings from her home on a part time basis. She also has a full time job as an esthetician and is paid as a W-2 employee earning $46,000.. Her gross income for the current tax year is $71,000. The hairstyling income is $25,000. Due to supply and marketing costs, the business expenses not associated with the home office total $27,275. Expenses associated with the home office total $4,200. How much of the home office expense, if any, may Sandy deduct for the current year? A) $275 B) $4,200 C) $0 D) $2,000

The home office expense deduction is limited to the earned income from the business. In other words, the home office expense deduction can generally neither create nor add to a loss. In this situation, the $71,000 of gross income is reduced by the $27,275 of business expenses not associated with the home office. Sandy may claim a loss against ordinary income with her Schedule C. However, none of the home office expenses would be deductible in the current year. Note that the entire $4,200 of home office expenses would be subject to a carryforward.

In February, Bryan purchased a new high-speed copier for use in his printing business. The cost of the copier was $8,250, sales taxes were $550, and installation charges totaled $1,200. Assume that Bryan opts out of the bonus depreciation provision. What is the first-year cost recovery deduction using the straight-line method? A) $880 B) $1,000 C) $945 D) $2,000

The installation charges of $1,200 and the sales taxes of $550 must be capitalized—that is, added to the cost of the copier to give a total basis of $10,000. A copier is five-year property. (Copiers, cars, computers, and computer peripherals are five-year properties; furniture and other equipment are seven-year properties.) The straight-line rate for five-year property is 20% (100% divided by five), but the half-year convention limits the deduction to half of a full year's depreciation in the year of acquisition. Thus, $10,000 times 10% equals $1,000. If Bryan had not opted out of bonus depreciation, the entire $10,000 would be depreciated in the first year.

For two years, Lisa Carson was able to pay the premiums on her whole life policy without borrowing. For the past two years, she has borrowed from the cash value of her whole life policy to pay the premiums. Last year, she paid $95 of interest on the funds she borrowed. What are the tax implications in this situation? A) The interest expense is tax deductible because it does not exceed $100. B) The interest is deductible because Lisa is in the business of continuing her insurance and the interest is deductible business interest expense. C) The interest expense is not tax deductible. D) The interest expense is not tax deductible because it does not exceed $100.

The interest expense is not tax deductible because interest on a loan incurred to purchase personal life insurance protection is considered personal interest, which is not deductible. Personal loan interest is not tax deductible, regardless of whether the lender is a bank or a life insurance company.

Arlene, who is 75 years old and single, is listed as a dependent on her son's tax return. During the current year, she received $1,700 interest on a savings account, $1,000 of interest income from municipal bonds, and $11,000 of Social Security benefits. She also earned $1,250 from a part-time job. What is Arlene's gross income? A) $2,950 B) $14,950 C) $13,950 D) $5,700

The interest income on the savings account and the earned income from the part-time job are included in Arlene's gross income. The interest income earned on the municipal bond is tax exempt and is excluded from her gross income. Her gross income is equal to $2,950 ($1,700 + $1,250). The Social Security benefits will not be included in her gross income.

A taxpayer intends to use a home equity loan to obtain funds to purchase municipal bonds. Which of the following is CORRECT regarding the income tax implications of this scenario? A) None of these choices apply. B) The municipal bond interest becomes taxable. C) The interest on the home equity loan is not deductible. D) The interest on the home equity loan is fully deductible.

The interest on the municipal bond continues to be tax exempt. There is no deduction allowed for the interest on funds borrowed to purchase tax-exempt securities.

Yukiko, an investor, has the following items related to her investments during the current tax year: Investment interest expense$4,000Dividend and interest income$3,500Investment adviser's fees$1,750Adjusted gross income$50,000 What is Yukiko's maximum allowable investment interest expense deduction for the current year? A) $3,500 B) $4,000 C) $1,750 D) $2,250

The investment interest expense deduction is limited to the taxpayer's net investment income of $3,500. Net investment income is simply the investment income of $3,500. The inclusion of the dividend income results in the largest investment interest expense deduction. However, including the dividends in investment income results in forgoing the potential preferential rates on the dividends.

Alisha, a CFP® certificant and fee-only financial planner, has assisted Roger, a self-employed physician, in income tax return preparation and investment planning during the year. On which of the following schedules may Alisha's fee be deductible by Roger on his federal income tax return? Schedule A—itemized deductions Schedule C—profit or loss from business Schedule D—capital gains and losses A) I, II, and III B) I and II C) I only D) II only

The investment planning fees are not deductible. This expense may not be added to the basis of securities. The tax preparation fee is partly nondeductible (the portion attributable to the individual return), and the portion of the fee for completing the Schedule C is a business expense that would be deducted on the Schedule C.

Fred, age 59, is a single taxpayer. He has wage income of $90,000 for the current tax year. Fred is not an active participant in a company-maintained retirement plan. In addition, he has the following: Long-term capital gains$4,000 Short-term capital losses$9,000 Loss from active participation rental real estate$3,700 Alimony paid to ex-wife$5,200 Gambling winnings$7,100 Gambling losses$4,100 Interest income$3,500 Sole proprietorship (Schedule C) income$2,000 Self-employment tax liability$283 Qualified home mortgage interest$11,890 Real estate tax paid$1,840 Investment interest expense$4,925 Charitable contributions (cash)$2,975 Total medical expenses$4,217 State and local income taxes$1,625 Consumer interest$2,180 Unreimbursed employee business expenses$1,560 IRA contribution$6,000 Fred's divorce was finalized in 2017. What is the amount of Fred's allowable itemized deductions? A) $35,712 B) $25,930 C) $26,647 D) $28,607

The itemized deductions total $25,930. This is composed of the qualified home mortgage interest of $11,890, the real estate tax paid of $1,840, the investment interest expense of $3,500, the charitable contributions of $2,975, the state and local income taxes of $1,625, and the gambling losses to the extent of gambling winnings ($4,100). Note that the unreimbursed employee business expenses are not deductible. Also, the medical expenses are not deductible because they do not exceed 7.5% of adjusted gross income. Also note that the investment interest expense of $4,925 is deductible only up to the amount of net investment income, which in this situation is $3,500 (the interest income). Consumer interest (interest on personal auto loans, credit card debt, etc.) is nondeductible. Home mortgage interest$11,890 Property taxes$1,840 Investment interest expense$3,500 Charitable contributions$2,975 State and local income taxes$1,625 Gambling losses (to extent of innings)$4,100 Allowable itemized deductions$25,930

Ron has the following income for the current tax year: Schedule C net income: $12,000 Flow-through of general partnership operating income: $15,000 Wages: $80,000 Net income from rental of real estate: $10,000 What is the amount of Ron's self-employment tax? A) $9,467 B) $1,696 C) $3,815 D) $5,228

The items included in the computation of self-employment income are the Schedule C income and the flow-through of general partnership operating income. The total of these items is $27,000. This is then reduced by 7.65% of this amount, and then multiplied by 15.3% to give us $3,815. Remember, the 7.65% subtraction always applies. Because the total of the self-employment income and the wages received are less than the wage base, no special computation is necessary.

Dr. John Welch, a dentist, is a single taxpayer. For the 2022 tax year, he has a taxable income of $300,000. Included in the taxable income is $50,000 of net long-term capital gains from the sale of an antique clock collection. At what rate will his net capital gain be taxed? A) 20% B) 25% C) 35% D) 28%

The long-term capital gain rate for collectibles is 28% if the taxpayer is in a marginal income tax bracket greater than 28%. John's taxable income places him in the 35% marginal income tax bracket, so his collectibles gain is taxed at 28%.

This year, Ken sold several securities that left him with the following types of gains and losses: Long-term capital gain: $8,000 Short-term capital gain: $1,800 Long-term capital loss: $2,200 Short-term capital loss: $1,000 What is the net capital gain or loss on Ken's security sales? A) Net long-term loss of $1,400 B) Net long-term gain of $2,640 C) Net long-term gain of $5,800 and net short-term gain of $800 D) Net long-term gain of $2,320 and net short-term gain of $800

The long-term gain and loss are netted, leaving a long-term gain of $5,800. Short-term gains and losses are netted, leaving a short-term gain of $800. These are left separate due to the disparate tax treatment of short-term versus long-term gains.

During the current tax year, Kendra sold several securities that resulted in the following types of gains and losses: long-term capital gain—$6,700; short-term capital gain—$7,000; long-term capital loss—$1,900; and short-term capital loss—$9,200. What is the net capital gain or loss on Kendra's security sales? A) Net long-term gain of $2,600 B) Net long-term gain of $4,800 and net short-term loss of $2,200 C) Net short-term gain of $2,600 D) Net short-term loss of $2,500 and net long-term gain of $5,100

The long-term items are netted, leaving a long-term capital gain of $4,800. The short-term items are netted, leaving a short-term capital loss of $2,200. The long-term capital gain is netted with the short-term capital loss to result in a net long-term capital gain of $2,600.

During the current tax year, Jamie sold several securities that resulted in the following types of gains and losses: a long-term capital loss of $6,700; a short-term capital loss of $7,000; a long-term capital gain of $1,900; and a short-term capital gain of $9,200. What is the net capital gain or loss on Jamie's security sales? A) Net long-term loss of $4,800; net short-term gain of $2,200 B) Net short-term gain of $7,300; net long-term loss of $300 C) Net long-term loss of $2,600 D) Net short term loss of $3,800

The long-term items are netted, leaving a long-term capital loss of $4,800. The short-term items are netted, leaving a short-term capital gain of $2,200. These are netted, leaving a net long-term capital loss of $2,600.

Sally has AGI of $250,000. In addition, she currently has passive income of $30,000 and passive losses of $48,000, $30,000 of which she uses to offset the passive income and $18,000 of which is suspended. Which of these activities has the greatest potential for reducing Sally's tax liability? A) Investing in an oil and gas limited partnership that is generating losses B) Investing in a newly created limited partnership involved in low-income housing that is producing deduction-equivalent tax credits C) Investing in an equipment leasing limited partnership that is producing passive losses D) Investing in active participation rental real estate that is producing a loss

The low-income housing credit is not subject to an AGI limitation if the low-income property was placed in service after December 31, 1989. Thus, she could use the deduction-equivalent tax credits to offset her tax liability. The active participation deduction is eliminated at $150,000 of AGI. The oil and gas limited partnership and the equipment leasing limited partnership would produce more passive losses that are nondeductible. The oil and gas limited partnership cannot be an oil and gas working interest, as the form of ownership would limit Sally's personal liability.

A lump-sum payment of the proceeds of a life insurance policy that is made to the beneficiary upon the insured's death A) is taxable if from a modified endowment contract (MEC). B) is taxable if the proceeds exceed the net investment in the policy. C) is generally exempt from income taxation. D) is typically taxable.

The lump-sum proceeds of a life insurance policy paid to a beneficiary at the death of the insured are exempt from income taxation.

Which of the following forms of inventory valuation provides the greatest flexibility in managing inventory? A) Specific identification B) First-in, first-out (FIFO) C) Last-in, first-out (LIFO) D) General identification

The major advantage of the specific identification method is that it allows maximum flexibility in managing inventory. For example, if the taxpayer has two identical inventory items, he may choose to sell the higher- or lower-cost item. The FIFO method treats inventory items that were purchased first as being sold first. There is no flexibility with this method. The LIFO method treats inventory items that were purchased most recently as being sold first. There is no flexibility with this method.

The marginal tax rate is obtained by A) dividing the calculated tax by total income. B) finding the tax bracket of total income. C) finding the tax bracket of the taxable income amount. D) dividing the calculated tax by taxable income.

The marginal tax rate is found by finding the tax bracket that contains the taxable income amount; it is the amount at which all subsequent taxable amounts will be taxed (until entering the next tax bracket). The effective tax rate is calculated by dividing the calculated tax by taxable income.

Which of the following best describes the marginal tax rate? A) The amount of tax and total income B) The amount of tax and taxable income C) The rate which is paid on the last taxable dollar D) The effective tax rate used

The marginal tax rate is found by finding the tax bracket that contains the taxable income amount; it is the amount at which all subsequent taxable amounts will be taxed (until entering the next tax bracket). The effective tax rate is calculated by dividing the calculated tax by taxable income.

John and Karen Postman will spend a total of $5,000 on day care for their two children (ages 9 and 10) in the current tax year. These expenses were incurred to allow both John and Karen to work outside the home. Their adjusted gross income is estimated at $138,000. What is the amount of child and dependent care credit, if any, to which they are entitled? A) $1,000 B) $1,750 C) $0 D) $600

The maximum amount of qualifying expenditures on which the credit may be based is $3,000 per child, or $6,000 for two or more children. In this situation, they spent $5,000. This is multiplied by 20% for taxpayers with an AGI greater than $43,000. Thus, $5,000 × 20% = $1,000.

John and Karen will spend $7,000 on day care for their two children (ages 9 and 10) in the current tax year. These expenses were incurred to allow both John and Karen to work outside the home. Their adjusted gross income is estimated at $138,000. What is the amount, if any, of child care credit to which they are entitled? A) $1,200 B) $960 C) $1,400 D) $480

The maximum amount of qualifying expenditures on which the credit may be based is $3,000 per child, or $6,000 for two or more children. This is multiplied by 20% for taxpayers with an AGI greater than $43,000. Thus, $6,000 × 20% = $1,200.

Which of these statements regarding the child tax credit is NOT correct? A) The maximum credit is $1,000 per qualifying child for 2022. B) The phaseout of the credit begins at $400,000 for a married couple filing jointly. C) A qualifying child is generally defined as an individual under the age of 17 (as of the close of the calendar year) for whom the taxpayer may claim a dependency exemption. D) A qualifying child includes a child or a descendant of a child, a stepchild, or an eligible foster child.

The maximum credit is $2,000, not $1,000, per qualifying child. A qualifying child is generally defined as an individual under the age of 17 (as of the close of the calendar year) who may be treated as a dependent. A qualifying child includes a child or a descendant of a child, a stepchild, or an eligible foster child. The credit is phased out by $50 for each $1,000 of AGI over the threshold of $400,000 for a married couple filing jointly ($200,000 for single taxpayers).

Which one of the following is accurate regarding the Lifetime Learning Credit? A) The maximum credit of $2,000 is per tax return. B) The maximum credit of $2,000 is per student. C) The credit may only be used during the first four years of postsecondary education. D) Qualified higher-education expenses typically include tuition, books, and supplies

The maximum credit of $2,000 is per tax return.

Ayesha timely filed her 2020 income tax return, but her negligent failure to report gambling income resulted in an additional tax liability of $20,000. What is Ayesha's negligence penalty? A) $1,000 B) $4,000 C) $15,000 D) $2,000

The negligence penalty is 20% of the deficiency due to the taxpayer's negligence. For a $20,000 tax deficiency, 20% results in a negligence penalty of $4,000.

Frank, a taxi driver, inadvertently fails to report approximately $1,200 of his tips received during the tax year. If the IRS imposes a penalty due to the underreported income, it would most likely impose a penalty equal to A) 75% of the deficiency. B) 50% of the deficiency. C) 20% of the deficiency. D) 20% of the deficiency plus 50% of the interest.

The negligence penalty is likely to be imposed, due to the failure to make a reasonable effort to comply with the requirements of the Code. This carries a penalty of 20% of the deficiency.

Larry and Paula are a married couple who file their federal income tax returns separately. They are both over 65 and still provide full support for a son who has been blind since birth. They live together and do not itemize. They alternate listing their son as a dependent, and it is Paula's turn this year. Paula will be required to file a federal income tax return if her gross income is at least which of the following amounts in 2022? A) $19,400 B) $12,950 C) $14,350 D) $20,800

The normal filing threshold for the MFS filing status is $12,950 in 2022. For married taxpayers over age 65, the threshold is raised by $1,400 per spouse. The additional blind deduction applies only to the taxpayers themselves, not their dependents. Tangentially, if the other MFS spouse itemizes, the filing threshold is reduced to $5. (IRS pub 501, 2022) Because Larry and Paula still live together, neither can file as head-of-household with a dependent.

Three years ago, Myla received a gift of 100 shares of public utility stock from her aunt. The fair market value of the stock on the date of the gift was $20 per share. Her aunt had purchased the stock six years earlier at $6 per share. Myla sold this stock for $24 per share last week. What was Myla's basis in the stock when she sold it? A) $20 per share B) $24 per share C) $6 per share D) $14 per share

The only time that the gifted asset takes the sale price as the basis is when the fair market value on the date of the gift is less than the donor's basis and the asset is sold at a price between the fair market value on the date of the gift and the donor's basis.

Claudia, who has an AGI of $40,000, wants to donate a painting of an ancestor who served in the American Revolution to the museum in her town that houses a collection of Revolution Era items. Her basis in the painting is $1,750, and it has a fair market value of $2,000. How much can she potentially deduct as a charitable contribution this year, assuming it is her only donation? A) $2,000 (It is related-use capital gain property.) B) $1,750 (It is related-use capital gain property, so she must use basis.) C) None (It is the portrait of a relative.) D) $600 (The museum is a 30% organization, so she must use FMV.)

The painting is related-use, capital gain property. Claudia may deduct an amount up to 50% of her AGI if she uses the basis of the painting and 30% of AGI if she uses FMV. As long as Claudia's AGI is greater than $6,667, she can deduct the FMV of the portrait.

Kris Swenson anticipates adjusted gross income of $100,000 for the current tax year. She is considering making a gift of a painting to the American Red Cross in the current tax year. Kris's basis in the painting is $35,000. The painting has a current fair market value of $50,000. Kris has owned the painting for 15 years. If Kris does gift the painting to the American Red Cross this year, what is the maximum allowable charitable deduction she can receive in the current tax year? A) $30,000 B) $20,000 C) $50,000 D) $35,000

The painting would be considered use-unrelated tangible personalty. The deduction for use-unrelated tangible personalty is limited to basis, with a 50% of AGI limitation. Thus, the current-year deduction is $35,000. If the painting had been donated to an art museum, for example, the contribution would be of use-related tangible personalty. Since the painting had been held for the long-term holding period, the deduction would have been $30,000 (long-term capital gain property to a 50% organization uses FMV with a 30% of AGI limitation) with a $20,000 carryforward.

Ruth and Doug divorced last year. They have two children, ages 7 and 9. Their divorce decree states that Ruth has custody of both children. There is no written agreement for listing the children as dependents on their returns. However, Doug provides 75% of the child support, amounting to $15,000 per year. Based on this information, which parent is entitled to show the children as dependents on their income tax returns?. A) Ruth, because she has custody and there is no written agreement that Doug could claim the children B) Doug, because he provides over half of the child support C) Ruth, because the court awarded her custody of the children D) Doug, because he provides at least $1,200 per year for the children's support and Ruth cannot prove she contributes more than this amount

The parent with custody for a greater portion of the year is treated as providing more than half of a child's support, where the child receives more than half of his total support for the year from either or both parents who are divorced.

Ruth and Doug divorced last year. They have two children ages seven and nine. Their divorce decree states that Ruth has custody of both children. There is no written agreement for listing the children as dependents on Ruth's or Doug's income tax returns. However, Doug provides 75% of the child support, amounting to $15,000 per year. Based on this information, which parent is entitled to list the children as dependents for income tax purposes? A) Doug, because he provides at least $1,200 per year for the children's support and Ruth cannot not prove she contributes more than this amount B) Ruth, because she has custody and there is no written agreement stating Doug could list the children on his return C) Ruth, because the court awarded her custody of the children D) Doug, because he provides over one-half of the child support

The parent with custody for a greater portion of the year is treated as providing more than one-half of a child's support. In these circumstances, however, Ruth could potentially sign IRS Form 8332 which would constitute a written agreement that would allow Doug to claim their two children as dependents on his tax return in the current year.

Francine and Marshall have three children: Bill, Curt, and Rachel. For 2022 1) Bill, age 11, has $1,250 of interest income. 2) Curt, age 13, has $2,950 of salary from a part-time job. 3) Rachel, age 19 and not a full-time student, has $5,100 of dividends and capital gains. Whose income is subject to application of the parents' marginal rate? A) Curt B) Bill and Rachel C) None of the children D) Bill

The parents' marginal rates apply to unearned income above $2,300 received by an individual under the age of 19 at the close of the tax year, if the individual is not a full-time student. Bill has unearned income under $2,300. Curt has earned income, so the parental rate does not apply. Rachel is 19 and is not a full-time student, therefore the parental rate does not apply.

Which of the following statements concerning the passive activity loss rules is NOT correct? A) The passive activity loss rules limitation is a permanent disallowance rule. B) The $25,000 offset allowance for the small real estate investor is not available to taxpayers whose AGI is $150,000 or more. C) Losses from one master limited partnership activity may only offset income from that particular activity; they cannot be used to offset income from any other passive activities. D) Losses from passive activities may not offset portfolio or active income except under limited circumstances.

The passive activity loss limitation is not a permanent disallowance rule. When a taxpayer disposes of his entire interest in a fully taxable transaction to an unrelated purchaser (not a related party), his suspended losses from that activity, including any losses incurred in the year of disposition, are generally deductible in full.

In the current year, Keith had passive losses of $19,000 from a real estate limited partnership purchased in 1985. He also had passive income of $7,000 from an oil and gas limited partnership. Both limited partnerships are not publicly traded. What is the total amount of passive losses that may be used to offset active, passive, and portfolio income in the current year? A) $9,400 B) $1,900 C) $8,200 D) $7,000

The passive loss ($19,000) is deductible, but only up to the amount of passive income ($7,000) in the same year; thus, the answer is $7,000.

Which one of the following items is included in the computation of total income on the Form 1040? A) Penalty on early withdrawal of savings B) Life insurance beneficiary proceeds C) Tax credit D) Sole proprietorship loss

The penalty on an early withdrawal of savings is an adjustment to income; a tax credit is taken from the calculated tax; and life insurance proceeds are not taxed.

John, an accountant, is considering forming an S corporation for his practice. He will be the sole employee of the corporation. Which of the following statements accurately describes the income tax consequences of the proposed arrangement? A) The corporation would not be considered a PSC; therefore, the income would be subject to the graduated corporate income tax rates. B) The corporation would be considered a PSC, subject to a flat 21% tax rate. C) The corporation would be considered a PSC, subject to graduated tax rates. D) The corporation would not be considered a personal service corporation (PSC), and the income would be subject to John's personal income tax rates.

The personal service corporation (PSC) classification for a C corporation was effectively repealed with the Tax Cuts and Jobs Act (TCJA). S corporations do not have corporate income tax rates. As an S corporation, all income would flow through to John to be taxed at his individual rates.

Which one of the following is CORRECT regarding the Coverdell Education Savings Account? A) Distributions may be tax free even if made for K-12 expenses. B) Room and board may be covered with a tax-free distribution only if the student is full-time. C) Deductible contributions of up to $2,000 may be made per beneficiary. D) Distributions may be tax free only if made for a full-time student.

The predominant benefit of the Coverdell ESA is distributions may also be used to pay for K-12 expenses. This is unlike the 529 plan which is designed primarily to pay for college expenses (Note: a limited amount of $10,000 may now be withdrawn from a 529 for K-12 expenses).

Which of the following statements regarding the use of life insurance inside a retirement plan is CORRECT? A) The premiums paid are a taxable benefit to the employee. B) The premiums paid are a taxable benefit to the employer. C) The premiums paid are NOT a taxable benefit to the employee. D) If the employee dies prematurely, the survivors will receive no benefits.

The premiums paid are a taxable benefit to the employee. The main benefit to the employee is in the event of their premature demise, their survivors will still receive ample retirement benefits.

John owns a classic automobile that had a cost basis of $32,000. John paid $38,000 to have the automobile fully restored. John sells the automobile through an installment sale for $100,000. John is to receive a $25,000 down payment in the current year, and $15,000 per year for five years, beginning this year. What amount of gain must John recognize during the current year? A) $12,000 B) $4,500 C) $7,500 D) $12,800

The profit on the sale was $30,000 divided by the $100,000 contract price, which equals a 30% gross profit percentage. This is multiplied by the $40,000 of payments received during the year to calculate the amount of gain recognized, $12,000. The $38,000 of restoration costs are capitalized, added to basis, to give us the $70,000 basis.

Piotr and Lisa reside in a New York and recently became engaged. Piotr is a Russian immigrant and U.S. resident alien with a considerable fortune in oil stock. Lisa has never been married. While vacationing in Crimea, the couple drinks several bottles of wine. Piotr then presents Lisa with a diamond ring and a premarital agreement drafted by a local attorney. Which of the following are characteristics of a valid and enforceable premarital agreement? 1.) International law applies to premarital agreements. 2.) The purpose of the premarital agreement is to limit the presumed effect of the marriage on property acquired prior to or during the marriage. 3.) The signer's health is not a factor at the time of signing. 4.) If one party alleges duress or coercion at the time of the agreement's implementation, it is likely that the agreement will not be binding. A) III and IV B) II and III C) II and IV D) I and II

The purpose of the premarital agreement is to limit the presumed effect of the marriage on property acquired prior to or during the marriage. There are no international treaties to set standards for such agreements. A U.S. domiciled court may not recognize it as valid. The signer's health and mental state can also be a factor in determining the validity of an agreement. If one party alleges duress or coercion at the time of the agreement's implementation, it is likely that the agreement will not be binding. Premarital agreements are uncommon in first marriages, but very common in second marriages especially when there are children from prior marriages. A premarital agreement is, however, void if it contravenes public policy.

Mike and Molly reside in a community property state and recently became engaged. Each has three children from a previous marriage. Which of the following are characteristics of a valid and enforceable premarital agreement? 1.) It is very common for first marriages. 2.) The purpose of the premarital agreement is to limit the presumed effect of the marriage on property acquired prior to or during the marriage. 3.) It may contravene public policy. 4.) If one party alleges duress or coercion at the time of the agreement's implementation, it is likely that the agreement will not be binding. A) I and II B) III and IV C) II and III D) II and IV

The purpose of the premarital agreement is to limit the presumed effect of the marriage on property acquired prior to, or during, the marriage. If one party alleges duress or coercion at the time of the agreement's implementation, it is likely that the agreement will not be binding. Premarital agreements are uncommon in first marriages, but very common in second marriages especially when there are children from prior marriages. A premarital agreement is, however, void if it contravenes public policy.

Sheila, a single taxpayer, has taxable income of $480,000. Included in the taxable income is $50,000 of qualified dividends. At what rate(s) will her qualified dividends be taxed? A) 25% B) 15% and 20% C) 15% only D) 20% only

The qualified dividends straddle the $459,750 breakpoint (for 2022). Thus, a portion fall into the $41,675 to $459,750 range and are taxed at 15%. The dividends above the $459,750 breakpoint are taxed at 20%.

Your client is contemplating the exchange of two parcels of investment land for two similar parcels. Given the following details of the proposed transactions, compute the amount of recognized gain and loss, if any, on both parcels if your client does the exchanges. Parcel A: There were 10 acres of land acquired 15 years ago with a current basis of $50,000. In exchange, your client will receive 8 acres of land (FMV $80,000) and $20,000 of cash. Parcel B: There were 20 acres of land acquired 2 years ago with a current basis of $100,000. In exchange, your client will receive 12 acres of land (FMV $75,000) and $10,000 of cash. A) Parcel A recognized gain: $20,000; Parcel B recognized loss: $10,000 B) Parcel A recognized gain: $20,000; Parcel B recognized loss: $15,000 C) Parcel A recognized gain: $20,000; Parcel B recognized loss: $0 D) Parcel A recognized gain: $50,000; Parcel B recognized loss: $10,000

The realized gain in Parcel A is $50,000 and the recognized gain (the lesser of the gain realized or the boot received) is $20,000. The realized loss in Parcel B is $15,000. However, there is no loss recognized (deducted) in a like-kind exchange.

Which one of the following is a taxation function of the economic objective? A) Renovation of a historic home B) Reduction of taxes to stimulate the economy C) Excluding life insurance proceeds from taxation D) Charitable deduction

The reduction of taxes in order to stimulate the economy is due to the economic objective. Social objectives of the federal taxation system include the charitable deduction, excluding life insurance proceeds from taxation, and renovation of a historic home.

Lisa and William are married taxpayers who file as married filing separately (MFS) for income tax purposes as it is advantageous for their tax positions; although the couple does live together. Lisa has rental property she inherited from her uncle that will generate a loss this year of $14,000. Lisa meets the active participation standard. Lisa's AGI is $65,000 and William's AGI is $80,000. How much is Lisa's allowed passive activity loss this year? A) $12,500 B) $0 C) $14,000 D) $5,000

The rental real estate loss allowance is not available to taxpayers who file as MFS and have lived together at any time during the tax year.

Mary reported an income tax liability of $50,000 (on AGI of $300,000) on her tax return for 2022. This year, Mary expects to have an income tax liability of $60,000. She also has estimated that the amount of income tax withheld from her wages will total $35,000. What minimum amount of estimated tax payments must Mary pay (in equal quarterly installments) for 2022? A) $20,000 B) $0 C) $15,000 D) $19,000

The required annual payment (since the prior-year AGI is greater than $150,000) is the lesser of: - 90% of the current year tax ($60,000 x .90 = $54,000), OR - 110% of the tax shown in the preceding year ($50,000 x 1.10 = $55,000) - reduced by the withholding of $35,000. Thus, $54,000 reduced by the withholding of $35,000 equals $19,000.

Bob and Bonnie were divorced in 2016. As a result of a court order, Bob pays Bonnie $700 per month in alimony. He makes each month's payment with a money order. Earlier this year, Bonnie moved in with Bob, and they now share a two-bedroom apartment. Which of the following statements is accurate concerning the alimony payments by Bob? A) The payments are not deductible because the payments must be in cash. B) The payments are not deductible because the taxpayers are living together at the time of payment. C) The payments are deductible because they are equal each year of the agreement. D) The payments are deductible because they are being made as a result of a court order.

The requirement that the payments be in cash means that the payments may not be in the form of property. Payments are not necessarily deductible just because they are a result of a court order; there are other requirements that must be met as well. One of those requirements to have qualifying alimony is that the taxpayers must not be living together at the time of payment. The payments do not need to be equal each year. However, unequal and declining payments can trigger the alimony recapture (excess front-loading) rules.

Bill's 2021 income tax return, which was for a full year, showed an AGI of $140,000 and an income tax liability of $32,100. He estimates his 2022 income tax to be $38,000 and his total wage withholding to be $5,000. What minimum amount of estimated tax payments must Bill pay for 2022? A) $34,200 B) $27,100 C) $41,800 D) $32,100

The safe harbor for avoiding the underpayment penalty is the lesser of 90% of the current year tax liability or 100% of the prior year tax liability (110% if the prior year's AGI was over $150,000). 100% of the prior year tax liability is $32,100. 90% of the current year tax liability of $38,000 is $34,200. The smaller of these numbers, reduced by the $5,000 withholding, equals $27,100. To avoid penalties, the estimated tax should be paid to the tax authorities in quarterly installments.

For a taxpayer with an AGI in excess of $150,000 for the prior tax year ($75,000 if married filing separately), the estimated tax penalty safe harbor is A) 90% of the prior year's tax liability or 110% of the current year's tax liability. B) 90% of the current year's tax liability or 110% of the prior year's tax liability. C) 120% of the prior year's tax liability or 80% of the current year's tax liability.

The safe harbor is 90% of the current year's tax liability or 110% of the prior year's tax liability if the taxpayer's prior-year AGI exceeded $150,000.

Steve and Allison Parker, a married couple in their 40s, file a joint return. They earned combined salaries of $185,000. They received dividend and interest income of $860 from mutual funds. They have allowable itemized deductions of $14,000. They have net capital losses of $5,200. They have two children, ages 12 and 14. What is their taxable income for the 2022 tax year? A) $168,860 B) $182,860 C) $156,960 D) $156,660

The salaries combined with the income from the investments total $185,860. This is reduced by the $3,000 net capital loss to leave an AGI of $182,860. Remember that only $3,000 of net capital loss may be deducted in a given tax year. The AGI is then reduced by the greater of the itemized deductions ($14,000) or the standard deduction ($25,900 in 2022).

Lowell and Thelma are married and will file a joint return for the current tax year. They are contributing to their respective 401(k) plans through their employers. They have provided you with the following information. Lowell's salary (after 401(k) contributions)$75,000 Thelma's salary (after 401(k) contributions)$50,000 Alimony payments to Lowell's ex-wife$24,000 Net long-term capital loss$7,000 Property taxes$2,000 IRA contribution—Lowell$6,000 IRA contribution—Thelma$6,000 Lowell's divorce was finalized in 2015. Based on the information given, what is the couple's adjusted gross income for the current tax year? A) $118,000 B) $86,000 C) $98,000 D) $82,000

The salaries of $125,000 reduced by the $24,000 of alimony payments equals $101,000. This is further reduced by $3,000 of net capital losses. Remember that only $3,000 of net capital losses are deductible in a given year, with an indefinite carryforward of the excess. The $12,000 of IRA contributions is also deductible. Even though both spouses are active participants in company-maintained retirement plans, their MAGI (AGI without the IRA contributions) is only $98,000. This is under $109,000 (for 2022)—the beginning of the phaseout range for married couples filing jointly, when both spouses are active participants. The property taxes are an itemized deduction, and do not affect the AGI.

A client sold an apartment building last year for $100,000, paying a sales commission of $5,000 plus $2,500 in closing costs. The building originally cost $80,000 20 years ago. Total straight-line depreciation of $40,000 had been taken. The building had a mortgage of $60,000 that was assumed by the buyer. The client is in the 24% marginal income tax bracket. What is the seller's adjusted cost basis? A) $32,500 B) $52,500 C) $40,000 D) $37,500

The seller's adjusted basis is the $80,000 purchase price, decreased by the $40,000 of straight-line depreciation. The mortgage has no bearing on the basis of the property.

Which of the following forms of business would file a Schedule C to report the income or loss from the business? A) C corporation B) S corporation C) Sole proprietorship D) Limited partnership

The sole proprietorship is the business that files a Schedule C with the individual's Form 1040. The limited partnership files a Form 1065, and the C corporation files a Form 1120. The S corporation files an 1120-S. Note that a single-member LLC would also file a Schedule C, as it would be treated as a disregarded entity for income tax purposes.

Teddy, age 12, has interest income of $1,275. He also has earned income from an after-school job that totals $13,000. Teddy is eligible to be treated as a dependent on his parents' return. What is the amount of Teddy's standard deduction for 2022? A) $1,275 B) $1,150 C) $12,950 D) $13,400

The standard deduction for an individual eligible to be claimed, or treated, as a dependent is the greater of the limited standard deduction of $1,150 or the amount of earned income plus $400, not to exceed the full standard deduction amount of $12,950 (for 2022). $13,000 + $400 = $13,400, but the deduction is limited to the full standard deduction amount of $12,950.

In 2022, which of the following is a method in calculating total self-employment tax, where net income from self-employment is above the taxable wage base? Calculate self-employment income; multiply by 0.9235 (1 − 0.0765); subtract the taxable wage base, and from that, multiply the excess over the taxable wage base by 2.9% (Medicare portion of the tax). Multiply the taxable wage base by 15.3%, and add the results of the previous two steps together to arrive at the total self-employment tax. Calculate self-employment income; multiply by 0.9235 (1 − 0.0765); and multiply the result by 15.3%. Calculate self-employment income; multiply by 14.13%. Calculate self-employment income; multiply by 15.3%; multiply the excess over the taxable wage base by 2.9% (Medicare portion of the tax); and add the results of the previous two steps together to arrive at the total self-employment tax. A) II and III B) I only C) I and II D) I and IV

The steps to calculate self-employment tax for 2022 where net income from self-employment is above the taxable wage base are as follows: Step 1: Calculate self-employment income. Step 2: Subtract 7.65% or multiply by 0.9235 (1 − 0.0765). Step 3: From Step 2, subtract the taxable wage base and multiply the excess over the taxable wage base by 2.9% (Medicare portion of the tax). Step 4: Multiply the taxable wage base by 15.3%. Step 5: Add the results of Steps 3 and 4 together to arrive at the total self-employment tax.

Which of these rules or doctrines may limit the availability of income tax benefits from a particular investment? 1.) Tax conduit 2.) The substantial economic effect doctrine 3.) The at-risk rule 4.) The passive activity loss rule A) I and IV B) I and III C) II, III, and IV D) II and III

The substantial economic effect doctrine limits the ability to use special allocations in a partnership. The at-risk rule limits the ability to use leverage by attacking the use of nonrecourse financing. The passive activity loss rule limits the ability to deduct losses from activities in which the taxpayer does not materially participate.

The substitute basis of a qualifying asset received in a like-kind exchange is the asset's A) fair market value reduced by the gain realized but not recognized. B) fair market value increased by the gain realized but not recognized. C) basis increased by the gain realized but not recognized. D) basis reduced by the gain realized but not recognized.

The substitute basis of a qualifying asset received in a like-kind exchange is the asset's fair market value reduced by the deferred gain (the gain realized but not recognized). This deferred gain reduces the basis of the acquired asset, such that when that asset is sold, there is a larger gain recognized.

Willis has an active participation rental real estate activity. Last year, he had losses of $15,000 from the active participation real estate, and his AGI was $225,000. The $15,000 of losses were suspended due to his AGI. In the current year, Willis has an AGI of $90,000 and $6,000 of current losses from his real estate rental activity. What amount of loss may Willis deduct in the current year? A) $25,000 B) $15,000 C) $6,000 D) $21,000

The suspended losses of $15,000 from a former year plus the current losses of $6,000 equals $21,000.

Bob passed away during the current year. He had suspended losses from a limited partnership activity of $25,000. Bob's basis in the partnership was $1,000 and the fair market value at the time of his death was $18,000. What amount of passive losses, if any, is deductible on Bob's final income tax return? A) $14,000 B) $17,000 C) $0 D) $8,000

The suspended passive losses are "freed up" and deductible only to the extent that the losses exceed the step-up in basis. In this situation, the step-up in basis equals $17,000 (from $1,000 to $18,000). The losses of $25,000 exceed the step-up amount by $8,000.

Which of the following forms of business could NOT be a direct participation program (tax conduit)? A) Closely held C corporation B) S corporation C) Limited liability company (LLC) D) Limited liability partnership (LLP)

The tax advantages provided by direct participation programs are founded upon the principle that most types of business organizations function as tax conduits. Therefore, a closely held C corporation cannot qualify, as there is no flow through of gains and losses.

Ed deducted $6,500 in medical expenses as an itemized deduction on his prior-year tax return. During the current year, he receives a reimbursement from his insurance company of $5,000. Which of the following rules or doctrines may cause the reimbursement to be taxable to Ed? A) Tax benefit rule B) Assignment of income doctrine C) Step transaction doctrine D) Substance over form doctrine

The tax benefit rule converts otherwise nontaxable receipts into taxable income. The most common example is when a taxpayer is reimbursed in a subsequent year for medical expenses paid and deducted in a previous year.

Melanie has city income taxes of $800 and state income taxes of $1,000 withheld from her paycheck last year. She deducted these amounts on her income tax return for last year. This year, Melanie received a refund from the state for $200 due to an overpayment of the state tax. What is the proper tax treatment of the refund this year? A) The $1,000 deducted last year must be included as income in the current year. B) Melanie need not take any action because refunds are not income. C) Melanie must reduce the amount she deducts for state income taxes in the current year by the $200 refund. D) Melanie must include $200 in her gross income this year.

The tax benefit rule or doctrine converts an otherwise nontaxable receipt—in this case, the $200 state income tax refund—into taxable income. Melanie received a tax benefit by deducting the state income taxes the prior year, and any portion refunded to her would be taxable in the year it is received.

Which one of the following provides the greatest federal revenue from IRS taxation? A) Gift taxes B) Corporate income taxes C) Individual income taxes D) Estate taxes

The three main sources of federal tax revenue are individual income taxes, corporate income taxes, and payroll taxes. Individual income tax accounts for approximately 40% of the total tax revenue collected by the federal government. The federal estate tax and gift tax actually compose only a small percentage of annual tax revenues.

Which one of the following is NOT a type of tax audit performed by the IRS? A) An office audit B) A field audit C) A home audit D) A correspondence audit

The three types of tax audit performed by the IRS are as follows: a correspondence audit, which is usually performed through the mail because the disputed tax issue is minor; an office audit, which is usually restricted in scope to a specific item or items and is performed at the IRS office by an office auditor; and a field audit, which is an examination of numerous items and is usually performed on the premises of the taxpayer (such as a business office) by a revenue agent.

Janet and Bruce Robinson, both age 43, are married taxpayers filing jointly. They have itemized deductions consisting of the following: Home mortgage interest$19,500State income taxes$8,700Property taxes$5,200Charitable contributions$6,200Tax return preparation fee$895Unreimbursed employee business expenses$2,100Unreimbursed medical expenses$18,460 Their AGI for 2022 is $466,000. What is the amount of their allowable itemized deductions? A) $35,700 B) $42,595 C) $37,800 D) $39,600

The total itemized deduction amount is $35,700. Note that the tax preparation fee and the unreimbursed employee business expenses are not deductible. The medical expenses are deductible only to the extent that they exceed 7.5% of AGI for 2022, which they do not. The deduction for the state income taxes and the property taxes is capped at $10,000. Taxes of $10,000, mortgage interest of $19,500, and charitable contributions of $6,200 total $35,700.

Danielle created a revocable trust for her two minor sons. She named her bank as trustee. The trust property earned $30,000 in the first year and had taxable income of $28,000 after deducting expenses. This income was left to accumulate for future distributions to be made to each son equally when the youngest son attains age 18. To which of the following will the income of the trust be taxable? A) The oldest son after attaining age 18, then the sons equally after the youngest son attains age 18 B) Danielle C) Both sons equally D) The trust

The trust income will be taxed to the grantor, as the trust is revocable. A revocable trust is treated as a grantor trust.

Danielle created a revocable trust for her two minor sons. She named her bank as trustee. The trust property earned $30,000 in the first year and had taxable income of $28,000 after deducting expenses. This income was left to accumulate for future distributions to be made to each son equally when the youngest son attains age 18. To which of the following will the income of the trust be taxable? A) The oldest son after attaining age 18, then the sons equally after the youngest son attains age 18 B) Danielle C) Both sons equally D) The trust

The trust income will be taxed to the grantor, as the trust is revocable. A revocable trust is treated as a grantor trust.

Your clients, Mike and Elizabeth, have a 17-year-old dependent son, Josh, who has a part-time job during the summer months. They have heard of the kiddie tax but are unsure of how it would impact the taxation of Josh's summer wages. Which of the following statements is CORRECT regarding the taxation of Josh's income for 2022? 1. Josh has a limited standard deduction (up to $1,150) available. 2. Josh has a full standard deduction (up to $12,950) available. 3. Any income in excess of the available standard deduction and $1,150 is taxable at Mike and Elizabeth's marginal income tax rate. 4. The income is not subject to the kiddie tax rules because it is earned income. A) II and IV B) I and III C) II and III D) I only

The unearned income rules (the kiddie tax rules) do not apply to earned income. Thus, Josh may utilize up to the full standard deduction ($12,950 for 2022) against earned income.

Ann Hamilton owns 500 shares in the XYZ S&P 500 Index Fund. The basis of her investment in this fund is $4,500, while the fair market value is only $2,000. She wants to sell her shares to "lock in" the $2,500 loss, but she is considering buying 500 shares of the GRC Small-Cap Index ETF the following week because she believes that the value is going to increase significantly over a longer period. As her planner, what can you accurately tell Ann about this scenario? A) She should wait a minimum of 61 days after the sale to repurchase the shares so that the loss may be recognized. B) The loss would be a fully deductible capital loss. C) If the loss were disallowed, the basis in the newly acquired shares would be decreased by the disallowed loss. D) The basis in the newly acquired shares would be the amount paid for those shares, increased by the $2,500 disallowed loss.

The wash sale rule disallows a loss if substantially identical securities are purchased prior to 30 days after the sale that resulted in the loss. The basis of the acquired securities is increased by the amount of the disallowed loss. The S&P 500 mutual fund should not be substantially identical to the small-cap ETF because the funds track very different indices and because of the difference in the way ETFs trade compared with mutual funds.

Hayley owns 500 shares in the XYZ Growth Mutual Fund. The basis of her investment in this fund is $3,700, while the fair market value is only $1,200. She wants to sell her shares to lock in the $2,500 loss, but she is considering buying 500 shares of the same fund the following week because she believes that the value is going to increase significantly over a longer period. As her planner, what can you accurately tell her about this scenario? A) She should wait a minimum of 60 days after the sale to repurchase the shares so that the loss may be recognized. B) If she purchased shares in the ABC Value Mutual Fund within 30 days after the sale of the old shares, the $2,500 loss would be disallowed. C) The basis in the newly acquired shares would be the amount paid for those shares, increased by the $2,500 disallowed loss. D) The loss would be a fully deductible capital loss.

The wash sale rule disallows a loss if substantially identical securities are purchased within a period of 30 days before to 30 days after the sale that resulted in the loss. The basis of the acquired securities is increased by the amount of the disallowed loss. The mutual fund with a different orientation (growth versus value) should not be considered substantially identical.

Eleven months ago, Lynnette received 1,000 shares of stock from her uncle, Joseph. Joseph purchased the stock eight years ago for $12 per share. The fair market value on the date of the gift to Lynnette was $9 per share, and she sold the stock today for $5 per share. What is the amount and character of Lynnette's loss from the sale of the stock? A) $3,000 long-term capital loss B) $4,000 short-term capital loss C) $3,000 short-term capital loss D) $7,000 long-term capital loss

There are two components to this question. What is the basis, and is there tacking of the holding period? When the fair market value on the date of the gift is less than the donor's basis in the asset, the donee's basis in the asset for purposes of determining a loss is the asset's FMV on the date of the gift. In this situation, the $9 per-share value on the date of the gift would be Lynnette's basis. The next issue is the "tacking" of the holding period. In a situation where the donee uses the FMV as the basis, there is no tacking of the holding period. In this situation, Lynnette used the FMV; thus, she uses her own holding period of 11 months. If the donee uses the donor's basis, then the holding period is tacked. In other words, the donor's holding period is added to ("tacked") the donee's holding period.

Suzy, a single taxpayer, sells her residential rental property in the current year for $400,000. Suzy acquired the property in ten years ago for $300,000 and has been depreciating it using the straight-line method for realty under MACRS. Assume the amount of depreciation taken is $114,000. Suzy has taxable income of $190,000 and is in the 32% marginal income tax bracket. What is the amount and character of the gain recognized as a result of the sale? A) $114,000 gain taxed at 28%, $100,000 gain taxed at 15% B) $214,000 gain taxed at 25% C) $114,000 gain taxed at 25%, $100,000 gain taxed at 15% D) $214,000 gain taxed at 15%

There is $114,000 of unrecaptured Section 1250 income, creating 25% long-term capital gain; and $100,000 of regular long-term capital gain (at taxable income of $190,000, it is 15% rate). Calculate the gain realized and recognized. In this example, we have total gain recognized (Section 1231 gain) of $214,000, comprised of $114,000 of unrecaptured Section 1250 income and taxed at a maximum rate of 25%, and $100,000 of regular long-term capital gain, eligible for the maximum 15% or 20% long-term capital gain rates. The gain created by the straight-line depreciation on realty is unrecaptured Section 1250 income, subject to a maximum rate of 25%. The gain created by actual appreciation of the asset is regular long-term capital gain under Section 1231.

Kevin and Karen are married taxpayers filing a joint tax return. In 2022, their AGI is $320,000, and their net long-term capital gain income (included in the AGI) is $90,000. They have investment interest expense of $4,000 and state and local income taxes attributable to the investment income of $6,000. What is the amount of Medicare contribution tax that they must pay? A) $2,280 B) $3,040 C) $2,660 D) $3,420

They will pay the 3.8% Medicare contribution tax on $70,000. This is the lesser of the net investment income ($80,000) or the AGI in excess of the threshold amount ($320,000 - $250,000, or $70,000). The net investment income is the investment income of $90,000, reduced by the investment expenses of $10,000. In this situation, the $70,000 excess of the AGI over the threshold amount is subject to the Medicare contribution tax. Kevin and Karen will pay a $2,660 Medicare contribution tax (3.8% on $70,000).

Larry and Mary are married taxpayers filing a joint tax return. In 2022, their AGI is $360,000, and their investment income (included in the AGI) is $100,000. They have investment interest expense of $7,000 and state income taxes attributable to the investment income of $5,000. What is the amount of Medicare contribution tax that they must pay? A) $3,800 B) $4,180 C) $3,534 D) $3,344

They will pay the 3.8% Medicare contribution tax on $88,000. This is the lesser of the net investment income ($100,000) or the AGI in excess of the threshold amount ($360,000 - $250,000, or $110,000). The net investment income is the investment income of $100,000, reduced by the allowable investment expenses of $12,000. In this situation, all $88,000 of the net investment income is subject to the Medicare contribution tax. Larry and Mary will pay a $3,344 Medicare contribution tax (3.8% on $88,000).

Ron, age 43, and Sandy, age 41, are married with two children: Michael, age 12, and Victoria, age 8, who has been blind since her birth. Ron is an architect and general partner with XYZ partnership. Sandy is self-employed as an attorney and works out of a home office. Her home office is exclusively and regularly used for business, and the home office is her principal place of business. Their information for the tax year 2022 is as follows: Adjusted gross income: $217,300 Itemized deductions (including qualified residential mortgage interest, taxes paid, and charitable contributions): $33,000 Early in the current year, Sandy's father died. Sandy is the sole beneficiary of her father's entire estate. The estate is presently in the probate process. Sandy's mother, Lisa, age 68, has moved in with them but provides her own support. She was married to Sandy's father when he died earlier this year. This is Ron's second marriage. He makes monthly support payments to his former spouse and his daughter. Because both Ron and Sandy are considered to be self-employed, they make quarterly estimated tax payments each year to cover both their income tax and self-employment tax obligations. Assume Ron's ex-spouse successfully petitions the court to require Ron pay their daughter's private school tuition of $1,000 per month for the nine months of the school year. How much can Ron deduct on his income tax return for these payments? A) More information is needed B) $4,500 C) $0 D) $9,000

This is additional child support and is not tax deductible.

Tom and Jeanette are married taxpayers filing jointly. Jeanette is the owner of a chain of successful car washes, from which she receives a substantial portion of her income in cash. Last year, the car washes had sales of $160,000, but Jeanette intentionally failed to report $12,000 of the sales that she received in cash. Which one of the following penalties is the Internal Revenue Service most likely to apply if it determines that Jeanette underreported her income? A) Underreporting penalty B) Substantial understatement penalty C) Civil fraud penalty D) Negligence penalty

This is essentially taxpayer fraud that does not rise to the level of criminal fraud (most of the income was declared), but is willful. If imposed, the penalty is 75% of the portion of tax underpayment attributable to fraud.

When the IRS imposes penalties on federal income tax returns, the 75% penalty that is imposed on the tax underpayment that is attributable to fraud is said to be due to A) criminal fraud. B) negligence. C) civil fraud. D) a frivolous return.

This is essentially taxpayer fraud that does not rise to the level of criminal fraud. If imposed, the penalty is 75% of the portion of tax underpayment attributable to fraud.

Nate and Teddi own a house at the beach. They rented the house to unrelated parties for eight full weeks during the current year. Nate and Teddi used the house 16 days for their vacation during the year. After properly dividing the expenses between rental and personal use, a rental loss from the house was incurred as follows: Gross rental income $6,400Less mortgage interest and property taxes($7,000)Less other allocated expenses (operating expenses of $400 and depreciation of $600) ($1,000) Net rental loss ($1,600) What is the CORRECT tax treatment of the rental income and expenses on Nate and Teddi's joint income tax return for the current year? A) The $7,000 rental portion of mortgage interest and property taxes can be deducted. B) Because the house was only 20% personally used by Nate and Teddi, all expenses allocated to personal use may be deducted. C) The other allocated rental expenses deduction is limited to the gross rental income in excess of the deductions for interest and taxes allocated to the rental use. D) A $1,600 loss should be reported.

This is mixed-use property. The expenses must be allocated between rental use and personal use. The residence test is met—the personal use exceeds the greater of 14 days or 10% of the number of rental days. The rental expenses are only deductible to the extent of any rental income received. In this case, because mortgage interest and property taxes exceed the rental income, none of the other allocated expenses may be deducted, and only $6,400 of the allocated mortgage interest and taxes may be deducted against rental income. The remaining unused expenses may be carried forward until there is sufficient rental income to offset.

Assume that for each of the next five years, your client, Dan, will have the following: Active income: $100,000 Investment income: $50,000 Passive income: $0 Passive loss (from nonpublicly traded partnership): $25,000 Dan is considering an investment in a nonpublicly traded partnership that requires a $100,000 initial investment and will generate cash flow (pretax passive income) of $12,500 per year at the end of each year for the next five years. Upon liquidation at the end of the fifth year, Dan will receive a total of $100,000 after taxes. Dan's after-tax rate of return from other investments is 10% and his combined federal and state marginal tax bracket is 28%. What is Dan's net present value on the investment? A) ($2,523) B) ($15,023) C) $21,977 D) $9,477

This problem requires the calculation of the net present value of the cash flows as compared to the $100,000 initial cash outlay. The net present value of the $12,500 received at the end of each of the first four years is first calculated. The present value of these four payments is $39,623. The calculation of present value of the amount received at the end of year five of $112,500 ($12,500 + $100,000) using a 10% rate yields $69,854. The $69,854 combined with the $39,623 results in a combined present value of all payments of $109,477. Subtracting the initial cash outlay of $100,000 results in a net present value of $9,477. Note that no tax liability would result from the cash flows as they would be "sheltered" by the passive losses.

Which one of the following reflects the CORRECT sequence of steps in the tax calculation process? A) Total income minus adjustments to income equals AGI. B) Total income minus standard or itemized deduction(s) equals AGI. C) AGI minus adjustments to income equals federal taxable income. D) Calculate federal tax on total income.

Total (gross) income minus adjustments to income equals adjusted gross income (AGI). AGI minus standard or itemized deduction(s) equals federal taxable income.

Tim Jones is single, 21 years old, and in his third year of college. He has an AGI of $35,000 and receives no support from his parents. The college is a Title IV institution where students are eligible to receive federal financial aid, and Tim is pursuing an undergraduate degree in criminal justice. When Tim was 13, his parents established a Uniform Transfers to Minors Act (UTMA) for him, and funded it with EE savings bonds. When Tim was a freshman, he was convicted of a felony drug possession charge. Which one of the following is CORRECT regarding Tim's situation? A) Tim qualifies for the American Opportunity Tax Credit. B) Tim could use both the American Opportunity Tax Credit and the Lifetime Learning Credit in the same year. C) Tim may redeem the EE bonds potentially tax free if the proceeds are used for his qualifying education expenses. D) Tim qualifies for the Lifetime Learning Credit.

Tim qualifies for the Lifetime Learning Credit. His AGI is under the phaseout range. He is pursuing a degree at an eligible institution. The felony drug conviction would preclude the use of the American Opportunity Tax Credit but not the Lifetime Learning Credit. There is no exclusion available for EE bonds unless they are held by the individual who purchases the bonds or unless they are held jointly with a spouse. A bond that has been gifted to another taxpayer does not qualify for the exclusion. The American Opportunity Tax Credit and the Lifetime Learning Credit may not be claimed in the same year for the same student.

Which of the following are characteristics of a valid and enforceable premarital agreement? It may be orally executed by the parties that are affected. There should be a full and complete disclosure of each party's net worth prior to signing. It may be used to regulate an award of alimony upon divorce of the parties. There should be a written agreement with the willingly executed signatures of both parties. A) II and III B) II and IV C) III and IV D) I and II

To be valid, a premarital agreement must be in writing and contain a complete disclosure of each party's financial situation. It may not be used to regulate an award of alimony.

Which of the following is necessary to include in a premarital agreement? A) Financial resources and net worth of the more wealthy party B) Financial resources and net worth of both parties C) Financial resources and net worth of the less wealthy party D) Intentions to facilitate a divorce

To be valid, a premarital agreement must be in writing and contain a complete disclosure of each party's financial situation. It may not be used to regulate an award of alimony.

Which of these is NOT a requirement for claiming a home office expense deduction? A) The use of the home office must be exclusive to the business. B) The trade or business must be the wholesale selling of products or services. C) The home office must be used on a regular basis. D) The home office generally must be the principal place of business.

To claim a home office deduction, the office must be used regularly and exclusively as the principal place of business (or for storing supplies, as a day-care facility, or as a place where administrative or management activities take place, if there is no other fixed location where those activities may occur).

Sandra and Colby, a married couple, ask you to explain how taxable income is calculated after adjusted gross income has been determined. Which of the following is a deduction from adjusted gross income (AGI) to arrive at taxable income? 1.Additional standard deduction 2.Itemized deductions 3.Exclusions 4.Tax credits A) II and III B) IV only C) I, II, III, and IV D) I and II

To compute the taxable income, we subtract the greater of the standard deduction (including the additional standard deduction for elderly or blind) or the itemized deductions from AGI. We also subtract the qualified business income (QBI) deduction, if any, to arrive at the taxable income. Exclusions are not reported on the return, so they don't need to be subtracted. Tax credits are deducted from the tax liability.

Lisa owns a fourplex in which she lives and rents out two of the other three units for $1,200 per month each. Her parents, who are both age 70 and retired, live in the fourth unit and do not pay Lisa any rent. Residents pay their own utilities. Lisa has asked her financial planner how this arrangement with her parents will affect her income tax return. What should her planner tell her? A) Because the parents are not the planner's clients, a determination cannot be made. B) The $14,400 of forgone rent constitutes support, and Lisa may use the head of household filing status. C) The planner needs to know how much this rent makes up the total support of her parents and if any other amounts are paid for their care and support by Lisa before making any recommendations. D) Living rent-free with their daughter does not mean she is providing any support to them.

To make a determination on whether or not Lisa's parents are her dependents, the planner needs complete information of how much Lisa has paid, if anything, in addition to the forgone rent for their support and what percentage that is of her parents' total support.

Todd is employed at Wow Industries as an accountant. His employer deducted $8,500 from his paycheck in 2022 for federal income taxes. Todd also has a side practice for which he paid another $4,300 to the IRS in estimated federal income taxes for 2022. When he filed his return, he had a tax liability of $11,600 before a child and dependent care credit of $400. Which of the following statements is CORRECT? A) Because the child and dependent care credit is a nonrefundable credit, Todd's refund is $1,200. B) Todd's refund is $5,200 for 2022. C) Todd has an income tax refund of $1,600 for 2022. D) He cannot receive a refund in 2022.

Todd's refund is $1,600 [$12,800 in total tax deposits - ($11,600 tax liability - $400 child and dependent care credit)]. The child and dependent care credit only reduces Todd's tax liability to $11,200 and he has deposited $12,800. This entitles him to a refund of his excess tax payments of $1,600.

The following summarizes several financial events in the life of George during the current tax year. Received $100,000 from a life insurance policy due to the death of his brother Had gambling winnings of $45,000, while incurring gambling losses of $20,000 Received net royalties of $10,000 from an oil and gas investment Received $5,000 of unemployment compensation Had job-related moving expenses of $4,000 Contributed $6,000 to an IRA Assuming George is NOT a professional gambler, what is his total income for the current tax year? A) $60,000 B) $45,500 C) $155,000 D) $34,500

Total income is basically the starting point of the income tax calculation. The gambling winnings of $45,000, the unemployment compensation of $5,000, and the royalties of $10,000 are all included in income. Gambling losses are an itemized deduction, to the extent of gambling winnings; thus, they do not affect the total income. The life insurance proceeds received by reason of death of the insured are excluded from income. The IRA contribution is a potential adjustment to income, and does not affect the total income. Job-related moving expenses are only deductible for active duty military personnel who are undergoing a change of station.

Which of these is the main advantage of using the accrual basis of accounting? A) A deduction may be taken for an expense not yet paid. B) Total income recognized is generally less than under the cash method. C) The constructive receipt doctrine defers income. D) Income is recognized before it is received.

Total income recognized will be the same under either the cash or accrual method. The timing of the income recognition may be different between the two methods, but the total will be the same over time. The constructive receipt doctrine is typically relevant to the cash method of accounting, and it accelerates, not defers, income.

A taxpayer currently is being audited by the IRS, and the agent has proposed a tax deficiency with which the taxpayer does not agree. The client has asked you to research the issue. Which one of the following sources is considered to be the most authoritative and, accordingly, would have the highest precedential value in defending the taxpayer's position to the IRS? A) Private Letter Ruling B) Revenue Ruling C) Revenue Procedure D) Treasury Regulations

Treasury Regulations have the full force and effect of law. A Private Letter Ruling is never precedential, and Revenue Rulings and Revenue Procedures are merely administrative interpretations of the statutory tax law with lower authority than Regulations.

Dion and Issa are married filing jointly. Issa receives compensation from her employer who deducts federal withholding taxes and remits them to the IRS. Dion is self-employed and made only one quarterly deposit of his federal taxes this year. At the time of filing, the couple owed an unpaid tax liability equal to two-thirds of what Dion should have remitted to the IRS. What does this mean for the couple? 1.) Issa and Dion owe the remaining tax liability jointly and the IRS may require either, or both to pay it. 2.) If Issa is unwilling to pay the tax liability based on Dion's earnings, she automatically qualifies for innocent spouse relief. A) II only B) Both I and II C) I only D) Neither I nor II

Under the Tax Code, spouses who file a joint income tax return have joint and several liability for the payment of the taxes owed, no matter who the unpaid balance may be attributed to. Innocent spouse relief is granted only if certain conditions are met and is not granted automatically.

Which of the following methods of accounting is mandatory for businesses that maintain inventory and have gross receipts in excess of $27 million? A) Hybrid method of accounting B) IRS method of accounting C) Accrual method of accounting D) Cash method of accounting

Under the cash method of accounting, a taxpayer generally reports income when any cash is collected (or the constructive receipt income tax doctrine applies) and reports expenses when any cash payment is made. The accrual method is the conceptual opposite of the cash method and requires recognition of taxable income in the same tax year it is reported on the taxpayer's financial statements; it is mandatory for businesses maintaining inventory and have gross receipts in excess of $27 million. The hybrid method of accounting is a combination of the accrual method and the cash method of accounting. There is no IRS method of accounting.

For which of the following transactions does immediate recognition of remaining gain on an installment sale occur? 1) Blake entered into an installment sale with Jake when he sold Jake a motorcycle but canceled the installment sale one year into the three-year installment sale term. 2) Sarah entered into an installment sale with her sister, Josie, when she sold her an undeveloped lot but gifted the note to Josie three months later. 3). Josh needed cash immediately and sold his installment note he had on equipment he sold to Trevor to the bank. 4) Les needed a loan from the bank and used the installment notes he had for cars he sold to three customers as collateral for the loan. A) I, II, III, and IV B) IV only C) I and III D) II and III

Under the installment sale method, immediate recognition of remaining gain occurs with the following: -At the time an installment sale is canceled -When there is a gift of an installment sale to the obligor-debtor -when there is a sale of the installment note to another party -when an installment note is pledged as collateral for a loan If an installment obligation is canceled, it is treated as a disposition. To discourage sham transactions between related parties, when an installment sale is canceled, gain is immediately recognized.

Which of the following benefits that Claudia has received from her employer can be excluded from taxation? A) $5,000 of graduate education assistance. B) A company car that she uses for personal vacations. C) A year-end bonus. D) An athletic membership at a local club valued at $1,500 per year.

Undergraduate and graduate education assistance is excluded from an employee's income in any one year period, up to a maximum of $5,250. The other options are fully taxable.

Paul, age 16, is listed as a dependent on his parents' income tax return. During 2022, he earned $2,700 from a summer job. He also earned $2,700 in interest and dividends from investments that were given to him by his uncle five years ago. How much of Paul's income, if any, will be taxed to him in 2022 using his uncle's marginal tax rate of 32%? A) $0 B) $400 C) $2,700 D) $2,000

When applying the kiddie tax, the parents' marginal tax rate is always used (regardless of the source of the property generating the unearned income). Therefore, none of the income is taxed to Paul using the uncle's tax rate. The $400 of income ($2,700 − $2,300) is taxed to Paul at his parents' marginal tax.

Amy, age 12, is claimed as a dependent on her parents' income tax return. During 2022, she earned $2,300 from a summer job. She also earned $2,600 in interest and dividends from investments that were given to her by her grandfather five years ago. How much of Amy's income, if any, will be taxed to her in 2022 using her grandfather's marginal tax rate of 32%? A) $300 B) $2,700 C) $0 D) $2,300

When applying the kiddie tax, the parents' marginal tax rate is always used (regardless of the source of unearned income). Therefore, none of the income is taxed to Amy using the grandfather's tax rate. $300 of income ($2,600 − $2,300) is taxed to Amy at her parents' marginal tax rate.

When spouses who are legally married use the married filing jointly filing status, what is their liability for the combined income tax owed? A) Each spouse has liability for the tax due in proportion to their earnings reported on the return. B) Each spouse owes 50% of the tax due. C) Each spouse has joint and several liability for payment of the entire tax. D) A nonworking spouse has no liability for the tax owed.

When filing in this manner, spouses have joint and several liability for the payment of tax, meaning each spouse is responsible for the entire tax and not just half.

Jeff received 100 shares of stock from his aunt, Diane. Diane purchased the stock seven years ago for $12 per share. Assume that Jeff received the stock as a gift from Diane two years ago, when the fair market value was $8 per share, and assume that he sold the stock this year for $6 per share. What was Jeff's per-share basis in the stock? A) Basis cannot be determined B) $8 C) $6 D) $12

When the fair market value on the date of the gift is less than the donor's basis in the asset, the donee's basis in the asset for purposes of determining a loss is the asset's fair market value on the date of the gift. In this situation, the $8 per share value on the date of the gift would be Jeff's basis.

Paul was divorced from his spouse, Pat, late in 2022. As part of the property settlement agreement, Paul agreed to transfer his interest in a residential rental property to Pat in exchange for release of marital claims. Paul's cost basis in this real estate tract was $50,000. The tract was appraised at a fair market value of $100,000 at the time of its transfer to Pat. Which of the following is an income tax implication of Paul's transfer of the real estate tract to Pat? A) Paul's basis in the real estate is carried over to Pat for income tax purposes. B) Pat receives a basis in the real estate equal to the fair market value at the time of transfer. C) Paul is allowed a deduction equal to the excess of the fair market value over his basis in the property. D) Paul must recognize the gain on the real estate at the time of transfer as ordinary income.

When there is a transfer of property incident to divorce, the basis simply carries over to the other spouse. The transfer is not a taxable event. Pat will likely owe capital gains tax when disposing of the property.

Anne is the owner of 1,000 shares of the GRQ Mutual Fund Group. Recently, because of favorable market conditions, she sold 1,000 shares of the fund for a market price well in excess of her adjusted cost basis. She is now interested in computing her gain on these shares. Anne uses the specific identification method to determine basis. Which of the following is an income tax implication for Anne in the sale of her mutual fund shares? A) The gain is computed by subtracting the documented basis of identified shares from their sale price. B) The gain is computed using the excess of the fair market value over the cost of those shares last purchased. C) The gain is computed according to the average cost of all shares as compared to the total sale proceeds. D) The gain is computed using the excess of the fair market value over the cost of those shares first purchased.

When using the specific identification method of computing basis in the sale of mutual fund shares, it is necessary to identify the shares being sold based on their purchase date. The documented basis of those shares is then used to compute the gain (or loss) realized.

Which one of the following is a characteristic of a fixed annuity contract? A) If a corporation owns the annuity contract, the earnings are not tax deferred. B) The buyer may choose among a handful of investment options. C) Fixed annuity contracts are not tax advantaged, unlike other annuity contracts. D) The annuitant pays now for future fixed or variable payments.

With a fixed annuity contract, there is no ability to select the investment options; the payments are fixed. Fixed annuity contracts are generally tax advantaged (tax deferred), unless a corporation owns the annuity contract, in which case the earnings are currently taxable. Such is also the case with a variable annuity.

Which of these forms of business is generally recognized as having the most readily available access to additional capital? A) S corporation B) General partnership C) C corporation D) Sole proprietorship

With the ability to access capital from a potentially large number of shareholders as well as borrow from banks, C corporations are generally recognized as having better access to capital than any other form of business.

Cash value life insurance is often structured like an investment vehicle. However cash value life insurance contains important features that shelter the inside buildup from taxation. Which of the following will NOT be considered when determining whether a policy can maintain its tax favored status? A) The guideline premium and corridor test B) The death benefit C) The earned premium test D) The cash value accumulation test

Without a death benefit, a contract does not meet the legal definition of life insurance. There are currently two tests—only one of which must be met—in order to classify a product as life insurance for federal income tax purposes: (1) the cash value accumulation test and (2) the guideline premium and corridor test. There is no earned premium test.


Ensembles d'études connexes

Chapter 8 The cardiovascular system

View Set

Requirement That No Defense Exist

View Set

Chapter 43: Nursing Care of the Child With an Alteration in Urinary Elimination/Genitourinary Disorder

View Set

Chapter 5 - Membranes and Transport

View Set

Women's health/Disorders & Childbearing

View Set

Algebra (8th Grade) - Proportional Relationships -

View Set